Search
Menu

NASA Spaceflight Q&A

featured image

Ever wondered…

  • How do astronauts perform various daily tasks in microgravity?
  • How do the power and life support systems onboard the International Space Station work?
  • What do astronauts eat during space travel?
  • What effects do microgravity and radiation have on the human body during space travel?
  • What do astronauts do in their spare time while traveling in space?

Well, you’re in luck!


For the 2018-2019 season of FIRST LEGO League, the theme was “Into Orbit”, and teams were tasked with identifying “a human physical or social problem faced during long duration space exploration within our Sun’s solar system and propose a solution”. Teams had to do research surrounding this identified problem, design a solution, and share with others. Part of both the ‘research’ and ‘share with others’ elements involved many teams reaching out to NASA to ask subject matter experts (SMEs) questions they had involving challenges surrounding long duration spaceflight. The Robotics Alliance Project was integral in helping countless teams get their questions answered and their proposed solutions reviewed by NASA SMEs, as well as providing online resources to support their research. RAP is happy to share those Q&As and online resources with the world now – enjoy.

Disclaimer: The information provided here was accurately determined by NASA SMEs (Subject Matter Experts) at the time the answers were provided. Some answers have been combined together from more than one SME to provide a more complete response to the asked question. This information may no longer be accurate based upon new findings, procedures, etc. The opinions presented are that of subject matter experts, and not NASA as a whole. There may be counterarguments within NASA not expressed here.

Q&A about Environmental, Safety, & Resource Challenges in Space

Where do astronauts get the oxygen and water they need when they are on-board a spacecraft or the ISS?

Most of the oxygen comes from the Oxygen Generation System (https://www.nasa.gov/home/hqnews/2006/feb/HQ_06061_oxygen_ISS.html) which uses a process called electrolysis to split water molecules into oxygen and hydrogen. We can also deliver oxygen and other breathing gases aboard cargo ships, either in tanks on the Russian Progress or in special high-pressurized vessels called NORS (Nitrogen-Oxygen Recharge System) tanks delivered on U.S. commercial cargo ships. (NORS story and picture (https://www.nasa.gov/content/air-supply-high-pressure-tanks-ready-for-space-station).
Water is delivered on cargo spacecraft and transferred over to the station systems. We also use the Water Recovery System (WRS) (https://www.nasa.gov/mission_pages/station/behindscenes/waterrecycler.html) to recycle water onboard, collected from astronaut urine, condensation and other sources. Astronauts can access the water from potable water dispensers in the Unity module. Either cold or hot water can be dispensed, depending on what the water is for or what type of food they are re-hydrating.
https://www.water-technology.net/projects/iss_water_recovery/
See links on Life Support Systems: https://www.nasa.gov/content/life-support-systems
https://www.nasa.gov/centers/marshall/news/releases/2015/space-station-teaches-nasa-valuable-lessons-about-life-support-systems.html
Daniel Huot — Public Affairs Officer – International Space Station NASA JSC

How is trash and human waste disposed of in space?

Astronauts collect trash then store it in bags and place it in the logistics module of the International Space Station. The trash is stored until a vehicle is ready to depart from the International Space Station and burn up in Earth’s atmosphere.
This video demonstrates that process: https://t.co/DaS7BlPoMy
This is also a good video explaining “taking out the trash” in space: https://www.youtube.com/watch?v=JP5D0QK88Lw
In this video Suni Williams explains the bathroom in space pretty well: https://www.youtube.com/watch?v=5WSlGRBTFNI
Chris Hadfield talks about water and recycling urine: https://www.youtube.com/watch?v=ZQ2T9OJY1lg
This video gives some facts about astronaut solid waste: https://www.youtube.com/watch?v=E4j72djA2DE
Explanation and semi-demonstration of the WCS (Waste Collection System) at NASA: https://www.youtube.com/watch?v=0xI6coAS85c
Astronaut Clayton Anderson explains that restroom odors on the ISS are not a problem: https://gizmodo.com/how-nasa-deals-with-odor-inside-the-international-space-1648864449
Here is information about a proposed Torrefaction Process system to recycle solid human waste. The torrefaction process is not yet being used in space, but we are still working on it. A phase 2 SBIR is just finishing up and we will be looking for ways to integrate it into a future flight experiment. -Michael Ewert @ NASA JSC: https://ntrs.nasa.gov/search.jsp?R=20160003882
Other sources:
https://space.stackexchange.com/questions/26740/what-is-in-isss-waste-product-brine-and-how-is-it-disposed
https://www.nasa.gov/pdf/146558main_RecyclingEDA%28final%29%204_10_06.pdf
https://www.theguardian.com/science/2017/aug/22/srecycled-astronaut-urine-could-make-nutrients-and-tools
https://www.space.com/39710-orion-spacesuit-waste-disposal-system.html
Info about the UPA (Urine Processor Assembly): https://www.nasa.gov/mission_pages/station/research/experiments/1796.html
https://ntrs.nasa.gov/archive/nasa/casi.ntrs.nasa.gov/20110012703.pdf
https://ntrs.nasa.gov/archive/nasa/casi.ntrs.nasa.gov/20160009716.pdf

What are the challenges and risks involved with space debris, and how do we manage and mitigate those risks?

Orbital debris can actually be catastrophic when impacting another piece of debris. One catastrophic collision could cause a chain reaction that destroys pretty much everything in orbit. So we actually track in real time every piece of significant debris currently in orbit and give all rockets launch windows so that the debris doesn’t collide. We have gotten by with just these procedures but as space flight becomes more common, the demand for a clean-up device will increase. Clean-up isn’t easy because each piece of debris is moving at around 18,000 mph.
The hull of the ISS is covered in a thick blanket of Kevlar and other tough insulating materials to help protect the ISS (and astronauts) from both radiation and damage from very small, and therefore currently un-trackable, but crazy fast-moving, space debris.
https://www.nasa.gov/mission_pages/station/news/orbital_debris.html
https://www.orbitaldebris.jsc.nasa.gov/
NASA’s Micro-Meteroid and Orbital Debris Protection Design Handbook: https://ntrs.nasa.gov/archive/nasa/casi.ntrs.nasa.gov/20090010053.pdf
https://www.hq.nasa.gov/office/hqlibrary/pathfinders/debris.htm
https://www.nasa.gov/directorates/spacetech/niac/gregory_space_debris_elimination.html
https://www.nasa.gov/image-feature/researching-how-best-to-remove-space-junk
https://www.nasa.gov/mission_pages/station/research/experiments/2456.html
https://www.airspacemag.com/daily-planet/first-test-space-debris-cleanup-about-get-under-way-180968631/
Joe Altemus — Mechanical Design Engineer @ JSC

This page discusses the mission of the ‘NanoRacks-Remove Debris’ satellite & describes the demonstrations it would perform. Formal results from the September demonstration should be posted to the page once they are ready:
https://www.nasa.gov/mission_pages/station/research/experiments/explorer/Investigation.html#id=7350
Shows the demonstrations in video: https://www.surrey.ac.uk/surrey-space-centre/missions/removedebris
Discusses RemoveDEBRIS sent up to the ISS via SpaceX CRS-14 launch in April 2018 (even though the demo did not occur until September). So this prototype and proposed solution has been public knowledge since April: https://www.surrey.ac.uk/news/removedebris-deploys-international-space-station-begin-its-mission
https://news.sky.com/story/removedebris-satellite-launched-to-capture-dangerous-space-objects-11316216
Information from European Space Agency on their projections of the space debris dilemma and why a solution is necessary sooner rather than later: https://www.esa.int/Our_Activities/Operations/Space_Debris/Active_debris_removal
From what I could tell, the NanoRacks-RemoveDEBRIS (as of right now) is not reusable and does not return to the ISS on its own. Whether or not that will ultimately be the intention, I’m not sure. I would imagine the plan is for the capsules to both be reusable and return to the ISS (or other spacecraft) on their own. It appears with all 4 methods the debris is intended to fall out of Low Earth Orbit and burn up upon re-entry into Earth’s atmosphere (as opposed to the debris being brought to the ISS). The fate of the RemoveDEBRIS capsule itself, once it has deployed all the debris-removing mechanisms, is unclear. I would imagine at this point in time it will also change paths and head towards Earth to burn up. I would assume that is what was done with this demo capsule, unless they have a reason to keep it up there in LEO for now.

How do astronauts deal with the food packaging waste? How much waste packages is created per meal? Does NASA have packing that is re-useable during a trip to space? What do you do with leftover food?

Astronauts collect the food packaging waste in bags, compress the bag until it’s full, then store it in the logistics module. The trash is stored until a vehicle is ready to depart from the International Space Station and burn up in Earth’s atmosphere. When an astronaut finishes a meal, the entire package is waste. NASA does not currently have re-usable packaging, but it is being looked in to for future missions beyond the space station, such as bulk packaging for Missions to the Moon and Mars. There isn’t much leftover food on missions, but what is leftover becomes waste.
Anne Meier — NASA Labs, Development & Testing Division: Analytical Laboratories @ NASA-KSC

What type of screening does equipment go through in order to be eligible to be used in space? (exercise equipment, entertainment, etc)

There are many analyses and reviews that are part of a screening process which permit equipment to be flight certified. I’d like to refer you here: https://nodis3.gsfc.nasa.gov/OPD_docs/NID_7123_69_.pdf (Just looking at the table of contents could give you an idea). I am early on in my career but can share the insight I have. Conducting Hazard Analyses (HAs) and other analyses are probably the most important ways to ensure that the design is safe (for itself, for humans, and for the spacecraft). The ISS is one big, closed off, integrated system and so there are a lot of factors to consider. For instance, with exercise equipment: designs must consider where the equipment will be placed, then vibration analyses conducted (the microgravity allocations can change depending where you are at and the surrounding equipment which could be affected), then vibration isolation systems can be designed. Is power needed? If so, how can it be safely integrated and use available power? Does it need to communicate/transmit data? If so, will it be tethered, wireless, or transferred to a computer later? Will it use existing resources for down-linking information? What timeline allows it to fit in with the rest of the crew’s schedule (including setup and use)? When will it fly up? Will it need to return to Earth as well? Will it need maintenance? Is foreign object debris of concern? What about microbes? All these factors and more are considered to make equipment eligible and used in space. Sometimes the process is streamlined such as for innovation type projects, but all those points are still covered. In the end, you need working equipment too! Which often means producing a prototype, engineering development units (which go through testing in ground laboratories, microgravity simulators, and even parabolic flights), and then flight units. Ground testing might incorporate not only use of the equipment but also vibration, thermal/vacuum, and other testing. Hope that helps! If you’re looking to incorporate the process this season, that’s awesome! While working on the robot, consider what prototypes got you to the design you have today and the testing you went through! What have you done to ensure mission success? Keep on the lookout for opportunities for students to send items to space as well: https://www.nasa.gov/audience/foreducators/stem-on-station/get-involved
Kaitlin Lostroscio — Robotics: Digital Astronaut Simulation & Exercise Countermeasures Support (JSC)

What biomedical research is currently going on on the ISS?

There are so many biological experiments going on up here right now, certainly the direct DNA sequencing that I mentioned. We are also doing RNA sequencing, which is not an easy task, but we think we’ve found a way to do it up here on the ISS, which really revolutionizes how we perform occupational health up here. Other sorts of medical experiments that we’re doing… So we’ve got a special microgravity science glovebox that we do a lot of experiments in, where we’re able to cell culture, media exchange,.. We’ve also done some reproductive studies… We’ve looked at bovine sperm motility, as well. But when you look at biotech, one of the things I always like to bring up is what we call our region ecosystem. Every day, any thing that any astronaut or cosmonaut urinates out, we turn in to water. And that is important, because that is something we are going to carry forward from here to live on The Moon or live on Mars. You have to have that capability. So every day, the way we generate oxygen, the way we recycle urine and turn it in to water, the way we capture any kind of condensate from the air itself… Anytime I breathe out, that condensate, that humidity, is captured by the system, and utilized. You kind of think of how wasteful we are on the Earth. Well, up here, you can’t be wasteful with one thing. We are always looking at our water balance, always looking at what we need to put in the system, what we need to take out, and we’ve gotten pretty good at it!
Source: https://www.youtube.com/watch?v=LVCW_ubmwJ0&index=2&list=PLTUZypZ67cdv9X4DxjUGvN4wfN6EdHTUo
Astronaut Serena M. Auñón-Chancellor (M.D.)

Can you see space debris, and what do you do about it?

> Joe Acaba: Hopefully we wont see it. If it’s big enough that we can see it (people are tracking the debris for us), we can change the ISS’ orbit. A couple times a year the ISS orbit has to change to avoid objects that are large. The smaller objects, we see evidence of that, on spacewalks you might see dings on handrails, the cupola windows have been dinged, but the glass is very strong. (https://gizmodo.com/this-is-the-last-thing-you-want-to-see-when-looking-out-1776210630 )
> Mark Vande Hei: I saw the same evidence during spacewalks that Joe mentioned- dings on the handrails, dings on the windows of the cupola. An Italian astronaut that was onboard with us during the first half of our flight, he took a lot of pictures and captured some natural space debris interacting with the atmosphere. He actually saw the debris only when it started burning up, and you could see the flash of light as it burned up. It’s pretty cool to see!
Source: https://www.youtube.com/watch?v=K0l1RJ4Aag8
Astronauts Joe Acaba and/or Mark Vande Hei

As deep space comm moves from RF to light (laser/optical), will round-trip times decrease, or does optical only allow the signal to carry more data?

RF, light, laser, radar, are all examples of electromagnetic waves and are governed by the same laws of physics or electromagnetics. So when we say the speed of light, it applies to both optical and RF communications. If you search for “JPL’s Beam waveguide antenna” you can see how RF can be treated very similar to light with the use of mirrors. Optical communication does not decrease the round-trip time of communication; Optical comm only increases the amount of data (or bandwidth). Quantum communications may or may not be limited to those principles…the future will hold the answer.
Philip Baldwin — Network Operations Manager @ NASA’s HQ

What resources are on Mars that we can live off of?

You might be surprised! There’s a lot of resources on Mars, as it turns out. There’s a lot of frozen water, or ice, that is under the surface. It’s actually kind of insulated by the topsoil, which keeps the ice there. There’s a lot of uses for water, or ice. Obviously, our astronauts can drink it, they can water their plants with it, … We can use a process called electrolysis where we split the water into Hydrogen gas and Oxygen gas, and Oxygen gas is useful for our astronauts’ breathing air. And the Hydrogen, if you turn it into a liquid, and you turn the Oxygen into a liquid, both of those are actually really good rocket propellants.
The atmosphere on Mars is made up of mostly Carbon Dioxide. Carbon Dioxide, if you look at it chemically, there’s a lot of Oxygen in that molecule. You can separate the Oxygen from the CO2 and use it for breathing air. But also, using the magic of chemistry, we can actually turn the Carbon Dioxide and the water that I just mentioned above, into rocket fuel, which is a different kind of rocket fuel (a methane rocket fuel vs the Hydrogen that I mentioned before).
The rocks and the dust on Mars is actually a great resource that we can use when he have humans living and working on Mars. We can use them for building materials. There are also metallic elements and compounds that are embedded in the rocks and dust. We can use these metallic elements and compounds for 3-D printing and building of parts.
Of course in order to use these resources, we first have to find these resources. This is where robots come in… Find the resources, for example on Mars, is called prospecting. So prospecting means looking for resources, probing, sensing, … So the rocks and the dust, we don’t really have to look for; they’re basically all over the surface of Mars. The air and the atmosphere, once you land something on the surface of Mars you have the atmosphere all around it, so you don’t have to look for the Carbon Dioxide. But the water/ice is actually hiding from us. It’s under the surface, and it’s insulated by the soil and the dust on top of it. So we need to find the water, and we need to also find out what form it’s in. Is it in a liquid form, at a certain depth underneath the surface because it’s insulated so well? Is it more like a soft frost, or like a snow? Is it solid chunks, like rocks, underneath the surface? Is it solid sheets, maybe? We’ll have to send the right equipment based on what form the water, or ice, is in. And also, we need to find out the distribution or the location of the water, or ice. Is it spread out? Is it thick? Is it thin? Is it concentrated? Where exactly is it, because we want to send our equipment to the right location. So how do we find or pinpoint the right location, in order to maximize our yield when we’re needing to pull these resources from the surface of Mars?
http://www.ustream.tv/recorded/117572142
Kurt Leucht — Software Engineer @ KSC

How do we prospect resources on planets, like Mars?

There are lot of different ways we can prospect on a planet, like Mars.
The first way is to do remote sensing from orbit. This utilizes a neutron spectrometer instrument in order to detect ice near the surface of Mars. This instrument’s sensor actually penetrated the surface of Mars, up to 1 meter below the surface, in order to come up with the readings of sensing the presence of water or ice below the surface. This is typically the first process we go to when we start looking for resources on another planet. https://mars.nasa.gov/odyssey/mission/instruments/grs/ https://www.jpl.nasa.gov/spaceimages/details.php?id=PIA01543
Another way is to actually send down an impactor, something that’s actually going to crash into the surface, and then you use remote sensing to look at the cloud that basically comes up from that impact. https://www.nasa.gov/mission_pages/LCROSS/main/SSC_impacts.html
Another thing you can do is send a lander that has arms, like Viking I and II (see section “1” of this article: https://medium.com/@tanyaofmars/mars-phoenix-lander-10-years-later-8b90a8609a68 ). The second photo featured in section “1” is from NASA’s Phoenix Mars lander back in 2008. It landed near the polar cap of Mars. In the far left photo, in the lower left corner, there are some small, what look like rocks, and those small rock-looking things are not in the photo from 4 days later (Sol, or day, 20 vs Sol 24). So scientists are pretty sure that what look like rocks are actually ice, and that over those 4 days on Mars, or four “Sol”s as we call them, the ice disappeared, it evaporated, it sublimated. This was just a trench that was dug by the Phoenix Mars lander using a digging arm. So sending a robotic lander that has arms is another way.
A different way is to send a wheeled robot, for example, Curiosity, that drives around Mars. While it drives, it has a sensor that looks down and can penetrate the surface. This sensor- it’s a neutron detector- it detects Hydrogen or water/ice that is under the surface. https://marsmobile.jpl.nasa.gov/msl/mission/instruments/radiationdetectors/dan/
A different way, which is a concept we haven’t flown yet, is to send a wheeled robot that drills down under the surface, and takes a core sample from under the surface. What we’re planning on doing someday soon is sending a robot with a drill to The Moon to perform a core sample near the poles of The Moon, and then heat up that core sample and drive off the water molecules, and collect those water molecules just to prove whether or not there is water under the surface of The Moon, which we think there is, especially near the poles. And once we’ve proven this technology on The Moon, we plan to eventually send this sort of technology to Mars. https://www.nasa.gov/resource-prospector Another way we can look for a resource, like water, on Mars is to send a flying robot. The water you’re looking for may be in a hard-to-reach area, where a lander may not be able to reach, or a roving robot may not be able to reach. For example, in the very bottom of a very deep crater that has very steep walls, you may not be able to drive a rover down into that deep crater. So a flying robot could get into hard-to-reach area such as deep craters or caves. The sensors you would put on a robot like this would basically be the same sensors or samplers that you have on a wheeled robot. A flying robot needs to stay very lightweight, obviously, so you might leave your chemistry equipment on the lander and only have a very smaller sampler on the flying robot that picks a small sample and then flies back to the lander, deposits that sample into the lander for processing. https://www.nasa.gov/press-release/mars-helicopter-to-fly-on-nasa-s-next-red-planet-rover-mission.
Another way you could prospect on Mars, The Moon, or wherever you want to prospect, is to send a swarm of small, inexpensive robots. This is a concept and hasn’t turned into a real mission yet, but someday if NASA has a need to search a large area, they might want to send a swarm of small robots, rather than one large robot. If you send a swarm of robots, one cool benefit is that if some of your robots fail or break, the rest of your robots can continue, and your mission isn’t a failure. A swarm of robots can actually work together and cover a large area. http://www.ustream.tv/recorded/117572142
Kurt Leucht — Software Engineer @ KSC

What is the size of the living quarters on ISS?

The astronauts’ living quarters, or “cabin”, on the ISS is about the size of a refrigerator or a small closet. See videos:
https://images.nasa.gov/details-Expedition_56_Education_Interview_with_Microsoft_Youth_Spark_Live_2018_0813_689813.html
https://images.nasa.gov/details-Expedition_56_Education_Interview_with_McAuliffe_Shepard_Discovery_Center_2018_0809_688241.html
https://www.youtube.com/watch?v=tBVUTFPate0
Expedition 56 Flight Engineer (Astronaut) Ricky Arnold

What is the biggest catastrophe caused by space debris? How was the problem solved?

There have been two major space debris catastrophes to date. The first was when the Chinese used a ballistic interceptor to target and destroy their own defunct Fengyun-1C meteorological satellite on January 11, 2007. The second was the accidental collision of an operational US communications satellite, Iridium 33, and a decommissioned Russian communications satellite, Cosmos 2251, on February 10, 2009. Each incident created thousands of debris fragments and collectively currently account for a large percentage of the debris in Earth orbit.
Unfortunately, we don’t have any effective ways to remove orbital debris yet. At low altitudes, the Earth’s atmosphere helps clean the orbital environment by way of drag, which slowly lowers the altitude of orbiting objects over time and causes them to eventually enter the lower atmosphere and burn up.
To keep our spacecraft and astronauts safe, people at Johnson Space Center and Goddard Space Flight Center operating our crewed and robotic spacecraft work with the Air Force to predict when possible collisions may occur. If the probability of collision becomes high enough, the spacecraft operator may choose to perform a maneuver to slightly change the orbit to avoid the object. International Space Station flight controllers have performed several of these collision avoidance maneuvers over the lifetime of the space station.
See “Threats” section: https://en.wikipedia.org/wiki/Space_debris#Threats
Tim Stuit — Exploration Flight Dynamics Officer (JSC)

Do you ever have problems with spacesuits? If so, what problems?

The current spacesuits are at a point where they are workable in all the circumstances that we need them for. The biggest issue we have is the poor dexterity each crew member has in the suits. Years ago the suits used to be custom made for each crew member. This gave each crew member a much more comfortable time in their suits but was extremely expensive because everyone needed their own set of suits. Now the suits are a one size fits all from the 95% man to the 5% female (this means as you go up in percentile you are encompassing taller/larger people of each gender). Because of this large range of sizes we had to take some liberties in the design. The gloves are almost always going to feel big and the suit will almost always feel big as well. All this being said, we are pretty comfortable with where the suits are at as far as ISS operation. When the time comes to do a mission to another planet or to the moon there is a good chance that the suit design will be looked at again.
Joe Altemus — Mechanical Design Engineer @ JSC

How fast and how high do you go when on the ISS?

The space station travels through space at 17,500 miles per hour at an altitude of about 220 miles. We orbit the Earth about once every 90 minutes. During the orbit of the Earth we are in daylight for about 45 minutes and darkness for about 45 minutes. That means the sun will rise and set 16 times a day. You can see the space station from where you live at certain times. Visit http:// www.jsc.nasa.gov/sightings/index.html to find out when you can see the ISS.
Source: https://www.nasa.gov/centers/johnson/pdf/569954main_astronaut%20_FAQ.pdf
various astronauts

How long are the missions onboard the ISS?

The ISS missions, called expeditions, usually last about six months. There are three to six crewmembers on board at all times. Professional astronaut crews come from the U.S., Russia, Japan, Canada and Europe. NASA astronaut Mike Lopez-Alegria has flown the longest U.S. space station mission to date, at 215 days. Valeri Polyakov, a Russian cosmonaut, spent 437 days in space during one mission. A space shuttle crew is typically five to seven crewmembers. We have carried as few as two and as many as eight at one time, and space shuttle missions have been as short as two days and as long as 18 days.
Source: https://www.nasa.gov/centers/johnson/pdf/569954main_astronaut%20_FAQ.pdf
various astronauts

What does a launch feel like?

In a Russian Soyuz capsule, which is how astronauts who are part of expedition missions get to the space station, three people are crammed into a small space. The crew gets in the Soyuz about two and a half hours before launch. Once the crew is all strapped in, they perform a series of pre-flight checks of all the Soyuz systems. When the checks are finished, the crew waits while workers on the launch pad do the final rocket preparations.
Shortly before the time of launch you start hearing different noises below you and you know things are getting ready to happen. Then, it is as if a giant beast is waking up. You hear and feel the thumping and bumping of valves opening and closing as engine systems are pressurized. When the first engines light there is a terrific low frequency rumbling and things start to shake. Then the main engine lights and the rumbling and shaking get even louder. Slowly, slowly you begin to move up and away from the launch pad. But, very quickly you build up speed and the g-load, or the force of gravity or acceleration on a body, increases. You shake and rattle along and then there is a bang when the rescue system is jettisoned, another bang when the four strap on boosters separate, and another bang when the nose faring comes off. Now the windows are uncovered and you can see light coming in. At the second stage separation there is another bang and the g-load drops immediately. You go from about four and a half g’s down to about one and a half or two g’s. Then the third stage engine lights; you have a big push forward and the g-load builds again.
Eight and a half minutes after launch there is a loud bang and jerk and the last section of the rocket is jettisoned from the Soyuz spacecraft. And just like that, you are there–in space. It feels like you are hanging upside down in your shoulder harness. This is simply because there is nothing pushing you back into your seat anymore. Everything floats, including you.
In the space shuttle, astronauts were strapped in on their backs a few hours before launch. As the main engines lit, the whole vehicle rumbled and strained to lift off the launch pad. Seven seconds after the main engines lit, the solid rocket motors ignited and this felt like a huge kick from behind. The vehicle shaked a lot and the ride was rough for the first two minutes as you were pressed back into your seats with twice your weight. When the solid rocket motors burned out there was a big flash of light as they separated from the big fuel tank the shuttle was strapped to. Then the ride smoothed out. As you got higher into the thinning atmosphere and burned off most of the fuel, the vehicle accelerated faster and you were pressed back into your seat with three times your weight for the last two and a half minutes of the ride. This two and a half G’s felt like a giant gorilla was sitting on your chest making it more difficult to breathe. Eight and a half total minutes after liftoff, the main engines stopped and immediately you went from being squashed by the gorilla to being weightless.
Astronauts practice for launch frequently during training. We work in a simulator that mimics the noise and shaking of launch, but cannot mimic the acceleration and speed of a real launch. We are busy monitoring the shuttle systems during launch, but still have time to enjoy the feeling of going from 0 miles per hour to 17,500 miles per hour in eight and a half minutes.
Source: https://www.nasa.gov/centers/johnson/pdf/569954main_astronaut%20_FAQ.pdf
various astronauts

How do astronauts sleep in space?

The astronauts’ living quarters, or “cabin”, on the ISS is about the size of a refrigerator or a small closet. It is both sound and light-proof if they completely shut the door. They have a canvas sleeping bag that is strapped to a wall at one point so the astronaut doesn’t drift around the cabin or change orientation while they sleep. There are arm holes and a zipper, so the astronaut can be partially or fully zipped in, with their arms tucked inside or free-floating.
See videos: https://images.nasa.gov/details-Expedition_56_Education_Interview_with_Microsoft_Youth_Spark_Live_2018_0813_689813.html
https://images.nasa.gov/details-Expedition_56_Education_Interview_with_McAuliffe_Shepard_Discovery_Center_2018_0809_688241.html
Expedition 56 Flight Engineer (Astronaut) Ricky Arnold

Other sources: https://www.youtube.com/watch?v=tBVUTFPate0
https://www.space.com/35293-how-astronauts-sleep-in-space.html
https://www.youtube.com/watch?v=UyFYgeE32f0
https://www.nasa.gov/feature/ames/nasa-research-reveals-biological-clock-misalignment-effects-on-sleep-for-astronauts
https://www.nasa.gov/mission_pages/station/research/experiments/892.html
https://www.nasa.gov/audience/foreducators/stem-on-station/learning_launchers_got_rhythm
https://www.nasa.gov/mission_pages/station/research/experiments/307.html
https://ntrs.nasa.gov/archive/nasa/casi.ntrs.nasa.gov/20030068197.pdf
https://www.nasa.gov/content/study-compiles-data-on-problem-of-sleep-deprivation-in-astronauts
They also use a system called “Actiwatch” that is a wristband. Read more here: https://www.nasa.gov/mission_pages/station/research/experiments/858.html

How do astronauts prep for space walks?

Astronauts go through extensive training on Earth prior to their missions, and go through every planned out detail of their scheduled spacewalk(s). While on the spacecraft, prepping for their spacewalks, they have to get their suit ready, which takes a lot of time. The suit is basically like a mini-spacecraft with all the protection, padding, and safety mechanisms. They will rehearse the spacewalk, as a team, as well. Since the suit is so heavily inflated for the astronaut’s protection, the astronaut feels a lot of resistance when they move in any way. This makes spacewalks very tiring and challenging, but they are well-trained. Daily exercise helps to prepare the astronauts, both mentally and physically, for the exhausting challenges ahead of them.
Expedition 56 Flight Engineer (Astronaut) Ricky Arnold

What is the 3-D printing capability like on-board the ISS?

NASA is excited about 3D printing capabilities for future spaceflight; the following publications describe some of our research: “3D Printing in Zero‐G ISS Technology Demonstration” at https://ntrs.nasa.gov/archive/nasa/casi.ntrs.nasa.gov/20140011720.pdf presents the reasons why 3D printing is a good technology to pursue; “International Space Station (ISS) 3D Printer Performance and Material Characterization Methodology” at https://ntrs.nasa.gov/archive/nasa/casi.ntrs.nasa.gov/20150016233.pdf presents the plan for the first printer that went to the ISS; and “Summary Report on Phase I and Phase II Results From the 3D Printing in Zero-G Technology Demonstration Mission, Volume II ” at https://ntrs.nasa.gov/archive/nasa/casi.ntrs.nasa.gov/20180002403.pdf presents a great description, including many details, by the engineers and scientists at the Marshall Space Flight Center in Alabama of much of the research that was done with the ISS 3D printer. The Resources section is a great place to get more information. Engineers at Marshal and Johnson Space Centers show their excitement before sending the printer to the ISS at https://youtu.be/1Jwxn6EzW84 and https://youtu.be/vgZymJC4a-g . For future updates to NASA’s ISS 3D Printing research, please visit https://www.nasa.gov/mission_pages/station/research/experiments/1115.html . https://www.nasa.gov/content/international-space-station-s-3-d-printer THANK YOU for being part of space exploration. -Lucien
Lucien Junkin — NASA Space Exploration Vehicle (SEV) Chief Engineer

Why are spacewalks necessary, and is there a way to make them safer for humans?

It really comes down to dexterity and how things are designed. We design things with human assembly in mind and it is extremely difficult to make robots with the same dexterity that humans have. Humans are versatile and it is much easier to teach a human how to repair a new piece of equipment than it is to create a robot that can do it. In the future, as robotics advances, we will most likely use those robots to do our space walks for us. As far as safety is concerned, we do a significant amount of work to protect our crew members. Everything we design for space has to be what we call “two fault tolerant” which means that two things have to fail for something to break. In example: to prevent a bolt from coming loose you have to install it with a specific torque and you also need to use a locking helical insert. All of this work has been successful because we have never lost a crew member or had a serious injury from a space walk.
Joe Altemus — Mechanical Design Engineer @ JSC

Astronauts have two Oxygen tanks in their suit with them when they perform ExtraVehicularActivities (space walks) as their air supply. Space walks are extremely carefully planned and executed, and the astronauts are always on a tether. They constantly monitor their Oxygen supply, just like a scuba diver constantly monitors their air supply. If the supply is running is unexpectedly low and they are not done with their spacewalk, they would abort and return to the spacecraft (in our current case, that is the ISS- space shuttles were retired years ago).
Information about the training for EVA space walks and the onboard-ISS astronaut preparations of space walks: https://spaceflight.nasa.gov/station/eva/outside.html
FAQs about EVAs: https://spaceflight.nasa.gov/shuttle/reference/faq/eva.html

What are some of the unique challenges encountered when making spacecraft repairs in micro-gravity and reduced-gravity environments?

There are a couple challenges that are unique. First, getting repair materials up to the space station take months, if not years of planning. Teams of people are involved, and the expense is great. Second, performing repairs in microgravity is typically more challenging than in a gravity field. Loose parts can float away and contaminate the environment, or become projectiles and cause further damage to your spacecraft. Although tools may basically work the same in space, you do not have “the ground” to react forces against.
Nathan Fraser-Chanpong — Electrical Engineer @ JSC

What are the risks involved with radiation exposure to astronauts, and how do we mitigate those risks?

You can learn more about the hazards of radiation and spaceflight at:
https://www.nasa.gov/hrp/hazards
https://www.nasa.gov/johnson/HWHAP/hazard-1-radiation
https://www.nasa.gov/johnson/HWHAP/radiation-shielding
https://www.nasa.gov/hrp/elements/radiation
https://www.nasa.gov/hrp/elements/radiation/risks
https://www.nasa.gov/feature/space-radiation-won-t-stop-nasa-s-human-exploration
Early Results from the Advanced Radiation Protection Thick GCR Shielding Project: https://ntrs.nasa.gov/archive/nasa/casi.ntrs.nasa.gov/20170005580.pdf
Check out pages 22-24 in particular: https://www.nasa.gov/sites/default/files/atoms/files/mars_radiation_environment_nac_july_2017_final.pdf
https://www.youtube.com/watch?v=3jRxgvwhGC0&feature=youtu.be
See links on protecting astronauts from dangerous radiation in deep space travel:
https://www.nasa.gov/feature/goddard/real-martians-how-to-protect-astronauts-from-space-radiation-on-mars
https://www.nasa.gov/directorates/spacetech/strg/nstrf2016/Passive_Radiation_Shielding
https://www.space.com/41887-mars-radiation-too-much-for-astronauts.html
Risks, monitoring, mitigation: https://www.nasa.gov/sites/default/files/atoms/files/nasa_space_radiation_ebook_0.pdf
Radiation exposure tracking:
https://www.nasa.gov/mission_pages/station/research/experiments/1043.html
https://www.nasa.gov/centers/johnson/pdf/514208main_ES-01_Passive_Dosimetry_Area_&_Crew_Monitoring_E.Semones.pdf

What are some of the challenges humans will face as we make plans to travel to and explore Mars?

One of the big challenges humans will face when exploring Mars is getting sufficient nutrition. Right now the packaged diet that astronauts eat in space is excellent, but over time vitamins degrade, and often people get tired of eating the same diet. We hope to use plants to help supplement the packed diet to add vitamins and minerals to the diet and to provide variety and new flavors. Plants will also help recycle the atmosphere, generating oxygen and taking up CO2 that humans breath out. Taking care of plants may also be psychologically beneficial for the crew, reminding them of Earth and giving them a relaxing activity. One unknown is radiation impacts on the plants as well as the crew. We do not understand the impacts of space radiation on plants and seeds, and we also don’t know how that will impact the humans. Astronauts will be exposed to radiation on the journey to Mars and on the Martian surface.
Gioia Massa — Life sciences project scientist NASA’s Kennedy Space Center

The big challenge on our minds concerned with traveling to Mars is the need to live off the land and not have to take everything with us for the long stay on the surface of Mars. Crews will live and work on the surface for a year and a half due to planetary alignments, so we will want or need to live off the land as much as we can. Living off the land will significantly reduce the launch mass and the cost. The major resource we can use on Mars is water/ice. Also Carbon Dioxide is quite easy to get our hands on. To solve that challenge of living off the land, we need to think outside the box, come up with new ideas (because we’ve not had to think that way here on earth), try those ideas out in the lab, and advance technologies to a point where NASA can fly them someday. We will need to find the water ice, we will need to dig it up because it’s hiding beneath the surface, we will need to process or separate the water molecules from the dirt or regolith, and we will need to store the extracted water. If we want to turn that extracted water into rocket fuel to send our crew back to earth at the end of their mission, we will need to separate the hydrogen and oxygen molecules and then store those gases or store them as super cold cryogenic fluids for long periods of time. These are not going to be easy tasks. Especially so far from the safety and comfort of earth. But we need to try in order to make the idea of humans living and working on Mars a reality.
Kurt Leucht — Software engineer for Swamp Works @ NASA’s Kennedy Space Center

How do humans in space communicate with mission controllers, friends, and family back on Earth?

For communication between astronauts and flight controllers, a system is used with many different channels of communication. The primary channel for the ISS to monitor is the “Space-to-Ground 1” loop, which allows the crew and flight controllers to converse. There is also the “Space-To-Ground 2” loop, which allows the ISS crew to talk to the rest of their crew. If astronauts are performing specific experiments, flight maintenance, etc. with specialists, the communication for those events takes place on different channels. When astronauts talk to friends and family, they can use e-mail, private phone channels, or even live video feeds for facetime.
Astronaut Mike Gernhardt — Astronaut; Associate Division Chief of ER (Software, Robotics, & Simulation Division) @ JSC

We have a fleet of satellites, the TDRS satellite system, which we use for commanding the ISS and real-time downlink of data, which is going on around the clock, there’s science going on here 24 hours a day, 365 days a year, with data being fed down to principle investigators all around the world, over 100 countries participating. So that fleet of satellites, the TDRS system, makes the payload data, the commanding of the ISS, and even things like downlinks with schools possible.
Sources: https://www.youtube.com/watch?v=HPjzXZwlayg&index=2&list=PLTUZypZ67cdv9X4DxjUGvN4wfN6EdHTUo&t=4s
https://www.nasa.gov/audience/foreducators/stem-on-station/downlinks-scan.html
https://www.nasa.gov/feature/goddard/2016/nasa-communications-network-to-double-space-station-data-rates
https://www.nasa.gov/feature/goddard/2017/tdrs-an-era-of-continuous-space-communications
https://www.nasa.gov/feature/goddard/2018/nasa-upgrades-space-station-emergency-communications-ground-stations
Astronaut Ricky Arnold

What systems have been used in the past, and what methods are currently used, to provide power and life support on spacecraft and space stations?

Some of the methods we have used in the past for power support are Radioactive Thermal Reactors, Solar, Fuel cells and batteries. The ISS currently uses solar arrays to power all its critical needs. Power is generated from 8 solar arrays (1 per power channel) at 160Vdc. This is then regulated at the user power level to the 124Vdc. The 8 power channels can feed up to 100kW of power to life support systems, payloads (science experiments), and critical avionics. Each channel has 3 battery sets (Li-ion has 1 battery per set and NiH2 has 2 per set). The Li-ion has 120Amp-hrs per battery and the NiH2 has anywhere from 50-80 amp-hr per battery. The voltages range from 125-110V (li-ion) and 100-80V NiH2. The wings, during the 45 minutes of each rotation that we’re in sunlight, store power, charging the batteries on the ISS. And then we drain the power out of the batteries during the 45 minutes that we’re behind the Earth, in the shade. So it’s a constant charging and draining process that provides the power to the station. In fact, due to the added electricity generated by solar power through the solar arrays completed in March 2009, the ISS crew was able to go from 3 to 6 crew members.
See links on Life Support Systems:
https://www.nasa.gov/content/life-support-systems
https://www.nasa.gov/centers/marshall/news/releases/2015/space-station-teaches-nasa-valuable-lessons-about-life-support-systems.html
https://images.nasa.gov/details-Expedition_56_Education_Interview_with_McAuliffe_Shepard_Discovery_Center_2018_0809_688241.html
Information about the electric system onboard the ISS, including the available outlet power: https://ntrs.nasa.gov/archive/nasa/casi.ntrs.nasa.gov/20000063506.pdf
Angad Mehrotra — Power Systems Engineer @ NASA-JSC + Expedition 56 Flight Engineer (Astronaut) Ricky Arnold

Power is generated entirely by the solar arrays which gets fed to station systems through a number of power channels with excess power being stored in large batteries kept on the outside of the station. Life support includes the water and oxygen resources listed above, along with devices to remove carbon dioxide, ventilation systems to keep air circulating (which doesn’t happen automatically in microgravity, and thermal control systems (heat/cooling).
Daniel Huot — Public Affairs Officer – International Space Station @ NASA JSC

What power and life support systems are being planned for future spacecraft and human habitats on other planets?

For future missions, if solar power is not an option, we can also utilize Radioactive Thermal Reactors for a long term exploration mission. Most missions have large batteries capable of being charged up and storing energy as well.
See links on Life Support Systems:
https://www.nasa.gov/content/life-support-systems
https://www.nasa.gov/centers/marshall/news/releases/2015/space-station-teaches-nasa-valuable-lessons-about-life-support-systems.html
Angad Mehrotra — Power Systems Engineer @ JSC

Since Orion is under development, this talks to the new technologies that have been designed and integrated into the new exploration missions flights (EM-1, EM-2 and beyond). While each mission will give us more data and ideas of new technologies, especially since technology will advance significantly between each mission, we don’t have any notions of what newer versions may be.
Spacecraft traveling to deep space cannot easily be resupplied and must be able to support life for long periods. Orion will support life in deep space because it has:
• An advanced carbon dioxide removal system requiring no chemical storage consumables
• Can store high quantities of consumables like water, food and oxygen
• High efficiency solar electric power generation and robust battery storage capability
• Smart, autonomous systems that can operate far from Earth
• Radiation-hardened electronics
• A structure and mass optimized for deep space travel
• Seats that help astronauts endure high G loads during lunar or planetary velocity returns
• A heat shield that can withstand lunar and planetary velocity returns of 25,000 mph (30 times the speed of sound) and temperatures of 5,000 degrees F as compared to 17,500 mph and 3,000 degrees from lower Earth orbit (LEO) — temperatures that are 70% higher than from a LEO return
Orion is equipped with a launch abort system to keep crew safe during ascent, and systems to sustain life in deep space, such as radiation protection; a heat shield for lunar and planetary velocity returns; crew accommodations such as a bathroom, exercise equipment and galley; propellant storage; water supply; and deep space guidance, long-range navigation and control that function where GPS cannot.
Systems to Live and Breathe: As humans travel farther from Earth for longer missions, the systems that keep them alive must be highly reliable while taking up minimal mass and volume. Orion will be equipped with advanced environmental control and life support systems designed for the demands of a deep space mission. Highly reliable systems are critically important when distant crew will not have the benefit of frequent resupply shipments to bring spare parts from Earth, like those to the space station.
Proper Propulsion: The farther into space a vehicle ventures, the more capable its propulsion systems need to be to maintain its course on the journey with precision and ensure its crew can get home.
Orion has a highly capable service module that serves as the powerhouse for the spacecraft, providing propulsion capabilities that enable Orion to go around the Moon and back on its exploration missions. The service module has 33 engines of various sizes. The main engine will provide major in-space maneuvering capabilities throughout the mission, including inserting Orion into lunar orbit and also firing powerfully enough to get out of the Moon’s orbit to return to Earth. The other 32 engines are used to steer and control Orion on orbit.
The Ability to Hold Off the Heat: Going to the Moon is no easy task, and it’s only half the journey. The farther a spacecraft travels in space, the more heat it will generate as it returns to Earth. Getting back safely requires technologies that can help a spacecraft endure speeds 30 times the speed of sound and heat twice as hot as molten lava or half as hot as the sun.
When Orion returns from the Moon, it will be traveling nearly 25,000 mph, a speed that could cover the distance from Los Angeles to New York City in six minutes. Its advanced heat shield, made with a material called AVCOAT, is designed to wear away as it heats up. Orion’s heat shield is the largest of its kind ever built and will help the spacecraft withstand temperatures around 5,000 degrees Fahrenheit during reentry though Earth’s atmosphere.
Radiation Protection: As a spacecraft travels on missions beyond the protection of Earth’s magnetic field, it will be exposed to a harsher radiation environment than in low-Earth orbit with greater amounts of radiation from charged particles and solar storms that can cause disruptions to critical computers, avionics and other equipment. Humans exposed to large amounts of radiation can experience both acute and chronic health problems ranging from near-term radiation sickness to the potential of developing cancer in the long-term. Orion was designed from the start with built in system-level features to ensure reliability of essential elements of the spacecraft during potential radiation events. Constant Communication and Navigation: Spacecraft venturing far from home go beyond the Global Positioning System (GPS) in space and above communication satellites in Earth orbit. To talk with mission control in Houston, Orion will use all three of NASA’s space communications networks.
Orion is also equipped with backup communication and navigation systems to help the spacecraft stay in contact with the ground and orient itself if its primary systems fail. The backup navigation system, a relatively new technology called optical navigation, uses a camera to take pictures of the Earth, Moon and stars and autonomously triangulate Orion’s position from the photos.
https://www.nasa.gov/exploration/systems/orion/index.html
https://www.lockheedmartin.com/en-us/products/orion.html
Mark Kirasich — Orion Program Manager @ NASA-JSC,
Laura Rochon — Public Affairs Officer – Orion Program @ NASA-JSC,
Brittney Thorpe — Public Affairs Officer – Orion Program & Exploration Ground Systems @ NASA-KSC

If an astronaut could add enhancements to the current EVA spacesuit, what would they want or need?

One issue with the EVA suits is the force required to grip things through the gloves with the pressure inside the suit constantly working against you. Visibility and range of motion are other issues that could be addressed in future suit designs.
https://www.nasa.gov/audience/foreducators/spacesuits/home/clickable_suit_nf.html
Info on spacesuits:
https://www.nasa.gov/audience/forstudents/5-8/features/nasa-knows/what-is-a-spacesuit-58.html
https://spaceflight.nasa.gov/station/eva/spacesuit.html
https://spaceflight.nasa.gov/shuttle/reference/faq/eva.html
https://en.wikipedia.org/wiki/Liquid_cooling_and_ventilation_garment
Nathan Fraser-Chanpong — NASA Electrical Engineer

Regarding radiation shielding: We are proposing passive shielding that consists of an outer layer of heavy metals and an inner lining of first polyethylene and then the inner-most layer being a lining of liquid hydrogen. You could then tap into the hydrogen in space for the return journey from Mars since liquid hydrogen is used as the fuel. Is this feasible? If so, might it be a potential solution?

The problem with GCRs is that whenever they impact another atom, they split it creating more radiation particles also known as secondary radiation. While these impacts slowly decrease in energy, they do multiply the number of dangerous particles. Putting a dense material on the outside will create lots of interaction between particles and can actually make the problem worse by creating lots of secondary radiation that can cause more problems than the initial GCR. The use of hydrogen is interesting because you can’t split hydrogen, so the energy still decreases, but you don’t produce the dangerous secondary radiation. Also, you don’t produce neutrons, which can get through any magnetic field that might be set up to protect against charged particles. However, the challenge is that hydrogen in its liquid and most dense form wants to be at -420 degrees Fahrenheit, which is a bit cold for astronauts to be living in. Overall, the use of hydrogen as a radiation barrier is a good idea.
The idea of using the hydrogen as a propellant is also interesting. Most in-space propulsion events are rather short (say a few minutes) and occur when entering or leaving an orbit. So the main propulsion events during return will be leaving Mars orbit and entering Earth orbit. If you use some of the hydrogen leaving Mars, then you have less protection for the return trip. However, you could use it for Earth orbit entry. Depending on how much risk you take on, the radiation exposure determines if you could use all the hydrogen at that propulsion event.
I’ve put a few links below for some papers/presentations we have put together previously on how you might store hydrogen in various polymers to minimize secondary radiation.
https://doi.org/10.1063/1.4860619
https://www.google.com/url?sa=t&rct=j&q=&esrc=s&source=web&cd=5&ved=2ahUKEwjop4Kb6uzdAhWL61MKHXcYA5QQFjAEegQIBRAC&url=http%3A%2F%2Findico.cern.ch%2Fevent%2F255141%2Fcontribution%2F1583525%2Fattachments%2F446793%2F619603%2F2MOrD1-03_Bulge_Testing_ICMC.pdf&usg=AOvVaw2lnuUcrWu7VQdo9q3sHDc3
Mr. Wesley Johnson — Cryogenics Team Lead @ NASA GRC

Would a greater oxygen availability (i.e. plants) prevent astronauts from feeling fatigued (or help with any other challenges)?

Oxygen levels are a very complicated problem. It is unlikely that we would increase oxygen levels in order to offset muscle fatigue concerns, because higher oxygen content leads to increased risks of fire. After the Apollo 1 accident we are very careful about oxygen levels. You can actually find requirements about these kinds of things from a human health and performance perspective here:
https://www.nasa.gov/feature/human-integration-design
https://www.nasa.gov/hhp/standards
Dr. Sherry Thaxton — Human Health & Performance division @ JSC

Regarding lack of sunlight in space and how it can affect the astronauts with their mood and mentality: What kind of lights are used in the ISS? Do they use the lights that can be used for seasonal light disorder? Do the lights have a slow brightness to them and slow dimming to help recreate the effects of sunrise and sunset?

There is an ongoing Lighting Effects experiment on the ISS and details can be found here:
https://www.nasa.gov/mission_pages/station/research/experiments/explorer/Investigation.html?#id=2013
In the event the link does not work, the experiment title is “Testing Solid State Lighting Countermeasures to Improve Circadian Adaptation, Sleep, and Performance During High Fidelity Analog and Flight Studies for the International Space Station”
https://www.acsh.org/news/2018/06/07/astronauts-seeking-better-sleep-change-lights-mimic-sun-13057
Jeffrey Sugar — EVA Hardware Manager @ JSC

Are odors and smells as issue in spacecrafts?

As far as odors and smells are concerned in spacecraft, here are some resources: http://theweek.com/speedreads/450621/nasa-professional-smeller
https://spaceflight.nasa.gov/shuttle/support/people/galdrich.html
https://www.reddit.com/r/space/comments/970lzz/i_am_the_chief_sniffer_and_volunteer_nasalnaut/
https://gizmodo.com/how-nasa-deals-with-odor-inside-the-international-space-1648864449
https://www.nasa.gov/mission_pages/station/research/long_duration_sorbent_testbed

How is the outside of the ISS currently maintained? For the future, what about a robot that will be controlled from inside the ISS to make any repairs and perform maintanence? We could use bulk metallic glass as the main building material, and augmented reality technology to help better the process of remotely fixing any problems.

I organized a panel on advanced robotics a few years ago, and one of my panelists was the fellow from the Canadian Space Agency who designed Dextre, the robot that performs maintenance on the exterior of the ISS. Launched in May 2008, it’s a two-armed dexterous manipulator with a zero fault tolerant design that is operated exclusively from the ground. That’s about as far as NASA has ventured, to my knowledge. After all, the ISS operating environment makes it difficult to operate such a robot:
• Harsh environmental conditions (thermal, materials)
• Limited time & access for inspection & maintenance
• Limited spares stowage; Getting parts to/from orbit is costly and infrequent
• Human spaceflight
– Being crew-tended increases flexibility & capability
– …but also increases safety & fault tolerance requirements
• Training burden of a complex vehicle with a small crew
– “Skills-based” maintenance training
– Even for those tasks that are specifically trained, it may have been a long time since the crew member saw it.
Re-supply Infrastructure is particularly challenging. And even with your plans for augmented reality, it would likely require the ISS crew to train the robot—with limited spare time for such tasks. Also, in designing the perfect space robot, you would have to make sure that it was constrained from doing anything that could harm the ISS or the crew:
• Robust hardware design
– Robust thermal & materials design
– Anticipate less robust components: cameras & lights
– Lubrication design to withstand vacuum
– Multiple strings / channels / command paths
– Allow the option to “limp along” (degraded operations) indefinitely
• Flexible software design
– Reconfigurable software (quick parameter updates)
– Auto-responses can be inhibited or changed easily
– Even safety constraints can be overridden by the operators
• Operational flexibility
– Can be operated/commanded from any location
– Expect unexpected tasks
Your use of metallic glass is interesting. I have some samples in a display case where we here at JPL are starting to use the material for stuff like springs and bearings, since it has some neat properties. And it is certainly robust (see above). But I don’t think the technology is sufficiently advanced at present to construct a robot out of it. We tend to build spacecraft hardware out of aluminum—lightweight, can we welded, many alloys with differing properties to pick from. Some titanium for strength, though it is harder to work with. Sometimes composites—where outgassing is not a problem. Basically, JPL is loath to fly technology where we don’t know how it will behave in space.
The augmented reality is a good idea. NASA has used it to plan maintenance tasks on the ISS, and JPL surface property scientists are using OnSight as a visualization tool during operations to assess the safety of the next sol’s proposed Mars rover drive, as well as to conduct virtual meetings on Mars.
It’s hard to estimate the cost. Dextre cost over $200 million: it’s expensive when you are making just a single copy.
David Oberhettinger — Chief Knowledge Officer (CKO) at NASA/CalTech Jet Propulsion Laboratory (JPL)

What would be the size of a Mars-bound spacecraft? What would be the size of an astronaut’s living quarters?

This study suggests a Mars-bound spacecraft would be about 41 metric tons, and the individual living quarters would be at least 25 cubic meters per person.
https://ntrs.nasa.gov/archive/nasa/casi.ntrs.nasa.gov/20170002219.pdf
Nathan Fraser-Chanpong — Electrical Engineer @ JSC

What types of personal items will the Mars-bound astronauts be able to bring? Is there a maximum size/weight of the items?

Planning for human Mars mission is not yet complete. Space Shuttle and Soyuz launches might serve as an indicator of what might be allowed during future, deep space missions. For Space Shuttle, each crew member was allowed a Personal Preference Kit (PPK) that was up to 20 items, fits in a 5″ x 8″ x 2″ bag, weighing up to 0.68 kg (1.5 lbs). On Soyuz launches, crew are allowed up to 1 kg (2.2 lbs).
https://www.forbes.com/sites/quora/2018/06/26/how-many-personal-items-can-astronauts-bring-to-space/#4d922fc3a30e
Bill Bluethmann, Ph.D. — Robotics Engineer @ JSC

What will be the most effective way to produce power for electronic devices inside the Mars-bound spacecraft?

In general on current human spacecraft, power has been generated using solar panels. These panels collect the energy from the sun and convert it into electrical power (DC power) to be stored in batteries on the spaceship. Solar panels work better in space than they do on Earth because they don’t have to deal with the interference of that atmosphere. However, as you go further away from the sun, the power generation will decrease some. Another consideration is when you land on Mars, the martian atmosphere and dust will also decrease power generation by solar panels, so they will need to be kept clean. A second option that is used by un-maned spacecraft is nuclear power, specifically radioisotope thermoelectric generators. These do not need the sun to produce power. BUT, these produce large amounts of radiation that could be detrimental to human health and would need to be isolated from the human crew.
Logan Farrell — Robotics Engineer @ JSC

What type of data storage has proven to work best for electronics inside spacecraft?

Often, solid state data recorders are used in deep space. These are relatively low performing compared to earth-based data storage. However, they are able to operate in the extreme radiation, thermal, and vacuum environment of deep space.
On the International Space Station, Astronauts and Cosmonauts use Laptops to connecting into the ISS main computers and to use for email and web surfing.
Some good resources for space processors and data storage can be found at the following links.
https://www.hpe.com/us/en/insights/articles/the-space-station-gets-a-new-supercomputer-1708.html
https://www.swri.org/solid-state-recorders
https://mars.nasa.gov/mro/mission/spacecraft/parts/command/
http://www.cpushack.com/space-craft-cpu.html
https://en.wikipedia.org/wiki/RAD750
Bill Bluethmann, Ph.D. — Robotics Engineer @ JSC

What is the most effective way to keep loose objects static in space, so they do not float around the spacecraft?

Velcro is very effective at holding loose items in place in space. When doing an assembly or disassemly task involving small parts one strategy astronauts use is a long piece of tape, fastened down on both ends with the sticky side out, and then they press the screws, bolts, etc. onto the tape. That way the parts are still next to the astronaut, not getting lost around the space station, when they need them again a few minutes later.

What type of fabric are the astronaut sleeping bags made of? Has the temperature been measured inside a sleeping bag while an astronaut is sleeping in it? If so, what were the findings?

The sleeping bags currently being used on The International Space Station are made of lightweight Beta cloth, also referred to as Beta fabric. Beta cloth is a Polytetrafluoroethylene (PTFE) impregnated cloth that is easily cleaned and maintained. You probably know PTFE by its more common name, Teflon. When designing the International Space Station, NASA did many studies evaluating Beta cloth; one fine study is entitled “Beta Cloth Durability Assessment for Space Station Freedom (SSF) Multi-Layer Insulation (MLI) Blanket Covers” that can be found at https://ntrs.nasa.gov/archive/nasa/casi.ntrs.nasa.gov/19930015285.pdf . Marshall Space Flight Center in Huntsville, Alabama has been studying fabrics for future spaceflights and some of their results can be found at https://ntrs.nasa.gov/archive/nasa/casi.ntrs.nasa.gov/20170006996.pdf . You can see the Beta cloth in this terrific overview of sleeping in space by NASA eClips at https://youtu.be/brMKdXJN_fE . Although there is not a lot of data available for the temperature inside the sleeping bag, there has been a great deal of analysis done on the temperature of the crew quarters where the astronauts sleep. For example, James (Broyan), Scott (Cady), and David (Welsh), and their colleagues did a study on the “International Space Station Crew Quarters Ventilation and Acoustic Design Implementation” that can be found at https://ntrs.nasa.gov/archive/nasa/casi.ntrs.nasa.gov/20100017014.pdf . Over a decade ago, James (Broyan), Melissa (Borrego), Juergen (Bahr) and their colleagues described the development of the crew quarters at https://ntrs.nasa.gov/archive/nasa/casi.ntrs.nasa.gov/20080013462.pdf . As you’ll find, James has years and years of experience with crew quarters. Again, although not specific to sleeping bag temperature, Thilini (Schlesinger), Branelle (Rodriguez), Melissa (Borrego), and their colleagues put together a terrific description about the performance of the crew quarters on The International Space Station that can be found at https://ntrs.nasa.gov/archive/nasa/casi.ntrs.nasa.gov/20130011142.pdf . THANK YOU for doing space exploration research. -Lucien
Lucien Junkin — NASA Space Exploration Vehicle (SEV) Chief Engineer

Is it safe to use magnets in the ISS?

Yes, it is. See this video of how magnets work on the ISS: https://www.youtube.com/watch?v=NvJcrGlzGAg
One thing to note: most surfaces on spacecraft, inside or outside, are unlikely to be magnetic. Spacecraft are primarily made of strong, lightweight materials such as aluminum and carbon fiber.
Nelson Brown — NASA Armstrong Flight Research Center

What systems are used to regulate temperature on the ISS?

See the following sources:
https://science.nasa.gov/science-news/science-at-nasa/2001/ast21mar_1 https://www.nasa.gov/content/cooling-system-keeps-space-station-safe-productive

How are plants grown on-board the ISS? Where do all the resources come from that are necessary to grow plants? Based on what we know of various locations in space, how do we plan to grow plants in various places we explore?

See the following sources about growing plants in space:
http://www.ustream.tv/recorded/117798919
https://www.nasa.gov/mission_pages/station/research/news/flowers
https://www.nasa.gov/mission_pages/station/research/experiments/1294.html
https://www.nasa.gov/mission_pages/station/research/experiments/2621.html
https://www.nasa.gov/feature/can-plants-grow-with-mars-soil
https://www.skyandtelescope.com/astronomy-news/some-plants-grow-well-in-martian-soil/
https://www.acs.org/content/acs/en/education/resources/highschool/chemmatters/past-issues/2016-2017/april-2017/growing-green-on-the-red-planet.html
https://www.popsci.com/nasa-growing-food-in-space#page-5
https://eos.org/articles/tests-indicate-which-edible-plants-could-thrive-on-mars
https://www.nasa.gov/mission_pages/station/research/news/meals_ready_to_eat
https://www.nasa.gov/mission_pages/station/research/Giving_Roots_and_Shoots_Their_Space_APH
https://www.nasa.gov/feature/how-does-your-space-garden-grow
https://www.nasa.gov/mission_pages/station/research/news/sprouts_in_space.html
https://blog.waikato.ac.nz/physicsstop/2015/10/05/plants-in-circular-motion/
https://www.reddit.com/r/askscience/comments/12nush/how_would_a_plant_react_to_being_grown_while/
https://www.nasa.gov/vision/earth/technologies/aeroponic_plants.html
https://www.maximumyield.com/fogponics-a-new-spin-on-aeroponic-gardens/2/1401
About plant pillows:
http://gravitationalandspaceresearch.org/index.php/journal/article/download/749/777
http://cssf.usc.edu/History/2009/Projects/J2019.pdf
http://cssf.usc.edu/History/2009/Projects/J2002.pdf

How is Oxygen created in spacecraft?

The most common means of generating oxygen in a spacecraft is the electrolysis of water. This YouTube video shows a simple water electrolysis experiment that those of you who have had chemistry, your teacher may have performed as part of a class demonstration: https://www.youtube.com/watch?v=T-OwWOYHhMI
Presently, water electrolysis is used to make oxygen for the astronauts (and cosmonauts) on the International Space Station (ISS). The actual electrolysis device is a bit more complicated than this simple experiment. For one thing, it doesn’t require salt to be added to the water. A wet polymer membrane provides the conductivity to “complete” the circuit along with providing a barrier to collect the oxygen separate from the also generated hydrogen. The generated hydrogen is vented overboard, or can be used to recover additional oxygen from carbon dioxide. The ISS has this additional technology; it is called a Sabatier reactor and converts captured carbon dioxide (from the cabin atmosphere) and feeds it to the Sabatier reactor with some of the generated hydrogen. The chemical reaction is: CO2 + 4H2 –> CH4 (methane) + H2O. The methane is vented overboard, and the water is fed to the water electrolyzer. This process can recover close to 50% of the oxygen respired by the crew, and cuts down on the amount of water that has to be carried up to the ISS via resupply vehicles. Another fun fact: Water electrolysis is also the technology used to generate oxygen on Navy submarines.
Dr. Bob Green — Chemical engineer @ GRC

How is water heated on the ISS, and how much can be heated at once?

The astronauts and cosmonauts on the International Space Station heat water by using the Potable Water Dispenser, or PWD. The PWD can supply up to approximately two liters of hot water every 30 minutes. Laura (Shaw), Jose (Barreda), and their colleagues created a great reference for the PWD that can be found at https://ntrs.nasa.gov/archive/nasa/casi.ntrs.nasa.gov/20080013432.pdf . Also, since their reference, Katherine (Toon), Randal Lovell, and their colleagues did a study concerning the performance of the PWD on ISS entitled, “International Space Station USOS Potable Water Dispenser On-Orbit Functionality vs. Design” that can be found at https://ntrs.nasa.gov/archive/nasa/casi.ntrs.nasa.gov/20100014345.pdf . Recently, Brandon (Maryatt) and his colleagues documented the lessons learned from the ISS’s PWD entitled “Lessons Learned for the International Space Station Potable Water Dispenser” that can be found at https://ttu-ir.tdl.org/ttu-ir/bitstream/handle/2346/74108/ICES_2018_114.pdf. THANK YOU for being part of space exploration. -Lucien
Lucien Junkin — NASA Space Exploration Vehicle (SEV) Chief Engineer

Regarding nail trimming in space: In an effort to not have loose nail clippings roaming around a spacecraft, would a 3-D printed attachment to go on the end of a vacuum cleaner, with a hole for fingers and toes be advantageous and/or desirable?

Using a vacuum to collect loose particles is very effective. The main problem is having enough hands to control the work (nail clippers, vacuum nozzle, and fingernails). Therefore, an nozzle adapter that you could mount (ie velcro) then put your fingertip on and trim with your remaining hand would be awesome!
Jeff Stone — Former Flight Controller/Trainer @ JSC

Is human waste (poop) recycled in any way on ISS? Could it be turned in to methane gas, and then used as fuel for heat or cooking?

There is no solid human waste currently being processed on the ISS. It is deposited in bags, which are in turn deposited in a sealed tank. The tanks are then loaded into returning freighter ships that burn up in the atmosphere.
I am sure it is possible to recycle it in some way, but the energy derived would have to be more than the energy used or it would not be worth it.
Here is information about a proposed system to recycle solid human waste: https://ntrs.nasa.gov/search.jsp?R=20160003882
Jeff Stone — Former Flight Controller/Trainer @ JSC

Is there currently (or has there ever been) any composting on ISS?

ISS has not done any composting experiments that I am aware of. It would likely take a fair amount of crew time to execute, and crew time is the rarest resource on board!
Jeff Stone — Former Flight Controller/Trainer @ JSC

What kinds of batteries do we use on the ISS?

Li-ion primarily, few other batteries, but Lithium-ion is the most common type of battery used on the space station.
Angad Mehrotra — Power Systems Engineer @ JSC

Regarding space debris damage: Would it be helpful to have a device (robot) that goes over the exterior railings of the ISS, locating and filling in all the craters formed by micrometeriods, so the astronauts do not cut their gloves on spacewalks?

That’s a really great and imaginative idea! I think that would be a wonderful addition to the Space Station. An autonomous robot that crawls outside the Space Station repairing every tiny little dent or anything else it finds wrong. Keep thinking outside the box! Or outside the Space Station, in this case!
Kurt Leucht — Software engineer for Swamp Works @ NASA’s Kennedy Space Center

Regarding the medical kits sent to and used on ISS, how are the medicines in these kits packaged, and what are the materials used in the packaging (what are the packages made of)?

The individual medicines and medical aids are packaged in plastic bags. These plastic bags are then put in a case made from Beta cloth, or Beta fabric. Beta cloth is a Polytetrafluoroethylene (PTFE) impregnated cloth that is easily cleaned and maintained. You probably know PTFE by its more common name, Teflon. When designing the International Space Station, NASA did many studies evaluating Beta cloth; one fine study is entitled “Beta Cloth Durability Assessment for Space Station Freedom (SSF) Multi-Layer Insulation (MLI) Blanket Covers” that can be found at https://ntrs.nasa.gov/archive/nasa/casi.ntrs.nasa.gov/19930015285.pdf . Marshall Space Flight Center in Huntsville, Alabama has been studying fabrics for future spaceflights and some of their results can be found at https://ntrs.nasa.gov/archive/nasa/casi.ntrs.nasa.gov/20170006996.pdf . Scientists, doctors, and nurses at the Johnson Space Center presented the lessons learned of the ISS Medical Kit, along with lots of pictures of the ISS Medical Kit, at https://ntrs.nasa.gov/archive/nasa/casi.ntrs.nasa.gov/20110010924.pdf . Further, Dr. Joe Dervay, a NASA flight surgeon, describes medical kits at https://youtu.be/cOkJUVEriVw . THANK YOU for being part of space exploration. -Lucien
Lucien Junkin — NASA Space Exploration Vehicle (SEV) Chief Engineer

Has anyone ever used magnetic forces to simulate gravity in the ISS? If not, has this been considered? Would this be a possibility for astronauts when exercising?

Interesting idea. The challenge with using magnets on ISS is that humans aren’t affected by magnets in a manner akin to gravity. Checkout a relatively old article about more subtle effects of (very large) magnets on the human body.
https://www.nytimes.com/1983/02/22/science/do-enormous-magnets-affect-the-human-body.html
Bill Bluethmann, Ph.D. — Robotics Engineer @ JSC

Because magnetism is an exponential force with distance, it can be challenging to simulate gravity across your entire body. If you wanted to apply magnetic force to exercise, you need to figure out how to convert the magnetic force to be constant over a large motion of whatever exercise you are attempting. You can look at exercise bikes and how they use magnetic resistance. Research the exercise equipment used on the international space station to learn how each piece of equipment operates. https://www.nasa.gov/audience/foreducators/stem-on-station/ditl_exercising
Mr. Asher Leiberman — Robotics Supervisor @ JSC

What stage are we up to in the development of Artificial Intelligence robots? Are there any problems with using AI robots instead of human astronauts? What do AI robots need to do to become equivalent to a human astronaut?

There are a couple projects where smart robots are flying as testbeds on the International Space Station. In 2011, Robonaut flew to the International Space Station aboard the Space Shuttle Discovery as part of the STS-133 mission. Robonaut is built in a humanoid form for the purpose working in a human environment, using human tools and interfaces. In addition to the humanoid form, Robonaut, SPHERES and Astrobee serve the role as free flying inspection robots. SPHERES is a free flyer research platform and Astrobee will serve as a facility once commissioned in 2018-2019.
Initially, these advanced robots will help performing routine task such as inspection and maintenance. As capabilities grow, tasks such as emergency management, logistics and advanced maintenance/inspection. A general challenge to AI robots in space is a lack of computing power in space-qualified computers. Due to radiation effects, computing in space tends to lag terrestrial computing. There is an on-going joint effort between NASA and the US Air Force to modernize space computing (called High Performance Space Computing).
Find out more about Robonaut, SPHERES, Astrobee and HPSC at the links below.
https://robonaut.jsc.nasa.gov/R2
https://www.nasa.gov/astrobee
https://www.nasa.gov/spheres/home
https://gameon.nasa.gov/projects/high-performance-spaceflight-computing-hpsc/
R2 problems on ISS after attaching legs: https://blog.newrelic.com/engineering/state-of-artificial-intelligence/
CIMON: https://www.ibm.com/thought-leadership/smart/de-de/ai-in-space/
CIMON: https://www.youtube.com/watch?v=KnpJI3WeiBg
CIMON: https://www.youtube.com/watch?v=XkgNBNw8VEw
Bill Bluethmann, Ph.D. — Robotics Engineer @ JSC

As a partial answer, the ability of a robot to autonomously perform tasks requires it to understand or at least be able to learn its environment – where things are located, what changes when things are moved around by crew, how it fits within the workspace. Tasks should be simple and the same each time they need to be performed. Ideally, the interfaces that the robot interacts with are all the same or at least limited in their variety to avoid having to interface with a lot of tools and to simplify what the robot has to learn.
The program behind the AI can be limiting as well. The best example of this is one given to us when I was in college (~30yrs ago) in which a genetic algorithm was being developed to teach a robot to identify images of a forest with military tanks in it. After a training period, the algorithm was able to correctly identify which images had tanks and which did not 100% of the time. Then another set of images was provided (still forest with tanks or without) and the algorithm was getting them all wrong. It took a bit for the programmers to figure out what was going on. They finally realized that in the original set of images, the tanks had all been in the forest on a sunny day while the pictures without tanks were on a cloudy day. The algorithm had learned to detect sun/cloud and not the presence of tanks. We need to enable robots to learn, but we also need to be wary of them learning something unexpected!
Dr. Monica Visinsky — Senior Engineer, Oceaneering Space Systems @ JSC

Are there opportunities to modify the color scheme & ambiance in some respective areas of the ISS to make it more aesthetically pleasing, and to emulate certain activities? For instance, in the exercise/treadmill area, could more yellow/orange/green be used to simulate the outdoors/daylight? In the sleeping quarters, could black/dark blues be used to mimic nighttime?

The new LED light assemblies allow the crew and the ground to modify the color temperature of the lights in the various modules. While this is not my area of expertise, I know that much research has been done on optimal lighting for various activities for maintaining crew health and efficiency. The new lights have been on board for several years and I would bet there are studies out on their effectiveness.
https://www.acsh.org/news/2018/06/07/astronauts-seeking-better-sleep-change-lights-mimic-sun-13057
Jeff Stone — Former Flight Controller/Trainer @ JSC

Was UMPQUA’s washing machine completely developed, and if so, did it make it into space? Are there any washing machines in space? We have read clothes don’t “feel clean” after a couple days… Is it the water that helps them feel clean, or would a dryer sheet plus a cleaning without water (i.e. with IR light) help?

Here are some helpful resources on the concept of washing machines in space: https://sbir.gsfc.nasa.gov/SBIR/abstracts/11/sbir/phase2/SBIR-11-2-X3.02-9012.html
Other studies UMPQUA has worked on: https://sbir.gsfc.nasa.gov/sbir_search?page=1&searchText=UMPQUA&searchType=award
Another abstract https://onlinelibrary.wiley.com/doi/abs/10.1021/bp0100732
IVA Clothing Study (Updated 2017): https://www.nasa.gov/mission_pages/station/research/experiments/1084.html
Spinoff relating to alternative laundry technologies: https://spinoff.nasa.gov/spinoff2001/er9.html
Some final advice – Having a way to keep clothes clean in space (especially without water) is of great interest, so keep working at your concepts!
Kaitlin Lostroscio — Robotics: Digital Astronaut Simulation & Exercise Countermeasures Support (JSC)

See video of Astronaut Doug Wheelock talking about clothes used on ISS, lack of a washing machine, & what is done with worn clothes: https://www.youtube.com/watch?v=lYnOs3lGL88
Skip to 9:20 in this video about not washing clothes in space: https://images-assets.nasa.gov/video/Expedition_56_Education_Interview_with_Poudre_High_School_2018_0912_701862/Expedition_56_Education_Interview_with_Poudre_High_School_2018_0912_701862~small.mp4
https://www.newsweek.com/2015/07/17/introducing-astronaut-clothing-future-349955.html
https://www.smithsonianmag.com/smart-news/there-no-laundry-space-nasa-works-reducing-astronaut-stench-2-180952031/
https://www.nasa.gov/vision/space/livinginspace/Astronaut_Laundry.html
https://ntrs.nasa.gov/archive/nasa/casi.ntrs.nasa.gov/20150003039.pdf
https://spaceflight.nasa.gov/living/spacewear/index.html

Has UV light been used to clean clothing in space? Could UV light cause fires in space? What about Infrared light?

NASA doesn’t currently clean clothes in space BUT we are working on technology to clean clothes in space. NASA understands that cleaning clothes in space will be very important for long duration missions. Michael (Ewert) and Frank (Jeng), along with their colleagues at the NASA Johnson Space Center, did a great study on cleaning clothes in space entitled “Will Astronauts Wash Clothes on the Way to Mars?” that can be found at https://ntrs.nasa.gov/archive/nasa/casi.ntrs.nasa.gov/20150003039.pdf . The study also discusses using UV light to clean clothes, “Ozone has been used for sanitation of clothing, sports gear and other items. Ultraviolet lamps are used to generate ozone. During testing at Johnson Space Center (JSC), clothes were sanitized in bags using ozone with concentration of 2 to 4 ppm for 7 to 8 hours. In ZONO Sanitech applications, clothing or sports gear are placed in a cabinet with ozone at 7 to 8 ppm for 30 minutes. Catalysts are used to eliminate residual ozone when the sanitizing is finished. Concerns of using ozone include its toxicity and the stringent allowable concentration in a closed spacecraft cabin as well as potential material degradation. The maximum ozone concentration in the Occupational Safety & Health Administration (OSHA) Permissible Exposure Limit (PEL) is 0.1 ppm 8-hour time-weighted average.” Concerning fires in spacecraft, a general guideline you can use is, “What can cause a fire in your home, can cause a fire in a spacecraft.” NASA is VERY concerned about fires in spacecraft and we are very, very safety conscious about fires; never will NASA forget the lessons learned from Apollo I where Gus Grissom, Ed White, and Roger Chaffee had their lives taken in a spacecraft fire. THANK YOU for being part of space exploration. -Lucien
Lucien Junkin — NASA Space Exploration Vehicle (SEV) Chief Engineer

Regarding radiation: How much shielding is good enough for adequate radiation protection for a 2-year visit to Mars (with 6-month trips to and from & a 1 year stay)?

Radiation exposure is described in terms of dose. The dose that an astronaut receives depends on how much time they spend in certain shielding environments, as you indicate in your question. The NASA limit for radiation exposure in low-Earth orbit is 50 rem/year, with career exposure limits based on gender and age: https://www.nasa.gov/sites/default/files/atoms/files/space_radiation_ebook.pdf
From the above link “Radiation exposure for astronauts aboard the ISS in Earth orbit is typically equivalent to an annualized rate of 20 to 40 rems (200 – 400 mSv).20 The average dose-equivalent rate observed on a previous Space Shuttle mission was 3.9 μSv/hour, with the highest rate at 96 μSv/hour, which appeared to have occurred while the Shuttle was in the South Atlantic Anomaly region of Earth’s magnetic field (1 Sv = 1,000 mSv = 1,000,000 μSv).21 For a six-month journey to Mars an astronaut would be exposed to roughly 300 mSv, or a total of 600 mSv for the round-trip. If we assume that the crew would spend 18 months on the surface while they wait for the planets to realign to make the journey back to Earth possible, they will be exposed to an additional 400 mSv, for a grand total exposure of about 1,000 mSv. Note that an astronaut repeating the same journey on multiple occasions could receive less or more radiation each time, if they are in the line of a CME or SPE.”
Nathan Fraser-Chanpong — Electrical Engineer @ JSC

See links on protecting astronauts from dangerous radiation in deep space travel:
https://www.nasa.gov/feature/goddard/real-martians-how-to-protect-astronauts-from-space-radiation-on-mars
https://www.nasa.gov/directorates/spacetech/strg/nstrf2016/Passive_Radiation_Shielding
https://www.space.com/41887-mars-radiation-too-much-for-astronauts.html
You can learn more about the hazards of radiation and spaceflight at:
https://www.nasa.gov/hrp/hazards
https://www.nasa.gov/johnson/HWHAP/hazard-1-radiation
https://www.youtube.com/watch?v=3jRxgvwhGC0&feature=youtu.be

Are contaminants on spacesuits after EVAs an issue? If so, how are they removed from the spacesuit and the ISS?

On the ISS, contaminants resulting from natural space are not much of a concern but we do have procedures to address contaminants that result from the ISS systems. Some concerns are contamination from ammonia, oxidizer, and hydrazine. For example, the “EVA Checklist” for the ISS at https://www.nasa.gov/centers/johnson/pdf/123834main_iss_eva_sys_checklist.pdf , have a procedure resulting from “Crew Visually Detects Plume, Leaking QD, White Crystals, or “Snow” coming from Orbiter/Station” that has the astronauts “Perform detailed visual inspection of EMU.” If “Contamination detected on EMU or seen contacting EMU?” then astronaut is to “Brush off crystals using hydrazine brush”. You can easily find all the decontamination procedures by searching “decontamination” in the referenced checklist. Also, at the end of a spacewalk, the astronauts follow the checklist to do the following:
Wipe with drying towel: LTA, legs, boots; HUT, suit arms; Gloves.
Wipe LTA crotch with stericide (in EMU Servicing Kit).
Lightly wipe seals on LTA waist ring, arm wrist rings, HUT neck ring, helmet interior with lint-free wipe (in EMU Servicing Kit).
For future missions that involve operations on the surfaces of the Moon and Mars, contaminants, including Lunar and Martian dirt, will also be a concern. During Apollo Lunar EVA operations, lunar dirt contamination was a significant issue for the astronauts. For example, on Apollo 15, David Scott commented after completing EVA1, “I guess what we ought to do is not get this clean stuff dirty. … Not too sure we’d get this dirty.” The interaction between David Scott and Jim Irwin after EVA #1 is at https://www.hq.nasa.gov/alsj/a15/a15.eva1post.html . John Young and Charlie Duke had a similar experience with lunar dirt on Apollo 16 after their EVAs; their interaction along with technical debrief is at https://www.hq.nasa.gov/alsj/a16/a16.eva1post.html . All of the Lunar Surface Journals have been compiled at https://www.hq.nasa.gov/alsj/main.html . During the design of the Space Exploration Vehicle (SEV), which is the human rover being developed by NASA for the Moon and Mars, the Apollo astronauts were passionate that we find a way to keep the dirt on the outside of the vehicle after an EVA; we incorporated suitports that support rear entry spacesuits that can be learned about at https://www.nasa.gov/pdf/464826main_SEV_FactSheet_508.pdf .
Lucien Junkin — NASA Space Exploration Vehicle (SEV) Chief Engineer

Regarding artificial gravity & centripetal force: We know a rotating spacecraft could simulate artificial gravity in space, but we can’t find in-depth material that explains how it can be achieved and various factors and forces that affect it- shape/size, pros/cons, etc.

Astronauts may be able to tolerate up to 15 rpm. The Human Research program is trying to determine if treadmill exercise and centrifugal artificial gravity for 1.5 hours per day per crewmember would be sufficient as an adaptive countermeasure to microgravity. This can open up the trade space to many spacecraft configurations that could provide artificial gravity in the 2020’s.
Mr. John J. Zipay — Deputy Branch Chief, Structures Branch (JSC)

Newton’s First Law tells us that objects have inertia. Inertia is the concept that if an object is in motion in a particular direction, then neither its speed nor its direction of motion will change unless an external force acts on the object.
Suppose we have a large cylinder in space, and an astronaut stands on the inside surface of the cylinder. If the cylinder begins to rotate, then the inner surface of the cylinder constantly “pushes” the astronaut toward the center of the cylinder, rather than let the astronaut fly out in a straight line. This “push” is called centripetal force.
If we were to use a rotating space station to create artificial gravity, we’d really want to focus on the centripetal force felt by astronauts standing on the inside edge of the space station.
There are some factors that I’m uncertain of. For example, does the centripetal force need to be as strong as the force of gravity felt at the Earth’s surface (requiring either a fast rotational speed or a small radius), or would something less than that be enough to keep astronauts strong for long trips? What is the best size/shape for a rotating space station? I’ll use my best judgment to help answer these questions, and I’ll leave it to you to search for more fulfilling answers.
For the first, we know that astronauts lose bone and muscle mass when in Low Earth Orbit (LEO). Astronauts’ bones and muscles don’t need to withstand the force of gravity, so they atrophy. In LEO, astronauts feel close to 0 gravitational acceleration (and therefore near 0 gravitational force, since Force = Mass x Acceleration). On the Moon, astronauts feel about 17% the gravitational acceleration felt on the Earth’s surface (1g). On Mars, they would feel about 38%. We know astronauts have some difficulty coming back to Earth from LEO, so an artificial gravity greater than zero should help prevent bone/muscle loss. I’m not sure if the effects of a gravitational acceleration between 0 and 1g on bone/muscle loss have been studied, since it’s difficult to simulate a gravitational acceleration less than 1g while on the Earth’s surface. We could probably better study this in a rotating space station in LEO, or with a plane descending along a very specific arc!
For the second question, which is really two questions, what is the best size and shape for a rotating space station? In terms of size, we’d want the astronaut’s head to feel about the same centripetal force as their feet. Since the astronaut’s head is closer to the center of the rotation than their feet, their head will always feel less centripetal force than their feet. This difference would probably be less noticeable on a larger space station. In terms of shape, we’d probably want to think about two factors: maximizing the area of simulated gravity per unit mass (a cylinder is best for this), and maximizing the structural strength of the space station (a donut-like shape is best for this). It’s difficult to explain why these shapes are best for these purposes without a little math. If you’re interested in a more in-depth answer (with math and pictures!), feel free to contact me and I’ll do my best to do some math and find some resources for you!
Some resources:
https://en.wikipedia.org/wiki/Centripetal_force
https://physics.stackexchange.com/questions/12634/why-dont-astronauts-in-orbit-get-stuck-to-the-ceiling
https://www.quora.com/Why-doesnt-the-ISS-have-a-centrifugal-gravity-module-Is-it-because-its-too-expensive-or-because-it-just-cant-be-done-with-current-tech
Michael Bernard — Intern, Advanced Caution and Warning System Project (JSC)

What different types of lasers are used in space? Are the lasers able to push things (change their location/path)? If so, how? Would lasers be a way to slow down space debris or re-direct it towards Earth so it burns up in Earth’s atmosphere?

Slowing down space debris in space using a satellite-based or ISS-based laser would absolutely be possible. Practically, it may prove difficult to aim the laser. Cost is another potential barrier. This solution might be considered in the future to protect the ISS, other spacecraft, and Earth. However, it also poses an international political problem. See this article for more details: https://phys.org/news/2018-01-china-space-junk-lasers.html
Here are more resources about this:
https://en.wikipedia.org/wiki/Laser_broom
https://www.livescience.com/61453-china-satellite-lasers-clean-space-debris.html
https://www.sciencedirect.com/science/article/pii/S1000936114001010
https://en.wikipedia.org/wiki/Radiation_pressure
https://physics.stackexchange.com/questions/55964/do-photons-actually-generate-a-slight-kinetic-force
https://www.encyclopedia.com/science/news-wires-white-papers-and-books/lasers-space
https://www.photonics.com/Articles/Lasers_Find_Varied_Uses_in_Space_Applications/a52252
https://www.nasa.gov/feature/jpl/lasers-in-space-earth-mission-tests-new-technology

Regarding radiation shielding: Would lead plates on the inside of a spacecraft protect the astronauts & cargo from radiation? Would the lead plates melt on reentry? Would lead poisoning be an issue?

Lead plates would certainly be effective, but they are also toxic as you mention, and very heavy. There are other materials (like water!) which are effective radiation barriers, which have fewer negative attributes, and could be used for other purposes, as well.
Jeff Stone — Former Flight Controller/Trainer @ JSC

How do you clean up spills- both simple (i.e. soup) and contaminants (i.e. )?

Astronaut Chris Hadfield demonstrates how simple spills are cleaned up on the ISS using rags, paper towels, and sometimes wet wipes. He then demonstrates using the Contaminated Clean-Up Kit to clean up some contaminant that might leak and cause problems. https://www.youtube.com/watch?v=8Hj3GnPRsJ4
Astronaut Chris Hadfield

Does NASA plan to use any omnidirectional treadmills in space in the future? If not, why?

Omnidirectional treadmills have potential for a variety of benefits (e.g. countermeasures for bone, muscle, cardiorespiratory, sensorimotor, psychological) but the associated large mass and volume are significant challenges for space exploration vehicles. Studies are being undertaken to determine whether a treadmill capability is required or if it will be sufficient to use only resistive and other aerobic modalities (eg “weightlifting”, rowing, cycling) which have smaller mass and volume footprints.
Kaitlin Lostroscio on behalf of the Exploration Exercise Device Lead — Roboticist/Biomechanics Specialist @ JSC

Regarding radiation: Do the BNNTs (Hydrogenated Boron Nitride Nanotubes) block all three types of Radiation (Galactic Cosmic Radiation, Solar Particle Events (Solar Flares), and Secondary Neutrons)? The raw BNNT matter can be turned into yarn. Would it be possible to weave this into a mesh to place inside the walls of the hull of spacecraft to block the radiation?

Regarding radiation – I have not considered BNNTs, but for passive radiation protection in space the concept is typically thought to be water or polyethylene, both for high density hydrogen within their molecules. The high density Hydrogen protons help to absorb energy from incoming neutrons as well as offer up electrons that slow down high energy ionized radiation approaching the habitat. However, both these take significant mass and multiple launches to make a reasonably protective shield. Perhaps the hydrogenated BNNTs could help. It certainly would be lighter.
I have previously proposed an active radiation shield using magnetic fields to turn away the incoming radiation. I believe this is the approach that is needed likely to be combined with a passive shielding method.
Mr. Shayne Westover — Mechanical Design Test Engineer @ JSC

The general process of radiation shielding is controlled first and foremost by mass, plain and simple. Second order effects are controlled by density, atomic number, etc. Water and plastics like polyethylene contain lots of hydrogen, which has the (~) same mass as protons and neutrons making it effective at stopping those particles just due to conservation of momentum. BNNTs have a lot of attractive mechanical and radiation-shielding properties, but their gross effectiveness is going to be driven by mass. Put it this way, to cut the flux of GRC iron in half you’d need many inches of aluminum given the broad range of kinetic energies. The same holds true for the amount of water. Water would be used in combination with the actual spacecraft structure. How much water you’d use will be constrained by the launch mass capability of the rocket balanced with everything else in the payload – unfortunately, not a simple answer aside from the technical/reliability and logistics complications of closed loop container system.
Jonny Pellish — Electronic Parts Manager @ GSFC

You are very much on the right track! Hydrogenated Boron Nitrite Nanotubes BNNTs are among the materials considered for protection against space radiation. In determining if it would be effective shielding for all three types, it’d be good to consider the characteristics associated with them and the properties provided by BNNTs. Galactic Cosmic Rays are of greatest concern from a health perspective. They consist of heavy, high energy particles like Hydrogen protons from other stars and supernovae which are travelling near the speed of light. There can also be Helium, Carbon, Oxygen, Neon, Magnesium, Silicon, Iron, and other nuclei. Solar radiation consists of x-rays, solar particles, and the like, from the Sun. Cosmic Background Radiation are microwaves from the early stage of the universe. “We know that hydrogen is effective at (1) fragmenting heavy ions such as are found in galactic cosmic radiation (GCR), (2) stopping protons such as are found in solar particle events (SPE), and (3) slowing down neutrons such as are formed as secondaries when the GCR and SPE interact with matter. Hydrogen, however, by itself is not a structural material. ” https://www.nasa.gov/directorates/spacetech/niac/thibeault_radiation_shielding.html . Materials such as polyethylene (a plastic) have been considered due to their high hydrogen content, but alone, it does not posses structrual characteristics desired for load bearing applications in spaceflight (we want to be able to integrate with the spacecrafts). You’ll find more information in that paper as to why BNNTs are a solution of interest. https://www.nasa.gov/sites/default/files/atoms/files/niac_2011_phasei_thibeault_radiationshieldingmaterials_tagged.pdf In summary: they can be processed for structural uses, the nanotube molecular structure is attractive for hydrogen storage, and boron has one of the largest neutron absorption cross sections of all the elements on the periodic table (and nitrogen can absorb as well). Connect the dots and we have quite an interesting multi-function solution! Another presentation of interest: https://www.nasa.gov/pdf/638828main_Thibeault_Presentation.pdf . However, integrating with the spacecraft structure is a challenge on its own as spaceflight has years of testing and processes in use for other materials. So your idea of adding a mesh to the walls is interesting! Since it has successfully been made into yarn, as you’ve said, it is flexible enough to be woven into the fabric of space suits: https://www.nasa.gov/feature/goddard/real-martians-how-to-protect-astronauts-from-space-radiation-on-mars . So, it could likely be a protective liner in spacecraft as well. On Page 5 of Thibeault’s report, a graph is provided showing the dose of GCR which passed through 30cm of different materials. BN is more effective than water, and when hydrogen is added, it becomes the best solution (better than polyethylene) according to the report. You can likely determine the weight from the figure on the following page (which provides density against dose for different materials; remember that a lower dose is better!). To determine how thick your mesh would need to be, consider the intensity of the source of radiation throughout a spaceflight mission, and an acceptable dose for the duration of the mission in order to determine some design requirements. I’d be interested to know what you find out! For more on space radiation: https://www.nasa.gov/pdf/284277main_Radiation_MS.pdf . Best of luck this season and thank you for thinking about big problems which must be solved in space exploration!
https://scidoc.org/IJNST-2167-8685-04-001e.php
Kaitlin Lostroscio on behalf of the Exploration Exercise Device Lead — Roboticist/Biomechanics Specialist @ JSC

When you launch on a Soyuz, is it smooth, harsh? Do you remember much of it? What do astronauts do in the Soyuz capsule while you’re on your way to joining the ISS?

> Mark Vande Hei: I definitely remember the launch. We train so much in a simulator that did a really good job of getting us used to that small space. Because it was so smooth, I was a little disappointed it wasn’t giving me a better story out of it. I had a little window over here [gesturing with his hands], and certainly the view out that window was different than the simulator, and mostly it was just the blackness of space. The one thing that was very different was when we went from one stage of the rocket to the next stage, there would be a momentary stopping of the acceleration, which felt like a jolt forward. So that was shocking as we changed, but otherwise it was very, very smooth…
> Joe Acaba: …Yea, until we got to the space station. And it’s a funny story, as we were approaching, you know, you go through the whole launch sequence. You have a few hours, like Mark said, you’re just checking out the systems. And you start to approach the space station, and you get your first glimpse out the window of this beautiful laboratory that we’ve built. And then Mark…
> Mark Vande Hei: …Which just happens to be at the same time the commander who is looking at the space station through his periscope. It’s a very intense time for him because we’re approaching another spacecraft. And he’s gotta be very ready to respond to a failure of the automatic system, because he has to do it manually, possibly. So it’s very tense for him. And I’m trying to support him all the time, but I get this view of these massive solar arrays out the window, and I was like, “WOW!” He just looked at me, and gave me the quiet sign…
> Joe Acaba: Yea, because we were hot mic’ed, so everybody could hear everything we’re saying.
> Mike Vande Hei: I was very excited…
> While waiting to dock at the ISS, we double-check everything. The Soyuz does a lot autonomously, so astronauts double check everything it does in case one system fails- they could take over manually. Time from launch to dock at ISS can be anywhere from 6 hours to 2 days, depending on the orbit line-up and other factors.
Source: https://www.youtube.com/watch?v=K0l1RJ4Aag8
Astronauts Joe Acaba and/or Mark Vande Hei

What about using a parabolic mirror as an energy source on the moon? Our research has shown they can concentrate the sun’s rays on Earth to temperatures of 500 degrees F. Does the moon’s lack of atmosphere or other conditions affect the use of a parabolic mirror to harness the sun’s rays? Are shadowed craters the best place on the moon to try to harness the energy of the sun to melt lunar ice?

Using a parabolic mirror on the moon is a great idea, and NASA is actually studying a concept similar to that in order to direct sunlight into deep craters to allow solar powered robots to operate in that environment.
https://spectrum.ieee.org/automaton/robotics/space-robots/transformers-could-make-harsh-lunar-environments-robot-friendly
The lack of atmosphere on the moon is actually a benefit in this case! No water molecules or clouds to get in between the sun and the reflector! As far as the best location on the moon to harness the suns energy, that would be the poles. If you consider the phases that the moon goes through throughout the month, you can see why. The moon is always facing us here on earth. And from our perspective, the moon goes from fully sunlit, to fully dark, and back again each and every month. So if we set up a solar reflector in the middle, or the equator, of the moon then it would be in total darkness for about 2 weeks out of each month. But the closer we get to the poles of the moon, the more days of the month we will be in sunlight.
http://aa.usno.navy.mil/faq/docs/moon_phases.php
In theory, the very top (North Pole) and very bottom (South Pole) of the moon will never see the sun go below the horizon. That’s just theory, though. In reality, the sun is not quite exactly constant anywhere on the moon. But it’s close enough at the poles.
https://www.space.com/957-perfect-spot-moon-base.html
The happy coincidence is that the same place where the sun almost never goes down on the moon (the poles), is the same place where water is most likely to be found in deep craters near the poles where the bottom of the crater is always in cold shadow, which keeps the water frozen and in place!
Kurt Leucht — Software engineer for Swamp Works NASA’s Kennedy Space Center

What kind of tech is used with fighting fires (thermal imagers, advanced firefighting agents, breathing equipment for astronauts, protective clothing for astronauts)? How are fires detected in space? Do smoke alarms work the same in zero-g? What does mission control do when there is a fire on the ISS? How do astronauts conduct firefighting training on earth and in space?

Smoke alarms in zero-g do work similar to those on Earth but smoke does not “rise” so the placement of the alarms is different. The ISS uses photoelectric smoke detectors placed on the ventilation filter intake ducts. When a smoke detector alarms, the ISS Crew and ISS Mission Control are notified. Alana (Whitaker) has compiled a presentation about ISS fire safety entitled “Overview of ISS US Fire Detection and Suppression System” at https://ntrs.nasa.gov/archive/nasa/casi.ntrs.nasa.gov/20030053429.pdf . Astronaut Samantha Cristoforetti explains training for an ISS fire in her blog entitled “Fire!” at http://blogs.esa.int/astronauts/2013/01/30/fire/ . Gary (Ruff), an engineer at Glenn Research Center’s Microgravity Combustion Science Branch, explains fires in zero-g at https://www.nasa.gov/audience/foreducators/k-4/features/F_Fire_Prevention_in_Space.html . Robert (Frost), a NASA Instructor and Flight Controller, explains ISS fire safety procedures very well, “Fire Safety utilizes a concept called the “Fire Triangle” – three things are needed for a fire: heat, oxygen, and fuel. Preventing, detecting, and suppressing fires is done by attending to sides of that triangle. First, we do our best to design the spacecraft to not have a lot of fuel. That means avoiding the use of materials that easily burn. Then we include ways to remove heat and oxygen. Each module, and some of the racks, has one or more smoke detectors. In microgravity, there is no natural air convection. In order to detect smoke, the smoke detectors are mounted inside ventilation pathways. Air is forced through these ventilation pathways by fans. If a fire is ignited, that fire will produce particulates (smoke) that will be carried by the ventilation to the nearest smoke detector. Should a smoke detector detect such particulates, it will annunciate a fire. The central computer will detect that annunciation and trigger alarms and also initiate automatic responses such as removing power from the rack (taking away a source of heat) and cutting off ventilation in that area to prevent the flow of oxygen to the fire. For most small fires, this should immediately extinguish the fire. The crew, upon hearing the fire alarm, will go to the source of the fire and insert a fire extinguisher nozzle into the closest fireport and fill the volume with fire suppressant chemicals. This should quickly extinguish any fires not extinguished by the automatic isolation response. Should a fire be too large for these methods to extinguish, the crew can seal off the entire module and air can be removed to starve the fire. To avoid excessive exposure to toxic byproducts of a fire, the crew can don masks that can either filter air or provide a dedicated source of oxygen. If procedures are followed, the crew should not be exposed to excessive risk when dealing with a fire. However, fire in space is certainly something that has to be respected and can be deadly. In February, 1997, a fire broke out on the Space Station Mir. That fire was particularly dangerous because it involved a lithium perchlorate canister. The job of that canister was to produce oxygen, making it very difficult to extinguish. Once it began to burn, the oxygen in the canister allowed the canister to get so hot that it began melting metal. The crew were unable to extinguish that fire, so they had to use the fire extinguishers to prevent the fire from spreading to nearby materials and structure until the canister eventually consumed all of its oxygen and the fire could be suppressed. There were a lot of lessons learned because of that incident and those lessons learned greatly reduce the chance of something similar happening on ISS.”
Lucien Junkin — NASA Space Exploration Vehicle (SEV) Chief Engineer

How does internet work on the ISS, and how reliable is it?

This article explains the internet capabilities on the ISS back in 2015, but the technology and capabilities have improved greatly since then. If you watch live-streamed videos from the ISS you can tell the connectivity is strong and reliable. =)
https://www.theatlantic.com/technology/archive/2015/06/the-internet-in-space-slow-dial-up-lasers-satellites/395618/

Regarding astronauts’ spacesuit gloves: We are writing to ask about the problem of nail loss in space due to astronauts’ nails making contact with the hard “thimbles” in the glove fingertips (https://news.nationalgeographic.com/news/2010/09/100913-science-space-astronauts-gloves-fingernails-injury/). We’re wondering how the gloves work and how they are made.

Fingernail trauma during Extra Vehicular Activity (EVA) operations, or spacewalks, has certainly been a problem over the years that has garnered much attention. The space suit designers at the NASA Johnson Space Center have captured an incredible amount of data in a publication entitled “Spacesuit Glove-Induced Hand Trauma and Analysis of Potentially Related Risk Variables” at https://ntrs.nasa.gov/archive/nasa/casi.ntrs.nasa.gov/20150003049.pdf ; the resources at the end of this research is tremendous. Amy (Ross) has been part of many of the glove designs and discussed the implementation of the Phase VI glove at https://www.nasa.gov/audience/foreducators/spacesuits/careercorner/amy-ross.html . Amy also has a nice video discussing gloves and a pioneer in space suits, Joe Kosmo, at https://youtu.be/5drXh2y_QI4?t=79 ; I have had the honor to work with both Amy and Joe. There are many resources for the glove design, here are a few: “Phase VI Advanced EVA Glove Development and Certification for the International Space Station” at https://pdfs.semanticscholar.org/16d3/a1976698a4cb800b67016f4d3ed81cd38d26.pdf presents a great description of the Phase VI Glove that Amy, Joe, and their colleagues at the NASA Spacesuit Lab developed; “The Effects of Extravehicular Activity (EVA) Glove Pressure on Hand Strength” at https://ntrs.nasa.gov/archive/nasa/casi.ntrs.nasa.gov/20100008464.pdf provides some great pictures without the outer shell of the glove and was also coauthored by Scott (England) who grew up participating in FIRST and has mentored many FIRST robotics team; “Enhancements to the ISS Phase VI Glove Design” at https://ttu-ir.tdl.org/ttu-ir/bitstream/handle/2346/59677/ICES-2014-67.pdf provides great information about the evolution of the glove that Amy & Joe helped create; and “Spacesuits and EVA Gloves Evolution and Future Trends of Extravehicular Activity Gloves” at https://pdfs.semanticscholar.org/4d88/035e71c1e53b978d4f2cebb9cbeeed7c666a.pdf presents a great history of spacesuit gloves. THANK YOU for being part of space exploration. -Lucien
Lucien Junkin — NASA Space Exploration Vehicle (SEV) Chief Engineer

What methods are used to kill mold on the ISS?

Generally, most in-home disinfectants are not allowed to be used on the ISS. The life support system is not able to clean it out effectively. Most general purpose wet wipes, baby wipes, etc… can be used. Benzalkonium Chloride wipes are the ones used for science experiments and more critical cleaning applications.
Lyndon Bridgwater — Aerospace Engineer @ JSC

How is potable water stored on the ISS?

The bags used to store water on the US segment of the ISS are called Contingency Water Containers (CWC). They were developed for use on the Space Shuttle and have been very effective for their task. They are “plastic” bladders with a fabric container to protect the bladder. They are not filled to capacity, so they remain rather “squishy”, and thus can be stored in different shape and size volumes. Sorry, but I am not sure this is a problem needing to be solved.
Jeff Stone — Former Flight Controller/Trainer @ JSC

What would be the primary argument for using humans for deep space exploration, rather than just sending robots?

Humans are more mobile. They can get from point A to point B rather quickly because we’re more flexible. When things go differently than planned, humans can make real-time decisions, and we can change our plans quickly and appropriately really quickly. Also our computer and our sensors- our eyes and our brain- that we humans have… It’s really hard to put the same level of sensor in a robot. Our brains are able to make decisions rather quickly, whereas robots have to phone home every time things don’t look right, and wait for Mission Control to tell it what to do next. Whereas humans can make real-time decisions because we’ve got a really great computer in our head- our brain- and we have these really great sensors- our sense of touch- we have sense of touch. We can reach out and touch thinigs. We can look at things and examine things with our very high resolution eye balls. So there’s a lot of benefits to sending humans to Mars. But there is a downside to sending humans to Mars… Humans do add some complications. They add some risk and some cost- quite a bit of cost, as it turns out. And when we eventually do send humans to Mars, they are going to live and work there for a pretty long time on the surface. Because of the transit time between Earth and Mars and the orbits of the different planets and how they only line up at certain times of the year, the first humans that we send to Mars are very likely going to live and work on the surface for about a year and a half. That’s a very long mission. To support a year and a half mission on the surface of Mars, it’s going to require lots of resources on the surface of Mars, lots of infrastructure… The astronauts will need lots of breathing air, lots of food, lots of rocket fuel to be able to get back home at the end of their mission. They’re going to need infrastructure like a habitat, a greenhouse, … They’ll need a logistics warehouse of sorts with all kinds of spare parts that they might need during their mission. And because of all that, we’ll need to send a lot of stuff to Mars in order to support our human explorers. Sending a lot of stuff to Mars can be expensive, it adds risk, and it can be technically difficult to get a lot of things sent all the way to Mars, and landed safely on the surface. So NASA has been thinking about these problems, and one way we’ve come up with to overcome at least part of that downside is to just take less stuff. So if we can live off the land, off the surface of Mars, and use less resources that are already there on the surface of Mars, then we’ll take care of at least some of these problems. http://www.ustream.tv/recorded/117572142
Kurt Leucht — Software Engineer @ KSC

Why does space appear so dark?

In space, you don’t see sunlight the same way we see it on Earth. We sometimes see rays of sunlight and we see color changes in the sky because of the sun’s light passing through the Earth’s atmosphere. The rays are being radiated by the moisture in the atmosphere, causing us to be able to perceive the light from the sun. In space, and on other planets, the atmosphere is significantly thinner than that of Earth’s atmosphere, so sunlight is not perceptible, unless it is being radiated or reflected off a surface. So the sky will appear black in space, but from space, you can see planets, moons, stars, and other celestial objects because the sun is reflecting off those objects.
At 28:30 of this video, Astronaut Steven Swanson shows what it looks like from the “cupola”, or the observation area on the ISS. It’s beautiful! Astronauts never want to leave this area! =)
https://www.youtube.com/watch?v=QvTmdIhYnes

What is the power limit of a spaceship’s thrusters?

Rockets work according to the Tsiolkovsky rocket equation: dv = g*I_sp*ln(initial mass/final mass). The dv term is called delta-v and represents the energy change in a spacecraft as a result of running the rocket. The g term is the gravitational constant (9.81 meters per second^2) and the Isp term is the specific impulse of the rocket. The specific impulse of a rocket can be thought of as a measure of how fuel efficient a rocket is at converting rocket propellant into increased energy of the spacecraft. This term is also related to the “power” level of the rocket which I will revisit in a moment. The “ln” means the natural log of the term in the parenthesis. The fraction in the parenthesis is the initial mass of the spacecraft divided by the final mass of the spacecraft after consuming the rocket propellant.
In order to get to Mars and capture from some reasonable starting orbit around Earth, a reasonable delta-v might be ~5 km/s. If you are using a rocket with similar performance to the shuttle main engines that use liquid oxygen and liquid hydrogen which yields a specific impulse of ~450 seconds, then with a starting spacecraft “wet” mass of around 90 metric tons (90,000 kg), the final “dry” mass after consuming propellant will be ~29 metric tons (28,997 kg). You asked about power. How much power is in the thrust of that rocket? To determine that power level we need to know the thrust and efficiency of the rocket engine. Let us assume that the rocket engine is actually a single shuttle main engine with thrust of 2,279,000 Newtons. The equation that relates thrust, power, efficiency, and ISP is F = 2*eff*Power/(g*Isp). If we assume the rocket is 100% efficient (efficiency factor of 1), then the power of the rocket is 5.03 Gigawatts. That is a whopping amount of power. Where does it come from you ask? This power comes from the chemical combustion process of the hydrogen and oxygen, so with chemical rockets you do not need an external power source.
If you chose to switch to an electric propulsion system to get higher specific impulse and reduce the amount of propellant, then you need to provide external power. For example, a Hall thruster has a specific impulse of around 2000 seconds. This would make the final “dry” mass of ~70 metric tons which is a lot less propellant. However, there are no space power sources (nuclear or solar panels) available in the Gigawatt size, or Megawatt size. Perhaps we could envision having a set of solar panels big enough to provide the spacecraft with 1 Megawatt of power. At 60% efficiency, the thrust from the bank of hall thrusters would be around 61 N. Yes, very small by comparison to the shuttle main engine. As a result, you will have longer transit times, so it is a trade of increased mass for payload versus tolerating longer transit times.
Sonny White — Propulsion Systems Engineer

Recognizing the ISS orbits in Low Earth Orbit within the Earth’s magnetosphere, is anything currently used on the ISS to block out radiation, or are the radiation exposure levels just monitored?

In general, radiation is just monitored through a system of sophisticated and evolving sensors onboard the ISS. The magnetosphere provides a measure of protection (though it’s also responsible for the South Atlantic Anomaly – SAA). The ISS itself provides a lot of material shielding just due to its mass. You don’t need much else in a low-inclination LEO.
Jonny Pellish — Electronic Parts Manager @ GSFC

Regarding using water for radiation shielding… Does that not cause any issues with the water being potable? It seems as if that water would be dangerous to drink. Is it not?

At the radiation levels in space, it’s not going to materially harm the water. We use high-energy radiation at ground level to sterilize different types of food. It would take *significant* amounts of radiation to cause changes that would be potentially detrimental – e.g., water inside a nuclear reactor.
Jonny Pellish — Electronic Parts Manager @ GSFC

Our team proposes scaling down laser communication to use on Earth since the speeds are so fast and it could reach remote areas. Are there any laser communication relays setup on Earth for regular use? Are laser communications used on Earth, or only in space?

Lasers are used all the time for communications on Earth. A lot of people have lasers sending to data to and from their homes over fiber optic cables (such as Verizon FIOS). There are some laser communication relays already flying in space on the European Data Relay System. This system provides laser links between an orbiting spacecraft and the relay spacecraft with a relay link back to earth using radio waves. NASA’s first laser communications relay, LCRD, will relay data using lasers both between the relay and the users and between the relay and earth.
David Israel — Laser Communications Expert, NASA GSFC

We have learned that Magnetic fields on earth reduce the amount of radiation. Has NASA created (or considered creating) artificial magnetic fields to shield astronauts and habitats from radiation when on Mars?

Magnetic fields can reduce the amount of radiation. Certainly magnetic fields can be a significant benefit during long duration deep space flight. This would also be the case on Mars. I suggest we work on a device that protects the living space or the “village” on Mars. I think that would make a good first step. In regards to a solution to prevent space radiation on a crew, a team of ours through NIAC, did a study to do just that. The study can be found online:
https://www.nasa.gov/directorates/spacetech/niac/2012_phaseII_fellows_westover.html
Mr. Shayne Westover — Mechanical Design Test Engineer @ JSC

How do you currently sterilize your science equipment onboard the ISS?

Currently sterile items are autoclaved before flight, packed, and stowed in sterile packaging. The question becomes how WILL we sterilize things on an as needed basis when we are on the Moon or Mars? Currently NASA keeps colony counts down on the space station by disinfection wipes. But for exploration, there are a couple of methods that could be used. In general, keeping volatile chemicals in an enclosed space is something we avoid. So most likely we will use heat in the form of steam (water vapor plus a heat source) or radiation (if we used a radioactive power source it could also be used to help sterilize instruments, such as Cobalt 60 for example). Vaporized hydrogen peroxide could also be used. Because of the limited mass and volume in a spacecraft, whatever we use will probably have a dual use.
Dr. J.D. Polk — NASA Chief Health and Medical Officer

How do astronauts know how much radiation they are receiving at any given moment on a spacewalk?

NASA is currently working on how astronauts can monitor at any given moment their amount of radiation exposure. See this experiment for more info: https://www.nasa.gov/mission_pages/station/research/experiments/417.html
This experiment describes how astronaut radiation levels are monitored during EVAs (extra-vehicular activities): https://www.nasa.gov/mission_pages/station/research/experiments/168.html

Is there a current way to detect solar flares, and if so what are some ways to detect them?

Here is information about detecting solar flares and what is used to detect them:
https://hesperia.gsfc.nasa.gov/sftheory/flare.htm
https://www.nasa.gov/feature/goddard/2017/nasa-detects-solar-flare-pulses-at-sun-and-earth
http://solar-center.stanford.edu/SID/activities/swm.html

What are some ways to detect gamma rays?

Here is info about how gamma rays are detected:
https://imagine.gsfc.nasa.gov/science/toolbox/gamma_detectors1.html
https://imagine.gsfc.nasa.gov/science/toolbox/gamma_detectors2.html
https://www.lanl.gov/orgs/n/n1/panda/00326398.pdf
https://www.youtube.com/watch?v=mc-wQwaUh_Q

For Moon exploration/colonization: Can you melt ore found on the moon? If so, could you extract Oxygen from them, and store the Oxygen in tanks?

Great question. The samples returned by the Apollo missions contain no free oxygen, nor any water, ice, or water-bearing minerals. All lunar rock and soil do, however, contain oxygen albeit combined with metals or nonmetals to form oxides. The oxygen can be extracted using thermal, electrical, or chemical energy to break the chemical bonds. The mineral ilmenite (FeTiO3), lunar Basalt, soil with high iron content (usually also has ilmenite, olivine, pyroxene), and also volcanic glass (particularly those with high iron content) can be used for oxygen extraction. The amount of oxygen depends on the percent of oxygen in the compound and extraction method.
Dr. J.D. Polk — NASA Chief Health and Medical Officer

Were the Heat Melt Compactor (HMC) and the “Trash-to-Gas” experiments a success? If so, are they used in spacecrafts now? How can we help enhance the current plan for trash disposal? Can the GRACE-FO be reinitialized to shoot a laser to burn up trash instead of being used to detect climate changes?

Thank you for your question! Great to hear you’re looking into one of those problems where solutions are still in development – means that there are still new approaches to be found. Just yesterday, I participated in a crowd source initiative to design a trash receptacle for future waste processing systems, so it is very relevant. Here is some additional information to aid you in your search and find the answers to your questions:
Lessons Learned from HMC: https://www.nasa.gov/sites/default/files/atoms/files/hmc-lessons-learned-v5.pdf
What is NASA’s HMC: https://www.nasa.gov/ames/heat-melt-compactor
HMC Development Progress Report: https://ntrs.nasa.gov/search.jsp?R=20170011317. (Direct Link: https://ntrs.nasa.gov/archive/nasa/casi.ntrs.nasa.gov/20170011317.pdf)
Requirements for a HMC: https://ntrs.nasa.gov/archive/nasa/casi.ntrs.nasa.gov/20160008947.pdf
NASA article seeking new ways: https://www.nasa.gov/feature/nasa-seeks-new-ways-to-handle-trash-for-deep-space-missions
Repurposing Space Station Trash for Power & Water: https://www.nasa.gov/content/repurposing-space-station-trash-for-power-and-water/
In considering multi-purpose use for orbiting satellites, such as GRACE-FO, something to keep in mind is that the current disposal process (burning up in atmosphere or returning to Earth) works for current needs, so having a device outside of the International Space Station that can burn up trash is not needed. However, as you may find in your research (using links above or otherwise), for future space missions outside of Earth orbit, we would not want to dump out into space, but also we want to take advantage of any resource we can to produce resources we need!
Kaitlin Lostroscio — Robotics Specialist @ NASA-JSC

Can you contain gases in space and be able to safely manage them?

We can contain gases in pressurized vessels. That is the safest way to store them in space. We can also “reclaim” gases. For example, we reclaim oxygen from the atmosphere, condense it, and store it in tanks for space walks.
Dr. J.D. Polk — NASA Chief Health and Medical Officer

Our team is intrigued by the situation of a helmet taking on excessive water as documented with Luca Parmitano in 2013. Are you still dealing with the water leak problem in space helmets, or has a solution already been implemented?

Normal moisture in the helmet from the crewmember includes humidity from exhalation and sweat. That moisture is condensed on a cold plate in the PLSS and returned to the active thermal control system. The pathway for the condensed water to get back to the thermal loop was blocked, which caused the water to flow back into the suit, which happens to be in the helmet. Having an additional absorptive layer under the comm cap would potentially prevent the water from being absorbed. The layer should not be covered up by the comm cap if at all possible.
Mitigations have been developed, but the problem has not been fully eliminated. The potential for the same problem to occur still exists, but strong procedural controls have been put in place to prevent the water flow blockage from recurring. An absorptive pad was also placed in the helmet at the water outlet to absorb some water, and a snorkel was provided to allow the crew to breathe air from the torso if the helmet filled with water.
There is an absorptive pad that (that I believe is polyacrylate) that is placed near the ventilation inlet, where the water from EVA 23 came from as described above. It is expected that water that comes out of this orifice will contact this absorptive pad and be soaked up. Because the material swells when absorbing water, the crew should be able to detect when water is entering the helmet and come back to the airlock by feeling the pad get larger on the back of the head. The pad is limited in the amount of water it can absorb, however. See this article from years ago describing these components: https://www.space.com/24027-nasa-spacesuit-snorkels-spacewalk.html
This varies from crew to crew because head sizes vary quite a bit and location of the head in the helmet varies as well. This would be a difficult number to provide accurately, and whatever is placed on or near the head would need to go through a process of approval by the crew (called a crew consensus report).
If this project is pursued further, there are improvements that can be made to the NASA solutions of the HAP and snorkel. The HAP does not provide positive verification that water is being absorbed from the ventilation loop inlet. In fact, this has happened after the initial event of EVA 23 and it was not found until after the EVA was completed. See this article: https://www.space.com/28657-spacewalking-astronaut-spacesuit-water-leak.html
Developing a detection method for use with the HAP or an alternative method altogether is still of value. Keep in mind that the environment inside the helmet is pure oxygen and all materials placed in this space must be compatible with pure oxygen and not be susceptible to auto-ignition (like many hydrocarbons) or cause other toxicity hazards.
Full Investigation Report: https://www.nasa.gov/sites/default/files/files/Suit_Water_Intrusion_Mishap_Investigation_Report.pdf
Nathan Howard — Automation & Robotics Systems Engineer @ NASA-JSC

What would happen if an astronaut/private space explorer were to die in space?

See these two articles regarding this question:
https://www.businessinsider.com/what-if-someone-dies-in-space-2015-4
https://www.cnet.com/news/what-happens-to-the-unprotected-human-body-in-space/

Could astronauts use a modified pressure cooker onboard a spacecraft to cook their own fresh food?

A pressure cooker is a solid idea but has some added issues to overcome because it is a pressure vessel. We require a 2.0 Factor of Safety (FOS) for pressure vessels. This means that everything has to be designed so that it can handle twice as much stress as what the requirement calls for. In your case, the pressure cooker has to have the ability to handle at least 6 atm of pressure. This is the same for any kind of temperature requirements you may have. Keep that in mind when reviewing your design. One other thing to think about is packaging. Try to make the whole thing fit into an EXPRESS Rack, preferably one section of one. We use these all over the place on ISS and it makes it really easy to install on station. Overall you have a good idea that could be more complete with a little more refinement. Remember, engineering isn’t just making something that looks right, it’s about proving your design works through analysis and testing. Keep up the good work!
Joe Altemus — Mechanical Design Engineer @ JSC

What happens when you sneeze in space?

Sneezing in space is pretty much just like sneezing on Earth, except you should aim your sneeze much more carefully. If you’re inside the spacecraft, in normal clothes or a flight suit, you should sneeze into the crock of your elbow, capturing as much of the sneeze spray as possible. If you are in an EVA spacesuit, that’s when it’s really complicated, since you don’t have a way to cover your mouth inside the helmet. You must point your mouth downward as much as possible so the sneeze sprays downward, rather than straight out, onto the interior surface of the helmet.

How will we remove perchlorates from Mars’ soil in order to grow plants for fresh astronaut food?

See these two articles regarding this question:
https://www.researchgate.net/publication/242525435_Perchlorate_on_Mars_A_chemical_hazard_and_a_resource_for_humans
https://www.space.com/21554-mars-toxic-perchlorate-chemicals.html

Regarding the Phase VI glove used for NASA spacewalks: Could NASA replace the various types of RTV (room temperature vulcanized) silicone rubbers with RTV silicone rubbers infused with graphene to improve both strength and flexibility? We realize that graphene is still being developed as an additive, but we found a couple of journal articles that specifically have worked on graphene-infused RVT silicone. What does NASA consider to be the most significant challenge with EVA gloves? What other solutions to making a more flexible, tactile glove are being researched?

Yes, the Phase VI gloves are the gloves American Astronauts would use during an EVA right now. There are ongoing projects to develop new and improved gloves, but I don’t believe any of them have advanced passed the prototype stage yet to the point of being validated for spaceflight.I don’t believe you could simplify it to a “single, most critical challenge”. Your hands basically become your means of locomotion during EVA, and that brings a variety of challenges. Your hands are tremendously dexterous, but maintaining that in a spacesuit glove is currently impossible. It simultaneously needs immense thermal protection, to protect against nicks and cuts while being compliant enough to enable a functional amount of hand mobility and strength. I will say that making a glove that has as good or better resistance to puncture (as current gloves) while improving astronaut hand mobility would be a tremendous asset to spacesuit usability by crewmembers.The mobility in spacesuit gloves mostly comes from mobility features (i.e. joints), not the flexibility of the materials involved. Most work to improve gloves that I’m aware of focuses on alternative designs of mobility features and improved fit of the gloves, but if some advance in material science enabled the gloves to be constructed with fewer layers, yes, that may assist in developing a thinner, more dexterous glove. I’m not aware of any work on alternative materials for spacesuit gloves, but you may take a look at any academic papers coming out of the spacesuit labs at MIT and University of Maryland. I’m not aware of this, mostly because I’m not aware of any commercially available sources of a graphene-enhanced fabric that could then be integrated into a spacesuit glove for testing. I would suspect based on my understanding of the current challenges with manufacturing graphene in large quantities that this doesn’t exist yet. New materials would also then need to go through rigorous testing to ensure that it would be appropriate for use in space. However, if a strong, lightweight graphene-based-material was verified to be available and appropriate for use in space, it could be explored in future prototypes.
I’m checking with the best assets I’ve got. For further research, the foremost expert on spacesuits at NASA that I’m aware of would be Amy Ross. She has quite a few journal articles, social media interviews, and talks on youtube that may be worth reviewing. These are a few assets I quickly found online that may be relevant:
https://pdfs.semanticscholar.org/4d88/035e71c1e53b978d4f2cebb9cbeeed7c666a.pdf
https://youtu.be/E0P3BID_6Io?t=2164
https://www.researchgate.net/publication/240611931_Phase_VI_Advanced_EVA_Glove_Development_and_Certification_for_the_International_Space_Station
https://youtu.be/d__xlXqYFZc
Scott English — Former Spacesuit Specialist @ JSC

How do NASA’s space communications networks work?

https://www.nasa.gov/open/space-communications.html

Is there any recycling or repurposing of plastic waste onboard the ISS?

https://www.nasa.gov/mission_pages/station/research/experiments/2356.html
https://www.3dprintingmedia.network/nasa-refabricator-3d-printer-iss/
https://ntrs.nasa.gov/archive/nasa/casi.ntrs.nasa.gov/20170011108.pdf
https://science.nasa.gov/science-news/news-articles/the-in-space-refabricatorx
https://ntrs.nasa.gov/archive/nasa/casi.ntrs.nasa.gov/20180006339.pdf
https://ntrs.nasa.gov/archive/nasa/casi.ntrs.nasa.gov/20170000214.pdf


Q&A about Astronaut Nutrition and Consumption during Spaceflight

What are the biggest challenges concerning food on the ISS?

The lack of fresh food & the amount of waste the food containers create are two of the biggest problems. We hope for a good amount of fresh food to be grown to become an accent to the standard menu options (i.e. salad, fruit, vegetables).
Astronaut Mike Gernhardt — Astronaut; Associate Division Chief of ER (Software, Robotics, & Simulation Division) @ JSC

The biggest challenges to providing food on ISS is that here is no dedicated refrigerators and freezers for food, so all food has to be shelf stable and last for a very long time at ambient temperature.
Vickie Kloeris — Manager ISS Food System @ JSC

How do humans eat in space (logistics)? What kinds of foods can we take to space?

Eating in space is kind of funny. Astronauts use and learn different techniques, and you can really have fun with it. Grabbing food and eating it on the fly is common. Rather than using a knife and fork, like we do on Earth, astronauts eat with scissors & a spoon. For de-hydrated foods, we re-hydrate the food with warm or cold water, cut the package open with scissors, maybe pour a sauce in to pouch for added flavor, dip the spoon into pouch, and the food sticks to the spoon. You get so used to being in space that you can let one piece of food float around you after taking a bite, while you drink your drink, which is in a pouch and requires both hands. “You can’t spill your milk in space.” Funny personal anecdote after spending some time on ISS: I was having a conversation with my wife and went to use my hands to talk. Being used to the microgravity experienced on the ISS, I moved my sandwich in front of my face and pulled my hands away so I could talk with my hands. My sandwich immediately fell to the floor, reminding me where I was.
Astronaut Mike Gernhardt — Astronaut; Associate Division Chief of ER (Software, Robotics, & Simulation Division) @ JSC

We bring along several different types of food when we fly in space. Since we don’t have a freezer, refrigerator, stove or microwave, most of the food has already been cooked, then freeze dried and vacuum packed (meaning the water and air has been taken out of it), or it is thermally stabilized (meaning treated and sealed in a package to prevent spoiling), much like camping food. These packages of freeze dried food can be reconstituted by adding water and then warmed up in a small warming oven. We don’t want food that makes crumbs in space since crumbs would float all over the place and that could clog up equipment! Peanut butter, in fact, is the nearly perfect space food. It has no crumbs and almost everyone loves peanut butter. We can bring fresh foods with us, as long as we eat them early in the mission before they spoil. Source: https://www.nasa.gov/centers/johnson/pdf/569954main_astronaut%20_FAQ.pdf
various astronauts

Humans eat in space in exactly the same way we do on earth. Food is provided in packaging, in either freezedried (like backpacking food), thermally processed (like canned food in a pouch), or natural form (like cookies, crackers, and granola bars). The food we send has to be sized so that it can be eaten in one or two bites, like a small cookie or cracker, or it has to be slightly sticky, like a chewy granola bar. The food can’t generate a lot of crumbs, because crumbs float around in microgravity and can get in crewmembers’ eyes or foul air filters and electronic equipment. Thermally processed food, like thick soup or pudding, is easily eaten with a spoon. Freezedried food, like scrambled eggs or macaroni and cheese, has to be rehydrated with water before the package can be opened and the food eaten. Food that is moist will stick to itself, the package, or the utensil, and won’t float out of the open package.
Kimberly Glaus Läte — Space Food Systems Laboratory Manager @ JSC

How are the astronauts’ nutritional needs determined, and how is food chosen in terms of its nutritional value? What is analyzed in foods that will be taken in to space?

Astronaut nutritional needs are determined by the scientists in the Nutritional Biochemistry Laboratory, who analyze samples of blood and urine from astronauts in spaceflight. They give those requirements to the Space Food Systems Laboratory, where food scientists develops a wide variety of nutritious foods to meet the nutritional requirements. The best way to get adequate nutrition is through a variety of whole foods, not through supplements.
Dr. Grace Douglas — Advanced Food Technology Lead Scientist @ JSC

Is there any negotiation as to what the astronauts will take in to space (meal/snack/beverage-wise)?

Nutritionists work with the astronauts on what their food preferences are for their time in space. They have the astronauts sample the options, and the astronauts pick & choose what they want. The nutritionists then look over the astronauts’ selections and make recommendations to have a more balanced nutritional profile. They are good about working with special requests, for example if you enjoy international foods, more spice, certain snacks, etc. The ISS astronauts have their self-made “Pot-luck drawer”: Sometimes you try things on Earth and they don’t end up tasting the same in space, or you think you’ll like the taste of something on the menu, but you find you do not. Astronauts use what they call a “pot luck drawer”, where everyone puts their food items they do not want, as an exchange system. Chances are, someone else on the ISS will like the item, so they can eat it, rather than letting it go to waste.
Astronaut Mike Gernhardt — Astronaut; Associate Division Chief of ER (Software, Robotics, & Simulation Division) @ JSC

Can you cook in space?

What we think of as cooking isn’t really possible in microgravity – having surfaces hot enough to cook foods would present a hazard to the crewmembers who might burn themselves. Expedition 18 flight engineer Sandra Magnus did use some special techniques to enhance the space foods she had on hand. Sandra used a large Ziploc bag and knife to cut onion pieces, which she put in a foil bag and heated in the food warmer to soften and “cook” the onion. Sandra also combined foods together to create new dishes and add variety to their menu. Most ISS crewmembers have not been that inventive, but many like to use condiments, such as Sriracha sauce, garlic paste, sundried tomato paste, and pesto to add new flavors to their favorite foods.
See link: https://www.nasa.gov/mission_pages/station/expeditions/expedition18/journal_sandra_magnus_7.html
Kimberly Glaus Läte — Space Food Systems Laboratory Manager @ JSC

Do astronauts eat regular meals like we do on Earth- breakfast, lunch, and dinner? Do they share food with other astronauts from other countries?

Astronauts eat regular meals and snacks, just like we do on Earth. The full ISS crew typically eat together once a day at the crew table. The U.S. supplies mostly re-hydratable foods in pouches (MREs – Meals Ready to Eat), whereas much of the Russian food comes up hydrated, i.e. soups like lamb stew in a can. Astronauts from different countries enjoy sharing food with each other. For example, the Americans tend to enjoy the soups and stews from Russia, while the Americans have a shrimp cocktail that the Russians love.
Astronaut Mike Gernhardt — Astronaut; Associate Division Chief of ER (Software, Robotics, & Simulation Division) @ JSC

Do the astronauts take any vitamins/supplements while in space? If so, which and how often?

Astronauts take a daily multivitamin, but most nutrients are provided by the food. A diverse, whole food system with lots of fruits and vegetables is the best way to obtain adequate nutrition. Most vitamin D is obtained through exposure to sunlight, which astronauts are not exposed to in a space vehicle, so vitamin D is provided as a supplement.
Dr. Grace Douglas — Advanced Food Technology Lead Scientist @ JSC

What is the most common food-related complaint among astronauts?

The lack of variety of space food is probably the biggest complaint among astronauts. Just like we do on Earth, they get tired of the same foods over and over, only they don’t have the flexibility to change their options the way we do. The U.S. supplies mostly re-hydratable foods in pouches (MREs – Meals Ready to Eat), whereas much of the Russian food comes up hydrated, i.e. soups like lamb stew in a can. The crew tries to grow green onions and other simple plants for food. They do get fresh tortillas. The astronauts always get excited when a cargo ship arrives with fresh food. They tend to miss a lot of their favorite meals from home- cheeseburgers, pizzas, steak, barbecue, etc. The idea of an “infinite buffet” has been suggested to maintain variety and give astronauts choice. Essentially, there would be options of 10 base ingredients stored in bulk (meats, sauces, pastas, veggies, etc.). An astronaut would grab a big bowl, mix whichever ingredients he/she wanted, add water, and stir to “cook” the meal.
Astronaut Mike Gernhardt — Astronaut; Associate Division Chief of ER (Software, Robotics, & Simulation Division) @ JSC

How is the food packaging made durable enough for prolonged space travel?

Our primary concern with the food packaging is to protect the food inside from interacting with light, oxygen, and moisture. We use plastic films and laminated packaging materials to maintain low oxygen transmission and water vapor transmission rates, to minimize the exposure of the food product. By doing this, we extend the shelf life of our packaged foods to approximately 2 – 3 years, depending on the product. This is a sufficient amount of time for the ISS flight food system, but we will need a longer shelf life for the food systems we develop for planetary habitations, such as a lunar or Mars base.
See this article & the embedded videos:
https://www.nasa.gov/content/space-food-systems
https://www.eriesd.org/cms/lib/PA01001942/Centricity/Domain/1041/Space%20food%20packagingbrochure.pdf
Kimberly Glaus Läte — Space Food Systems Laboratory Manager @ JSC

Are there any recent developments in space food (production, storage, quality/desirability)?

We are investigating new technologies to produce space food, including microwave assisted thermal sterilization (MATS), which uses 915 MHz frequency and can sterilize foods faster than retort (canning) process, resulting in higher quality. This is not the same type of microwave in your kitchen, which uses 2450 MHz. This is just an example of a technology we are investigating, with the goal of achieving a five year shelf life. Advances in other technologies, including cold storage (low-resource requirement) and food packaging (high barrier, strong, light weight, process compatible) would also be beneficial.
Dr. Grace Douglas — Advanced Food Technology Lead Scientist @ JSC

Regarding using bees on long distance space travel to provide honey and then also to establish a colony on the new planet. From our extensive research we have seen that the last time they tried to take bees into space was the 1990s. This experiment did have some positives, can we find details of the original container/microenvironment created for them?

NASA has indeed flown bees into space multiple times. The most successful experiment was the “A Comparison of Honeycomb Structures Built by Apis Millifera” experiment (SSIP Experiment SE82-17), flown aboard the Space Shuttle mission STS-41C in April 1984. An internal Johnson Space Center report summarizing the experiment is available online at https://lsda.jsc.nasa.gov/books/publication/se82_17.pdf
The “Bee Enclosure Module” (BEM) that contained the experiment is currently stored at the Smithsonian National Air and Space Museum Udvar-Hazy Center. Photographs of the metal, plastic, wood, and paper BEM, along with dimension information, are available online at https://airandspace.si.edu/collection-objects/bee-enclosure-module-sts-41-c and https://www.si.edu/es/object/nasm_A19880031000?width=85%25&height=85%25&iframe=true&back_link=1&destination=spotlight/human-spaceflight The final science report from the experiment (“Survival, Behavior and Comb Construction by Honey Bees, Apis Mellifera, in Zero Gravity Aboard NASA Shuttle Mission STS-13”), with additional descriptions and photographs of the BEM can be found at https://hal.archives-ouvertes.fr/hal-00890665/document
Dave Lavery — Program Executive for Solar System Exploration

How do the astronauts know if their food is in good enough condition to consume?

All the food is dated with expiration dates. De-hydrated foods don’t really spoil. Stabilized food comes in aluminum packaging, so the only reason it would spoil is if it gets torn open and is exposed to air. There is very little risk of eating spoiled space food.
Astronaut Mike Gernhardt — Astronaut; Associate Division Chief of ER (Software, Robotics, & Simulation Division) @ JSC

The crewmembers living and working on the ISS can have high confidence in the food system. It is produced and packaged under very clean conditions, and submitted to multiple inspections and quality testing prior to be stowed in flight food containers and delivered to the ISS. However, the crewmembers can use sensory cues, just like you and I do when we consume food on earth. If something doesn’t look, smell, or taste the way we expect, we don’t eat it. The crewmembers use these same techniques to decide whether or not to eat a food product on-orbit.
Kimberly Glaus Läte — Space Food Systems Laboratory Manager @ JSC

What is the current food storage technology?

Currently food is stored in boxes full of packets of 6-day supply of foods for each astronaut. Space food is produced and packaged under very clean conditions, and submitted to multiple inspections and quality testing prior to be stowed in flight food containers and delivered to the ISS. However, the crewmembers can use sensory cues, just like you and I do when we consume food on earth. If something doesn’t look, smell, or taste the way we expect, we don’t eat it. The crewmembers use these same techniques to decide whether or not to eat a food product on-orbit. All the food is dated with expiration dates. De-hydrated foods don’t really spoil. Stabilized food comes in aluminum packaging, so the only reason it would spoil is if it gets torn open and is exposed to air. There is very little risk of eating spoiled space food.
https://images.nasa.gov/details-Expedition_56_Education_Interview_with_Microsoft_Youth_Spark_Live_2018_0813_689813.html
https://images.nasa.gov/details-Expedition_56_Education_Interview_with_McAuliffe_Shepard_Discovery_Center_2018_0809_688241.html
Expedition 56 Flight Engineer (Astronaut) Ricky Arnold + Kimberly Glaus Läte — Space Food Systems Laboratory Manager @ JSC

Is lack of fresh food for astronauts an issue?

Lack of fresh food for astronauts could be an issue. There have been several studies sponsored by the Human Research Program to look at growing food in space (“Pick and Eat”, for example). When astronauts are located far away and we can’t send fresh fruits and vegetables regularly this becomes more of a concern.
Dr. Sherry Thaxton — Human Health & Performance division @ JSC

What is the composition of the tortillas which are taken into space?

The composition of the tortillas is as follows: Enriched bleached wheat flour (wheat flour, niacin, reduced iron, thiamine mononitrate, riboflavin, folic acid), water, vegetable shortening (interesterified soybean oil, hydrogenated soybean oil), glycerin, salt, contains 2% or less of the following: leavening (corn starch, sodium bicarbonate, sodium aluminum sulfate, monocalcium phosphate, calcium sulfate), mono- and diglycerides, wheat flour, guar gum, soybean oil, enzyme, dough conditioners (fumaric acid, L-cysteine), preservatives (calcium propionate, sorbic acid).
Kimberly Glaus Läte — Space Food Systems Laboratory Manager @ JSC

If fresh, high protein, nutritious food were made available (i.e. grown on the spacecraft), would that be beneficial?

There are some people at NASA who have looked quite a bit into the concept of using algae as food, so there’s definitely a thought that it could be beneficial. There are also some disadvantages to consider such as astronauts not finding it appetizing, impacts of microgravity on growth, and microbiology concerns.
Dr. Sherry Thaxton — Human Health & Performance division @ JSC

Would 3-D printing food be a realistic and desirable option in space?

3-D printing technology needs more development before it can be used to print food in space, but 3-D printing would enable precision nutrition in a customized food form. This would help astronauts get some of the nutrients they may need in the correct amounts, even if they begin to degrade in the prepackaged food system. It would also provide some variety and food choice on a long mission with a closed food system.
https://www.nasa.gov/directorates/spacetech/home/feature_3d_food.html
Dr. Grace Douglas — Advanced Food Technology Lead Scientist @ JSC

How do astronauts re-hydrate food, and could that process be improved?

Skip to 31:40 in this video to see Astronaut Steven Swanson demonstrate how the potable (drinkable) water dispense works on the ISS. The potable water dispenser/rehydrating station has been onboard the ISS since 2010. The astronauts don’t really have any complaints about the potable water dispenser. Either cold or hot water can be dispensed, depending on what the water is for or what type of food they are re-hydrating.
https://www.youtube.com/watch?v=QvTmdIhYnes
https://images.nasa.gov/details-Expedition_56_Education_Interview_with_Microsoft_Youth_Spark_Live_2018_0813_689813.html

Are there any studies using meal replacement powders or liquid, such as Soylent, for future Missions to Mars? Do NASA’s food engineers have any studies on using spices to extend the shelf life of foods or beverages going into space?

We have looked at replacing a meal with nutritionally complete bars or shakes to provide mass savings for spaceflight. However, meal replacements are not as acceptable as a regular menu with more choice. Our goal is to provide variety to meet nutritional requirements and support some crew choice, even within the limited system. We use spices to provide flavor, but processing (drying, freeze drying, and retort thermostabilization) and high barrier packaging work best to extend the shelf life.
Dr. Grace Douglas — Advanced Food Technology Lead Scientist @ JSC

Has NASA ever tried instant cold packs to make things colder?

NASA does not use chemical “cold packs” on orbit for food or chemical heaters for food. This is due to the concern for these items giving off chemicals into the air inside the space station.
Vickie L. Kloeris — NASA Food Scientist Emeritus

Why aren’t there normal refrigerators/freezers in space for food?

NASA had normal refrigerators on Skylab. While designing the Skylab Space Station, which was operational from 1973 to 1974, the engineers had an extensive amount of refrigerator and freezer volume for a very Earth-like menu, including ice cream! Yep! The Skylab astronauts ate ice cream! You can discover what the Skylab engineers and scientists were designing at https://ntrs.nasa.gov/archive/nasa/casi.ntrs.nasa.gov/19720004405.pdf , https://ntrs.nasa.gov/archive/nasa/casi.ntrs.nasa.gov/19720004406.pdf , and https://ntrs.nasa.gov/archive/nasa/casi.ntrs.nasa.gov/19720004403.pdf . You can also discover exactly what each astronaut ate on Skylab that was refrigerated and frozen including ice cream and lobster at https://ntrs.nasa.gov/archive/nasa/casi.ntrs.nasa.gov/19720004403.pdf . For example, astronaut Joe Kerwin for his dinner would have to get his shrimp and prime rib out of the freezer. You can discover more about Skylab and the “galley” at https://ntrs.nasa.gov/archive/nasa/casi.ntrs.nasa.gov/19730025070.pdf . As freeze-dried technology improved and the “cooks”, or food scientists, improved at preparing tastier freeze-dried meals, NASA went away from food refrigerators and freezers but continue to use cold storage for experiments. In 1993, NASA concluded research began in 1985 entitled “Space Station Thermal Storage/Refrigeration System Research And Development” that is at https://ntrs.nasa.gov/archive/nasa/casi.ntrs.nasa.gov/19940010718.pdf . Since it is resource intensive to keep food cold while the spacecraft is on the launch pad, while it is travelling to the ISS, and while it is awaiting consumption, NASA opted for non-perishable, or “shelf stable”, food similar to soldier rations, Apollo food, and Space Shuttle food for the ISS. The “Meal, Ready to Eat”, or MRE, has greatly evolved over the years and is now “pretty good” food. Vicki (Kloeris), food scientist in the NASA Johnson Space Center Food Lab, explains all about ISS food including how the food is shelf stable so there is not a need for a refrigerator or freezer but how the astronauts have a “chiller” to make their beverages cold at https://youtu.be/JGQ344xx6rM . With all this information, there is certainly good arguments to introduce refrigerators and freezers for long duration space flights, especially when foods are grown in space. THANK YOU for being part of space exploration. -Lucien
Lucien Junkin — NASA Space Exploration Vehicle (SEV) Chief Engineer

Regarding farming in space: It seems like there is research underway to produce food from solid waste. Is it to be used as fertilizer? Has NASA tried to use solid waste for farming in space? Spirulina – is the research close to being put into use? Due to the sticky nature of water due to the surface tension – would hydroponic work better for farming in space?

There has been ground research in the past to consider how to recycle and reuse inedible plant mass. One can consider a bioreactor or using heat to produce a biochar. Solid human waste typically contains pathogens which can make crew members sick, so solid waste would need special consideration to be used. In the near term, many parts of the plants grown for food cannot be eaten and are good candidates for recycling as a compost, bioreactor, or perhaps a biochar.>br> We just completed a National Lab Experiment on the ISS called, Space Algae: https://www.nasa.gov/mission_pages/station/research/experiments/2552.html
Algae is a candidate organism for feed stocks and possibly nutrition supplementation, but unlikely as a major food source for now.
The challenge of using hydroponics in microgravity is even more difficult. On Earth, in 1g the water will stay in the trough and bubbles manipulated to keep the dissolved oxygen concentration high for the plant roots. Hydroponics is a good solution for the moon and Mars because at about 0.1g bubbles rise again however we still have to pay careful attention to corners and geometry because the force pulling the water down is much weaker to that on Earth which means the surface tension of the water can still impact system performance and suffer failures. So short answer, yes hydroponics is a great choice on surface systems. We are working with microgravity fluids experts on microgravity hydroponics designs for crop plants, but are in the very early stages.
Yes, NASA has and is considering aeroponics. The downside of aeroponics is that if any part of the system fails then the crops are effected negatively very quickly (hours to respond and repair). In the case of hydroponics, a system failure does not impact plant health as quickly since the roots are still in a nutrient solution (1g and partial g). You may be interested to know that we are working with microgravity fluids experts on how to use modified hydroponics systems in microgravity and other teams on how to use aeroponics systems in microgravity both with the goal of growing crop plants.
Trent Smith — VEGGIE Project Manager @ NASA’s KSC

Veggie PONDS are being used now on the ISS for growing plants. What plant do you think they should try to grow next and why? Some of the plants are being sent back to the US to be studied and the rest are being eaten. What have they learned from the plants that were returned to Earth?

Veggie PONDS design is still being fine tuned, and once complete, we will have two space pots to grow crop plants in VEGGIE. It is exceptionally difficult to water plants in microgravity due to the way water behaves. Plant roots need both O2 and water. Currently we use the VEGGIE rooting pillows, and with them very recently (10/25/2018), Dr. Serena M. Auñón-Chancellor started a crop of Red Russian Kale and Dragoon Lettuce. The next crop after Veg-03 G is Veg-03 H (yes, we are very imaginative). That crop is Wasabi Mustard and Extra Dwarf Pak Choi. These crops were chosen because many vitamins in the packaged diet degrade with time. Vitamins K, C, and B1 all degrade, and plants being natural molecular architects can supplement those vitamins for deep space astronauts to keep them healthy. The Kale, Wasabi, and Pak were chosen for the nutrient content and Dragoon lettuce scored highly in the organoleptic assessment (scientific taste test). Also dragoon lettuce had one of the highest crop yields of plants we’ve tested. We do ask the astronauts to share their harvest with our scientists on the ground. Half the harvest is returned for food safety analysis and to measure mineral content and other tests. For the most part, as long as the plants have the 5 cardinal factors of plant growth, they do quite well and we haven’t noted any major differences. Furthermore, and to me, most importantly, is that the plants the crews have eaten have tasted good and provided new textures and smells, which improves their experience eating, which can help with diet fatigue. I hope that my response is helpful!

Trent Smith — VEGGIE Project Manager @ NASA’s KSC Various resources regarding growing plants in space:
http://www.ustream.tv/recorded/117798919
https://www.nasa.gov/mission_pages/station/research/news/flowers
https://www.nasa.gov/mission_pages/station/research/experiments/1294.html
https://www.nasa.gov/mission_pages/station/research/experiments/2621.html
https://www.nasa.gov/feature/can-plants-grow-with-mars-soil
https://www.skyandtelescope.com/astronomy-news/some-plants-grow-well-in-martian-soil/
https://www.acs.org/content/acs/en/education/resources/highschool/chemmatters/past-issues/2016-2017/april-2017/growing-green-on-the-red-planet.html
https://www.popsci.com/nasa-growing-food-in-space#page-5
https://eos.org/articles/tests-indicate-which-edible-plants-could-thrive-on-mars
https://www.nasa.gov/mission_pages/station/research/news/meals_ready_to_eat
https://www.nasa.gov/mission_pages/station/research/Giving_Roots_and_Shoots_Their_Space_APH
https://www.nasa.gov/feature/how-does-your-space-garden-grow
https://www.nasa.gov/mission_pages/station/research/news/sprouts_in_space.html
About plant pillows: http://gravitationalandspaceresearch.org/index.php/journal/article/download/749/777
http://cssf.usc.edu/History/2009/Projects/J2019.pdf
http://cssf.usc.edu/History/2009/Projects/J2002.pdf
https://blog.waikato.ac.nz/physicsstop/2015/10/05/plants-in-circular-motion/
https://www.reddit.com/r/askscience/comments/12nush/how_would_a_plant_react_to_being_grown_while/
https://www.nasa.gov/vision/earth/technologies/aeroponic_plants.html
https://www.maximumyield.com/fogponics-a-new-spin-on-aeroponic-gardens/2/1401

The growth media and pots are being sent back to Earth so that the growth media can be tested for microbial contamination. What are the results so far? What do we need to do to be able to reuse the growth media and pots? On Earth, hydroponic growth media can be sterilized and reused. Is it possible to sterilize it in space? In the ISS Microgravity Science Glovebox, astronauts use UV light for decontamination after an experiment. Could the same system be used to sterilize pots and growth media?

Right now in VEGGIE, we do not re-use the plant pillows. VEGGIE is being used to learn how to grow crop plants and eat them safely. Food safety is a big driver, and one of our main challenges is how to wash space vegetables to ensure they are safe to eat. You are absolutely correct in considering how to keep the system clean and useful… Further down the road, when astronauts are routinely supplementing their diets with fresh food in a more sustainable food production system, the use of UV light is one consideration. However it is limited by the fact that those microbes in the shade are safe. That means that in order to be effective, surfaces generally need to be smooth and the full strength of the light needs to be directed to surfaces of interest. Other ways to sanitize include using plasma, which is easily generated under ambient conditions, but has its own challenges. In the future, we hope to grow plants without the use of granular media because it is heavy, it would be exceptionally difficult to process in microgravity, and it may not be needed. Our primary challenge with fresh food production is watering the plants, and an effective precautionary step to sanitize the produce. It is also important to recognize that produce has different textures, shapes, and sizes, thus one size may not fit all. Good question, I hope my response is helpful.
Trent Smith — VEGGIE Project Manager @ NASA’s KSC


Q&A about Physical Health of Astronauts during Spaceflight

Does the workout equipment onboard the ISS only help the muscles, or does it help the bones as well?

Exercise not only helps strengthen muscles but also aids bone formation! Why? It is good to consider why bone atrophy occurs in the first place. A good way to think about it is “use it or loose it”. On Earth, we have gravity and so the greater our body mass or the more books we carry in our backpack, the greater the load (force) our bones had to support to keep us standing up against gravity. On the International Space Station, there is only microgravity (barely any at all). Without gravity, our bones don’t have to work hard to support us. Our body thinks we don’t need them anymore, so they start to waste away (especially our lower limbs which are most used to carrying a load). But we need to keep our bones strong for when we return to Earth or explore other planets. So, how do we load them? Using resistive (“weight-lifting”) exercise on the Advanced Resistive Exercise Device (ARED) and by running on a treadmill while wearing a harness that pulls on you. Exercise doesn’t completely solve the problem because there is only ~2 hours a day where you are loaded and the rest where you are not (imagine if you laid in bed all day and only got up for 2 hours)! Ongoing research looks into medicines and nutritional supplements which can even be paired with exercise to make it more effective. Sometimes even the way the exercises are done help with effectiveness (short bursts throughout the day rather than two hours straight for instance). Other equipment and technologies are being experimented with to help as well. Also worth mentioning – exercise helps with other health issues as well. Aerobic (fast) exercise helps keep our lung capacity up to size, and exercise in general helps us mentally and keeps our other body systems working (cardiovascular system, immune system, etc.). If you’re interested in more on the science, you can research “bone remodeling in space” to find some good videos too. We call exercise and nutrition as some “countermeasures” for these health risks. Are there any new countermeasures or approaches you can think of? Best of luck this season!
Kaitlin Lostroscio — Biomechanics Specialist/ Mechanical Engineer in the Software, Robotics, & Simulation Division @ JSC

The exercise equipment on the ISS help with both muscle and bone density loss. There are two treadmills on space, a stationary bike, and an Advanced Resistive Exercise Device (ARED), which allows for a multitude of workout options for the entire body. https://www.nasa.gov/audience/foreducators/stem-on-station/ditl_exercising https://science.howstuffworks.com/exercise-in-space2.htm Video of Astronaut Scott Kelly demonstrating the many uses of ARED: https://www.youtube.com/watch?v=YxImeOomkUk Video @ 2:59 shows ARED: https://www.youtube.com/watch?time_continue=1&v=jbZ7lDlVeIo Video @ 1:00 shows the exercise bike: https://www.youtube.com/watch?time_continue=310&v=ntYP7cRozhk Video @ 20:44 shows the treadmill: https://www.youtube.com/watch?v=QvTmdIhYnes Treadmill: https://www.youtube.com/watch?v=P9VBkCpO-JU Info about ARED: https://www.nasa.gov/mission_pages/station/research/experiments/1001.html Info about COLBERT (Combined Operational Load Bearing External Resistance Treadmill): https://www.nasa.gov/mission_pages/station/research/experiments/765.html ; https://www.nasa.gov/mission_pages/station/behindscenes/colberttreadmill.html

What does micro-gravity, reduced-gravity, and radiation do to the human body? How is the effect of micro-G, reduced-G, and radiation on the body mitigated?

According to Expedition 56 Flight Engineer Ricky Arnold, on his first trip up to the ISS, it took 3-4 days before he really felt normal and perfectly comfortable. However, on his second trip up to the ISS, he immediately felt fine. It was as if his brain remembered being in space and just immediately adjusted. Source: https://images.nasa.gov/details-Expedition_56_Education_Interview_with_McAuliffe_Shepard_Discovery_Center_2018_0809_688241.html https://science.nasa.gov/science-news/news-articles/your-immune-system-in-space https://www.independent.co.uk/news/science/the-12-ways-your-body-changes-during-long-term-space-flight-according-to-nasa-a6773536.html See links on protecting astronauts from dangerous radiation in deep space travel: https://www.nasa.gov/feature/goddard/real-martians-how-to-protect-astronauts-from-space-radiation-on-mars https://www.nasa.gov/directorates/spacetech/strg/nstrf2016/Passive_Radiation_Shielding https://www.space.com/41887-mars-radiation-too-much-for-astronauts.html You can learn more about the hazards of radiation and spaceflight at: https://www.nasa.gov/hrp/hazards https://www.nasa.gov/johnson/HWHAP/hazard-1-radiation Microgravity affects an astronaut’s height! They get a little taller while on long-duration space flights. Normally on Earth, the weight of the body (due to gravity) squishes down on the intervertebral disc tissue in our spines (as well as the cartilage in joints like our hips and knees); however, in space, the body is “weightless”. Those squishy tissues expand, puffing up with fluid. When the astronauts return to the surface of the Earth, they return to their previous height as they re-adjust to living in 1G. The same sort of thing happens to people on Earth when we sleep. We are about 1 cm taller when we first get up in the morning due to the lack of load on our spine.
Expedition 56 Flight Engineer (Astronaut) Ricky Arnold

What causes a slowing of the cardiovascular system in space, what are the effects of this, and what is done to prevent this?

https://www.nasa.gov/mission_pages/station/research/experiments/756.html
https://www.nasa.gov/mission_pages/station/research/experiments/320.html
https://www.nasa.gov/mission_pages/station/research/cciss_feature.html

What causes fluid distribution issues throughout the human body while in space, what are the effects of this, and what is done to prevent this?

https://www.nasa.gov/mission_pages/station/research/experiments/1257.html
https://www.independent.co.uk/news/science/the-12-ways-your-body-changes-during-long-term-space-flight-according-to-nasa-a6773536.html

What causes bone density loss in space, and what is done to prevent this?

See link: https://www.nasa.gov/mission_pages/station/research/benefits/bone_loss.html

Does space travel increase the risk of an astronaut developing cancer? If so, what causes the increased risk? How is the risk mitigated?

Astronauts are unlikely to develop cancer while they are in space, but because they are outside the Earth’s protection of the atmosphere, they are exposed to higher levels of radiation on the space station than we experience on Earth. When astronauts venture to the Moon and on to Mars, they will leave the protection we have on Earth as a result of the van Allen radiation belts, and they will be more susceptible to a particular kind of radiation called galactic cosmic radiation (GCR). Additional exposure to radiation that astronauts experience in space increases the likelihood that they will develop cancer later in their life. It does not affect their ability to continue their mission, and it affects different astronauts in different ways, even if they are on the same mission. NASA sets limits for astronauts’ permissible exposure to radiation. The current limit is at 3% excess risk of death from cancer. The limit is the same for men and women, but every individual has other risk factors. For example, an older astronaut with less expected lifetime overall will have a lower overall risk because they have fewer years of life left in which to develop cancer. Cells that are exposed to radiation can die or can mutate. Some mutations and propagation of the mutation are what cause the cancer.
You can learn more about the hazards of radiation and spaceflight at:
https://www.nasa.gov/hrp/hazards
https://www.nasa.gov/johnson/HWHAP/hazard-1-radiation
https://www.youtube.com/watch?v=3jRxgvwhGC0&feature=youtu.be
Stephanie Schierholz — Public Affairs Officer, Human Exploration & Operations @ HQ

How does NASA monitor astronaut health during space travel?

https://www.nasa.gov/mission_pages/station/research/experiments/1025.html
https://www.ncbi.nlm.nih.gov/pubmed/20519129
http://www.asc-csa.gc.ca/eng/sciences/bio-monitor.asp
https://space.stackexchange.com/questions/23205/does-the-iss-have-a-watch-system-with-at-least-one-person-monitoring-at-all-time

Do any of our astronauts at all show effects of radiation exposure when they return from missions?

Space radiation health risks: https://www.nasa.gov/hrp/elements/radiation/risks
General space radiation article: https://www.nasa.gov/feature/space-radiation-won-t-stop-nasa-s-human-exploration
Mars radiation environment briefing to the NASA Advisory Council: https://www.nasa.gov/sites/default/files/atoms/files/mars_radiation_environment_nac_july_2017_final.pdf >> See pages 22-24 in particular. There’s a lot of other good info in the other charts, too.
Jonny Pellish — Electronic Parts Manager @ GSFC

How do you go to the bathroom in space?

The space potty is a little different than the Earth potty. First of all, to keep from floating away, you must use foot-loops or straps while ‘sitting’ on the seat. This holds you on to the seat, sort of like a seat belt. Secondly, the space potty uses suction, not water, to ‘flush.’
Source: https://www.nasa.gov/centers/johnson/pdf/569954main_astronaut%20_FAQ.pdf
See video starting at 9:08: https://www.youtube.com/watch?v=doN4t5NKW-k
https://www.youtube.com/watch?v=0xI6coAS85c
various astronauts

How do astronauts brush their teeth in space?

These days, astronauts on the ISS are able to use the toothpaste of their choice. Sometimes they share the tube of toothpaste among the crew members in nearby living quarters on the ISS.
Skip to 2:40 in this video to see Astronaut Suni Williams demonstrating how astronauts brush their teeth in microgravity: https://www.youtube.com/watch?v=tBVUTFPate0 >> She shows that you can either swallow the water & toothpaste from brushing your teeth, or you can spit in to a paper towel.
Astronaut Chris Hadfield demonstrates this, as well: https://www.youtube.com/watch?v=3bCoGC532p8
This response to a question posed online shows a NASA astronaut’s & an astronaut trainer’s response to the question about the safety of astronauts swallowing the toothpaste after brushing their teeth: https://www.quora.com/International-Space-Station-Is-it-ok-to-swallow-toothpaste-as-Chris-Hadfield-does-in-this-video
various astronauts

How much exercise do astronauts do each day onboard the ISS, and how much of each, for example ARED vs aerobic? Are you specifically using any pharmacological countermeasures?

We exercise about 2.5 hours a day up here, every day. It is hard time-lined into our schedule. That’s about an hour and a half on ARED, which is our Advanced Resistive Exercise Device, and that is our weight-lifting device, and it’s… kinda think of it as a bowflex for station. We can do squats, bench press, dead lifts, you name it… We do that pretty much 6-7 days a week. And then we spend another hour every day doing aerobic exercise, whether it’s on our treadmill, where we wear a special harness, and that basically provides loading, or gravity, to pull us down, or we work out on an exercise bike. So we take a good chunk of every day’s timeline for all six of us up here onboard the ISS to get all of our exercise in. But we’ve seen the studies, we’ve look at the research, we know that this is imperative, and we know that for even longer missions, heading out towards Mars, it’s going to be even more important. We’re lucky up here on station… station is big! It is the size of a 5-bedroom house. So we can have a massive weight-lifting machine. We can have a treadmill. How do we accommodate that on a smaller vehicle, or maybe we wont have a bigger habitat until we get to where we’re going to, so those are some of the challenges a lot of our engineers are dealing with right now.
Specifically looking at pharmaceuticals, there is nothing standard that we use. We take a daily multivitamin, but we get most of our nutrition from the food. We don’t take any specific pharmaceutical countermeasures, like the bisphosphonate (osteoporosis medication), for example, to counter the bone loss.
Source: https://www.youtube.com/watch?v=LVCW_ubmwJ0&index=2&list=PLTUZypZ67cdv9X4DxjUGvN4wfN6EdHTUo
Astronaut Serena M. Auñón-Chancellor (M.D.)

What is the most challenging physical (health or safety) problem an astronaut faces in space? Please describe how this problem is being mitigated.

The ascent into space is very risky and nerve-wrecking. Also, spacewalks are mentally and physically demanding because you’re working against the inflation pressure of the spacesuit the whole time, which is 4.3 pounds per square inch (the same as a basketball or football). (https://www.nasa.gov/audience/foreducators/spacesuits/home/clickable_suit_nf.html) You have a large amount of mass with no drag to slow you down, so once you get moving, you can go tumbling out of control if you get moving too fast. As a rule of thumb, an astronaut cannot go too slow on a spacewalk. You can never let your hands get moving faster than your brain. You must always be aware of the location of your tether, your buddy’s tether, the airlock, etc. Your mind is racing, trying to fully comprehend your situational awareness and manage your task timeline. You must be very methodical with your management of your tether, foot restraints, body restraint tether, and hand holds. It’s important to have 3 points of stability at all times during a spacewalk. Using the body restraint tethers and hand holds make spacewalks efficient, requiring only 5-10 seconds to set them the way you need them. If an astronaut should become de-tached from their tether, they do have a back-up plan to get back to their spacecraft! https://www.nasa.gov/missions/shuttle/f_saferspacewalk.html
Astronaut Mike Gernhardt — Astronaut; Associate Division Chief of ER (Software, Robotics, & Simulation Division) @ JSC

Does the astronauts’ dizziness continue over time on the ISS?

On Earth, sensory information from your inner ear and your vision match up and tells your body where you are. When you go in to space, the fluid in your inner-ear is floating around, so depending on how you move, you end up with conflicting stimuli that can cause nausea. I did not feel dizziness in space, but many do. Eighty-five percent of astronauts experience dizziness and/or nausea for a few hours, and a few have even been sick for 2-3 days with minor discomfort.
Astronaut Mike Gernhardt — Astronaut; Associate Division Chief of ER (Software, Robotics, & Simulation Division) @ JSC

What causes loss of taste/change in taste buds?

https://www.scientificamerican.com/article/taste-changes-in-space/
https://www.independent.co.uk/news/science/the-12-ways-your-body-changes-during-long-term-space-flight-according-to-nasa-a6773536.html
https://www.nasa.gov/audience/foreducators/k-4/features/F_A_Matter_of_Taste.html

What causes vision impairment in space, what is done to prevent this, and is it irreversible? What are the effects of intracranial pressure, and what is done to prevent this?

What causes vision change: https://www.youtube.com/watch?v=laMmaGFZFZI
Cause & monitoring of vision change & intracranial pressure: https://www.youtube.com/watch?v=A7bFzviLgTQ
https://www.nasa.gov/analogs/envihab/optic-health-in-astronauts
https://www.nasa.gov/feature/peek-into-your-genes-nasa-one-year-mission-investigators-identify-links-between-genes
https://www.smh.com.au/lifestyle/astronaut-scott-kelly-on-the-devastating-effects-of-a-year-in-space-20170922-gyn9iw.html
https://www.washingtonpost.com/national/health-science/the-mysterious-syndrome-impairing-astronauts-eyesight/2016/07/09/f20fb9a6-41f1-11e6-88d0-6adee48be8bc_story.html?noredirect=on&utm_term=.dad3baef170c
https://news.nationalgeographic.com/2016/11/nasa-astronauts-eyeballs-flattened-blurry-vision-space-science/
And some of the specific investigations:
https://www.nasa.gov/mission_pages/station/research/experiments/explorer/Investigation.html?#id=1124
https://www.nasa.gov/mission_pages/station/research/experiments/explorer/Investigation.html?#id=1008
https://www.nasa.gov/mission_pages/station/research/experiments/explorer/Investigation.html?#id=1126
https://www.nasa.gov/mission_pages/station/research/experiments/explorer/Investigation.html?#id=197
https://www.nasa.gov/mission_pages/station/research/experiments/1950.html
https://www.nasa.gov/mission_pages/station/research/experiments/1038.html

What causes renal stones in space, and what is done to prevent this?

https://www.nasa.gov/mission_pages/station/research/experiments/1018.html

What causes muscle loss in space, and what is done to prevent this?

https://www.youtube.com/watch?v=lssYrWDvv6w&feature=youtu.be
https://www.nasa.gov/mission_pages/station/research/news/cell_mechanosensing
https://www.nasa.gov/mission_pages/station/research/benefits/nibr
https://www.nasa.gov/pdf/64249main_ffs_factsheets_hbp_atrophy.pdf

What causes balance disorders/loss of proprioception in space, what is done to prevent this, and does this tend to have a lasting effect once back on Earth?

https://www.nasa.gov/mission_pages/station/research/experiments/1768.html

Is motion sickness/Space Adaptation Syndrome a common problem among space travelers? If so, what is done to prevent/reduce this?

For some, this is a brief problem, just like some people experience car sickness or boat sickness at varying levels, while others do not experience it at all. The body adjusts relatively quickly to the new environment.
Astronaut Mike Gernhardt — Astronaut; Associate Division Chief of ER (Software, Robotics, & Simulation Division) @ JSC

Is muscle fatigue an issue for astronauts?

Muscle fatigue may be an issue for astronauts, though the main concern is deconditioning of muscles. We provide very specific exercise and nutrition in order to avoid that.
Dr. Sherry Thaxton — Human Health & Performance division @ JSC

No, muscle fatigue is not an issue for astronauts, as you are not fighting gravity, so you expend less energy than you do on Earth. You don’t use your legs in space the way you use them on Earth (for walking); you walk with your hands using hand holds, rails, etc. Astronauts need to do exercise for both cardiovascular health and lower-body strength maintenance. Proprioception (being aware of where your body parts are and how much strength is applied behind your movements) is a challenge in space. It takes time to adjust, but once you adapt, you get really efficient at translating and stream-lined glides. When you stay on station for longer period of times, you adjust better than those who travel on shuttle (or other short-term spacecraft) missions due to the amount of time.
Astronaut Mike Gernhardt — Astronaut; Associate Division Chief of ER (Software, Robotics, & Simulation Division) @ JSC

What steps are taken to prevent germs and illness in spaceflight and onboard ISS? What provisions are in place onboard ISS should an astronaut develop an illness? Should an astronaut get sick during a mission to Mars, what would happen?

We keep some medical supplies and some equipment, such as ultrasounds, on the space station to treat potentially sick astronauts. One of the crew members also is designated as the Medical Officer for the flight, and receives some specific training. The entire crew has 24/7 access to medical doctors, nutritionists, and other support staff on the ground, and can have video teleconferences with them as needed. We tend to select, as astronauts, people who are very healthy. Astronauts also spend some time in quarantine prior to their flight to try to limit the likelihood they’ll get sick. As a result, we haven’t had major illnesses on station. We do study it though – Scott Kelly even gave himself a flu shot in space: https://www.nasa.gov/content/flu-vaccine-in-space. The most common is space sickness as a result of no “up” in space. Many astronauts experience that and throw up. The most important thing when that happens is to try to contain the vomit as much as possible, and clean it up quickly, before pieces float everywhere… Each astronaut has their own crew quarters, like a little vertical bedroom, so they could stay there if they were really sick. And if they experienced some sort of life-threatening illness or something requiring surgery, we could put them (and their crew mates) back into the Soyuz in which they arrived, and they could be back on Earth within a matter of hours. That won’t be an option on a journey to Mars; they would have to keep going, because you can’t just turn around. The space station was assembled by humans, and it does host some animals onboard, but in general it has fewer germs and viruses than we experience on Earth. Astronauts regularly clean the space station, too. And we’re learning how to sequence DNA in space so they could quickly identify viruses they find: https://www.nasa.gov/mission_pages/station/research/news/BEST_DNA_RNA
Stephanie Schierholz — Public Affairs Officer, Human Exploration & Operations @ HQ

Is Galactic Cosmic Radiation the main type of radiation that causes cell mutation? How much radiation does it take to cause the type of mutation that will give you cancer?

Galactic cosmic radiation is more dangerous for deep space travel because on Earth we’re protected from it, so the primary challenge is how to protect astronauts from it on missions.
Every body responds a little differently to radiation, so we can’t just say that a certain dose will cause cancer – it would increase the likelihood. Cancer isn’t contagious from person to person either.
A couple more pages that may help explain radiation more:
https://www.nasa.gov/analogs/nsrl/why-space-radiation-matters
https://www.nasa.gov/feature/goddard/real-martians-how-to-protect-astronauts-from-space-radiation-on-mars
Stephanie Schierholz — Public Affairs Officer, Human Exploration & Operations @ HQ

Regarding astronaut hygiene, specifically toothpaste: What is in Nasadent? Do a lot of astronauts use it? Is there a suction device in the hygiene area?

Concerning NASAdent, astronauts use regular toothpaste on the International Space Station but you can find a description of NASAdent at https://ntrs.nasa.gov/archive/nasa/casi.ntrs.nasa.gov/20020091883.pdf . Concerning “What is in NASAdent?”, it has many of the same ingredients as regular toothpaste except for the ingredient Sodium Lauryl Sulfate, or SLS, which is the chemical in toothpaste to create the foaming action. There are many astronauts that are ready to show you how to brush your teeth in space. Chris (Hadfield) from Canada brushes his teeth for you at https://youtu.be/3bCoGC532p8 . Kimiya (Yui) from Japan brushes his teeth for you at https://youtu.be/vcF5QacG8qA . Suni (Williams) gives you a tour of the ISS including brushing her teeth at https://youtu.be/ukws3oLMDc8?t=184 . Garrett (Reisman) from America does a quick brush at https://youtu.be/1lec-tv0uY8 . Remember to brush your teeth for two minutes as the American Dental Association recommends. And lastly, Samantha (Cristoforetti) shows you all about hygiene including brushing your teeth at https://youtu.be/nPUvzn3CTQc?t=96 . Concerning “Is there a suction device in the hygiene area?”, there are suction devices in the hygiene area but they are not used for “spitting” your toothpaste; astronauts either swallow their toothpaste or spit it into a paper towel. THANK YOU for being part of space exploration. -Lucien Information on NASA’s experiment to determine how periodontal tissues are affected by space flight: https://www.nasa.gov/mission_pages/station/research/experiments/495.html
Lucien Junkin — NASA Space Exploration Vehicle (SEV) Chief Engineer

Regarding cancer risk: Does re-entering Earth’s atmosphere cause any damage that would cause cancer to develop?

Re-entering Earth’s atmosphere does not cause damage that would cause cancer – in fact, that’s when the astronauts are coming back to the protection from radiation that our atmosphere provides. Re-entering Earth’s atmosphere causes astronauts’ spines that have stretched out from being in freefall in orbit around Earth to compress again. It causes them to feel the weight of gravity. The fluids that have spread out through their body go back to normal. There are lots of other effects from coming back to Earth’s gravity, but they are actually more protected from radiation.
A couple more pages that may help explain radiation more:
https://www.nasa.gov/analogs/nsrl/why-space-radiation-matters
https://www.nasa.gov/feature/goddard/real-martians-how-to-protect-astronauts-from-space-radiation-on-mars
Stephanie Schierholz — Public Affairs Officer, Human Exploration & Operations @ HQ

What time do the astronauts usually go to bed and wake up? (using CST for the sake of simplicity) Does NASA monitor the astronauts’ sleep patterns?

The exact time astronauts onboard the ISS go to sleep varies, but they typically go to bed around 4pm CST and wake up around 1am CST. Their work day time is such that they’re awake part of the time during Houston’s workday and half during Moscow’s workday.
Justin Ridley — Former Flight Controller for ISS Missions @ JSC

What time do astronauts usually exercise? (let’s use CST for the sake of simplicity)

The time of day astronauts exercise varies by astronaut and by day, due to schedule and exercise equipment availability. There are two treadmills, a stationary bike, and the ARED machine on board, so all 6 crew members can’t work out at the same time each day.
See link on examples of ISS astronaut schedules: https://www.quora.com/Whats-the-typical-daily-schedule-in-the-international-space-station

Do astronauts use caffeine and/or sleep-promoting medicine to combat fatigue?

Astronauts can drink coffee in space when they need a caffeine pick-me-up: https://www.nasa.gov/feature/national-coffee-day-java-in-zero-g
Astronauts can take a sleep-promoting medicine, if they are having trouble winding down at the end of their day.

How many calories a day (on average) do astronauts consume?

Astronauts eat three meals a day – breakfast, lunch and dinner. Nutritionists ensure the food they eat provides them with a balanced supply of vitamins and minerals. Calorie requirements differ for astronauts. For instance, a small woman would require only about 1,900 calories a day, while a large man would require about 3,200 calories.
https://spaceflight.nasa.gov/living/spacefood/

What effects does space flight have on your body?

A ‘normal’ body will adapt to the ‘abnormal’ environment of space in many ways. Immediately upon entering zero gravity, fluids in your legs and the lower part of your body move upwards towards your head. In fact, your face will feel and look swollen. Except for the occasional headache and congestion, astronauts aren’t bothered by this fluid shift. Some astronauts feel dizzy and have an upset stomach during the first few days of a space flight as they get used to zero gravity. This feeling usually goes away after three or four days. After a few days almost everyone is used to zero gravity and feels great. If you don’t exercise, your bones and muscles will get weak. Upon return to Earth, you must get used to gravity again. Sometimes this makes you feel dizzy or queasy. If you are returning from a two week flight, readjusting might take a day or two. If you are returning from a six month flight, it may take several weeks to feel ‘normal’ again.
Source: https://www.nasa.gov/centers/johnson/pdf/569954main_astronaut%20_FAQ.pdf
various astronauts

What do you do if you get sick in space?

We carry first aid kits and some special medical equipment into space with us. We also are trained to take care of most minor and some major medical problems. There is not a doctor on most flights, but all crewmembers are trained in basic first aid and CPR. We can talk to doctors on the ground if we need help. If someone gets very sick in space, we can make an emergency trip back to Earth.
Source: https://www.nasa.gov/centers/johnson/pdf/569954main_astronaut%20_FAQ.pdf
various astronauts

Do American astronauts use resistance suits on the ISS? If so, please explain. If not, are there particular reasons why not? Do astronauts use the “Penguin Suit”? If not, why not? What about the SkinSuit? Do they still wear that, and does it work?

American astronauts wear resistance suits on the International Space Station (ISS). For example, you can see astronaut Terry Virts in a Penguin Suit at https://images.nasa.gov/details-iss043e188795.html . Samantha (Cristoforetti) discusses her Penguin Suit and shared a group photo at https://twitter.com/astrosamantha/status/428095955515109376 where Chris Cassidy from Canada, Aleksandr Misurkin from Russia, Karen Nygen from America, and Luca Parmitano from Italy are all in Penguin Suits. Resistance suits are worn during spaceflight to mimic the effects of gravity on the body, thus counteracting the loss of bone and muscle in weightlessness. Concerning the SkinSuit, the SkinSuit was evaluated on the ISS by astronaut Andreas Morgensen; for up-to-date information on the evaluation, please visit https://www.nasa.gov/mission_pages/station/research/experiments/2081.html . The SkinSuit has also been evaluated in the Exercise Countermeasures Lab at the Johnson Space Center in Houston. Concerning “Does the SkinSuit work?”, yes, it “works” but more evaluation will be conducted to determine how effectively the SkinSuit “works”. Furthermore, you can find an evidence report that discusses the Penguin Suit and SkinSuit entitled “Human Research Program Human Health Countermeasures Element” at https://ntrs.nasa.gov/archive/nasa/casi.ntrs.nasa.gov/20150009486.pdf . THANK YOU for being part of space exploration. -Lucien
Lucien Junkin — NASA Space Exploration Vehicle (SEV) Chief Engineer

Regarding the use of resistance devices to help stretch and strengthen muscles: How would you measure if a resistance device helped? Is there a company or school in America working on this technology? Our hope is for an improved resistance device/tension unit to be designed to help both astronauts and people fighting Duchenne’s & other similar conditions.

Excited to hear that you are working on a solution that could help both humans with prolonged exposure in microgravity and humans on Earth who also need countermeasures for muscle atrophy (and may be restricted in how they can exercise). Many of the health effects in spaceflight connect with rehabilitation on Earth, so spin-off technologies are often produced and technology transfer is a goal for NASA. A lot of times, it can work vice versa too, where methods used on Earth could be applied as solutions to those spaceflight problems. I can share a few approaches for resistance exercise used on the International Space Station and planned for future space travel. Resistance type exercise devices include the Advanced Resistive Exercise Device – an all in one machine for weight lifting type exercises. It has been effective for exercise on the ISS, but design constraints for future exploration spacecraft call for smaller/more compact and lighter devices. Currently, single or double cable/strap devices are being tested. (Search for MED-2, HULK, ROCKY, DART, ATLAS). Motors, gearing, and flywheels are used inside of these devices a computer controls the load so you can get very specific force patterns which simulate what it feels like to lift free weights or even row. This opens the possibility of having non-constant loads and you can administer forces desired for application while you are in a certain position. This gives control whereas something like elastic bands does not always administer the load as desired (also, with bungees you have to be careful you are protected if they break)! The cables/straps on these devices can be attached to various bars/handles and harnesses for a variety of exercises. Astronauts have also tested resistance suits like a “penguin suit” https://images.nasa.gov/details-iss043e188795.html or Skinsuit https://www.nasa.gov/mission_pages/station/research/experiments/2081.html . There is another type of exercise device which, reconfigured, doubles as an exoskeleton. https://ntrs.nasa.gov/archive/nasa/casi.ntrs.nasa.gov/20150001646.pdf . Nutritional countermeasures/biochemistry is also implemented and has been shown to help exercise be more effective. In the future, electrical muscle stimulation may be of interest as well. Do you see an application for any of these technologies in Earth rehabilitation?
Regarding measuring if a resistance device helps, sometimes it is difficult to isolate which countermeasure (exercise, nutrition, medicine, sleep, etc.) maintained the muscle. It could be a combination of them. However, studies are conducted though where astronauts only use an exercise device while others use exercise and supplements. Sample sizes are usually small though. In the end, you are going to have to measure/inspect/compute muscle mass, strength, integrity, and the like. This is done pre-flight, during flight on the ISS, and post-flight. Ex: https://www.nasa.gov/mission_pages/station/research/experiments/352.html . As a biomechanics analyst, I work with computer models for bone and muscle loading. An open source tool utilized is OpenSim http://opensim.stanford.edu/ which has the capability to find activation-to-force conditions. Muscle lengths can be observed as well. If there is a target goal, then we can test which device best provides the desired effects and determine how best to use the device as well. I encourage you to conduct further research on how muscle health is measured in spaceflight and on Earth. If muscle health is maintained or strengthened after starting to use a resistance device (and nothing else has changed), then you know that the resistance device has helped.
Kaitlin Lostroscio — Robotics & Biomechanics Engineer

How do astronauts measure their muscle tone and bone density while on ISS? What type of equipment is used? Is anything currently done to measure astronauts’ bone density while in space?

Here’s a great source on Bone Densitometer use on the International Space Station: (https://www.nasa.gov/mission_pages/station/research/experiments/1148.html). Mineral density is an important measure for bone health, though there are other measures as well. Scanning bone structure provides information for determining bone strength. The TBone study uses this technique – mission page: https://www.nasa.gov/mission_pages/station/research/experiments/1777.html, video: https://www.youtube.com/watch?v=vqBE0F77LPM. The “Early Detection of Osteoporosis in Space” study uses this technique as well: https://www.nasa.gov/mission_pages/station/research/experiments/619.html. Results from the research so far show the need for stronger targeted countermeasures for trabecular (spongy) bone loss, even after return to Earth: https://www.ncbi.nlm.nih.gov/pubmed/28574653. So the combination of these tools and studies – rodent research included – can be very important in providing us a better understanding of the efficacy of our countermeasures (exercise, medicine, nutrition) and the effects of prolonged exposure to microgravity. With enough data, we can build computer models which help us to look into gaps in data and predict the effects of different countermeasures: https://ntrs.nasa.gov/search.jsp?R=20140006747. There may be ways to take a holistic approach to the problem as well. Other body mass measurement activities could aid: https://www.nasa.gov/mission_pages/station/research/experiments/640.html. Each technique has pros/cons and better measurement methods are always of interest. Do you have an idea for a method which would be easier to implement and take less crew time? Or maybe it uses less power, is easier to hold, or gives results quicker? Could your idea be used on Earth and in space? Best of luck developing your solutions this season!
Kaitlin Lostroscio — Robotics & Biomechanics Engineer

Body mass is measured in space to determine the muscle tone. There are a few types of equipment on the ISS. One is the Advanced Resistive Exercise Device (ARED); it uses adjustable resistance piston-driven vacuum cylinder along a flywheel to provide loading for crew member to experience a load and maintain muscle strength. Another one is the treadmill called the Treadmill Vibration Isolation System (TVIS). There is also a Cycle Ergometer with Vibration Isolation System (CEVIS); astronauts strap their shoes into the buckles and wear a seatbelt to tie them down.
https://www.nasa.gov/audience/foreducators/stem-on-station/ditl_exercising
https://science.howstuffworks.com/exercise-in-space2.htm
Ms. C — Education Specialist

I do not believe there is a current method to measure bone density on-orbit. I am also not sure about muscle tone, per se. But muscle mass can be measured by ultrasound while on orbit. Muscle strength is not currently measured on-orbit, except potentially through functional strength tests (such as mid-thigh pull). Determining strength of muscle groups of isolated joints can be accomplished through dynamometery, although the only device (hypothetically) technically capable of this on ISS is MARES, and that device is not currently utilized for different reasons. It is the hope of many physiologists that technology currently being developed at JSC’s Wearable Robotics Laboratory will soon fly to ISS to allow isolated joint dynamometry to determine changes in muscle strength during a mission.
Mr. Christopher Beck — Engineer/Project Manager, JSC Wearable Robotics Laboratory

If we made a contraption or suit for astronauts to wear for resistance (i.e. perhaps an exoskeleton suit), what materials should we use?

Wearable exercise suits have been prototyped a few different times over here at JSC. There wouldn’t be one specific type of material across the whole system. Materials are just another tool at an engineer’s disposal. For wearable things we want something strong but light. We want the mechanisms to resist human movement but not make them feel like they are carrying around a ton of weight. This becomes less important in space because of the micro-gravity but if we are using these devices on the ground, weight is very important (one big use on the ground for this would be for physical therapy after an injury). Aluminum is one of the most common materials in flight hardware because of its strength and weight. Carbon fiber is another good candidate for this type of application but it is much harder to work with. It really comes down to what the final design looks like and what pieces are feeling the highest forces.
This link describes the recent creation from a partnership between NASA and Rice University, based off the technology learned from Robonaut2’s robo-glove & the X1 exoskeleton, to create an augmented shoulder joint: https://www.nasa.gov/feature/nasa-s-newest-wearable-technology-takes-on-the-human-shoulder
NASA’s Exoskeleton suit (“X1”): https://www.nasa.gov/offices/oct/home/feature_exoskeleton.html , https://www.youtube.com/watch?v=BDMFmwASYpo
NASA’s current suit technology: https://www.nasa.gov/feature/the-next-generation-of-suit-technologies
Joe Altemus — Mechanical Design Engineer @ JSC

I suggest looking at exosuit designs for this application. You will find athletic wear-type fabrics that are breathable, which is important if the suit is exerting force on your body. Here is an example: https://wyss.harvard.edu/multi-joint-personalized-soft-exosuit-breaks-new-ground/
Asher Lieberman — NASA Engineer

Do astronauts have issues with their skin thinning, drying out, and/or becoming more prone to cuts, rashes, and infections? What do astronauts use to help keep their skin healthy in space?

In 2006-2007 a Skin Care experiment was performed on the ISS. See: https://www.nasa.gov/mission_pages/station/research/experiments/11.html. A 2015 article from a manufacturer discusses the same experiment. See: https://shizukany.com/2015/05/best-skin-care-product-ask-an-astronaut/
Robert Kershaw — NASA Engineer

Regarding bone density loss: Does NASA use vibration therapy? If so, is it whole body vibration or is it isolated to a certain area? Does NASA use compression clothing of any sort? Would it be feasible for astronauts to wear a cuff or ‘sleeve’ of some sort throughout their daily routine?

NASA has looked at vibration and other modalities to transfer force through the bone. Some companies have their chickens standing on vibrating plates in order to encourage larger drumsticks! But in a spacecraft, vibration can cause several problems. It can interfere with critical hardware and electronics, and it can interfere with critical experiments, such as protein crystallization. In fact, we have systems that isolate vibration on exercise devices in order to not impact that vibration to the vehicle.
See link about field testing of compression garments for lower body: http://www.marketwired.com/press-release/nasa-research-finds-bsns-compression-garments-help-astronauts-stand-up-without-fainting-2151785.htm
Dr. J.D. Polk — NASA Chief Health and Medical Officer

Regarding muscle atrophy in space: 1. Has NASA used or considering using PEMF (Pulsed electro magnetic field therapy) for preventing or treating muscle atrophy? 2. Is Electro magnetic therapy used for treating astronauts with any type of ailment?

NASA uses ultrasound and electric stimulation for acute musculoskeletal system injuries on the ground just like any physical therapy or sports medicine facility would. The challenge in using such modalities such as electromagnetic fields in space is that they can impart current to the vehicle, and may interfere with other systems or represent a heat load or fire risk depending on how they are used or implemented, and how their systems are isolated. But hot packs, cold packs, and other therapies can be used on orbit to help relieve aches and pains from a physical exertion.
Dr. J.D. Polk — NASA Chief Health and Medical Officer

As an active flight surgeon, I can say I am not aware of any use of EM therapy for astronauts on any regular/routine basis. I am sure NASA has considered PEMF and/or other forms of electro-stimulation to counter microgravity induced muscle atrophy but I do not know of any specific documentation of such analyses. I will add that I’ve been at NASA for 13+ years now and there are very few ideas not entertained by our team. None-the-less, we realize we’re not “know it alls”, and I’m personally always happy to get suggestions and questions from non-NASA folks. This helps us discover new things and makes our final products all the more robust.
Flight Surgeon at NASA’s JSC

Is astronaut sweat collected and recycled onboard the ISS?

Skip to 1:14 in this video to see Astronaut Chris Hadfield explaining how water/sweat is evaporated and picked up by the de-humidifiers of the space ship, goes into the water purification system, and is turned in to drinking water: https://www.youtube.com/watch?v=8Hj3GnPRsJ4
http://explorecuriocity.org/Explore/ArticleId/6420/humidity-on-earth-and-on-the-iss.aspx
https://science.nasa.gov/science-news/science-at-nasa/2001/ast21mar_1

Regarding “showering” in space: How many ounces of the water/soap combination are in their bags used for “showering”? How often do the astronauts wash off? Will they have to shower less on longer missions such as those to Mars? Will they be required to wear their clothing for longer periods of time on longer missions?

NASA Astronauts conserve water so they only use a few ounces from a “NO RINSE BODY BATH POUCH ASSY” for each “shower” or “sponge bath” or better yet, “towel bath”. On the International Space Station, astronauts place a few ounces of the body bath solution on a towel and use the towel to wipe their entire body. Astronauts bathe often, about once a day, because they do vigorous exercise and work hard all day long. Although not as refreshing as Earth baths, Space baths are certainly refreshing and reenergizing; also, good hygiene is necessary for the astronauts’ health. The Smithsonian has a great article on “How to Shower in Space” and can be found at https://airandspace.si.edu/stories/editorial/how-shower-space . Also, Karen (Nyberg), who also mentored a FIRST team, the Robonauts, before becoming an astronaut, shows how she washes her hair in space at https://www.nasa.gov/content/how-do-you-wash-your-hair-in-space . Mike (Fossum) shows all about taking showers in space at https://youtu.be/tDbbJWKKQu0 . For longer missions, Astronauts will bathe at similar intervals, most often daily. For bathing on The Moon and Mars, similar to Earth, gravity will help astronauts place water on their bath towel. Even in the Space Exploration Vehicle, which astronauts will be travelling and exploring in for a week or so, astronauts will take baths daily. Concerning clothing, at the International Space Station, Frank (De Winne) does a terrific job describing how often the astronauts change clothes at https://youtu.be/1JfqdBJDNZc ; astronauts wear pants and shirts for about 10 days and undergarments are typically changed every other day. For longer missions, it will be important to learn to wash clothes in space and NASA is working on various ways to clean clothes including placing clothes in a bag and cleaning the clothes by flowing air through the bag then cleaning the air. Michael (Ewert) and Frank (Jeng), along with their colleagues, did a great study on cleaning clothes in space entitled “Will Astronauts Wash Clothes on the Way to Mars?” that can be found at https://ntrs.nasa.gov/archive/nasa/casi.ntrs.nasa.gov/20150003039.pdf . So, concerning “Will they be required to wear their clothing for longer periods of time on longer missions?”, hopefully astronauts will be able to wear fresh clothes more often because cleaning clothes will be easier than sending clothes back to Earth to have them cleaned. THANK YOU for doing space exploration research. -Lucien.
Lucien Junkin — NASA Space Exploration Vehicle (SEV) Chief Engineer

Regarding how the lack of gravity affects an astronaut’s circulatory system: Would it be advantageous to utilize Intermittent Pneumatic Compression Devices to compress specific parts of the body while the astronaut is sleeping to stimulate circulation?What effect does a lack of gravity have on that sort of device? How would the flow of the astronaut’s blood in a zero gravity environment be affected?

Compression of an extremity actually pushes blood away from the extremity and into the central circulation. This would have the opposite effect of trying to increase the circulation in the lower extremities. It would also force blood into the upper thorax and head, and potentially worsen the intracranial pressure and high fluid load of the head and brain. A vacuum, such as the Lower Body Negative Pressure (note negative not positive pressure) might be a better option to draw the fluid away from the brain and thorax and into the extremities. Ultimately, nothing replaces gravity and stimulation quite like gravity and stimulation. If we had an unlimited budget, a large centrifuge would be ideal. But the lever arm would have to be large enough to give you enough gravity. You can calculate just how long the radius of that arm and thus how large the centrifuge would have to be by going to the website (https://www.artificial-gravity.com/sw/SpinCalc/). The difficulty with a large centrifuge is that it takes enormous mass, volume, and power (and money!!).
Dr. J.D. Polk — NASA Chief Health and Medical Officer

Adding to the idea of the Russian “Penguin Suit” and NASA’s “BioSuit”, what about adding muscle stimulation electrodes to an elastic suit to supplement exercise? What are some solutions, other than exercise, currently in use to prevent muscle loss and how effective are they?

We have looked at muscle stimulation, but I can’t recall anyone ever putting forward a concept of incorporating it in the suit. The difficulty with electrical stimulation is that you’d really have to crank up the electricity to get the equivalent effect of lifting weights or anaerobic exercise. That’s not to say that it’s not potentially additive (ie- lifting weights AND having a suit that stimulates the muscle). NASA has looked at exoskeleton assistive devices that could be strapped on to augment the strength of an astronaut, especially in the lower extremities. There are supplements, nutrition, electrical stimulation, bands to give resistance, but nothing quite replaces good old exercise. In addition to muscle strength, exercise helps with metabolism, immunity, circulation, sensory biofeedback and a host of other positive impacts.
Dr. J.D. Polk — NASA Chief Health and Medical Officer

Has NASA thought about using a dental vacuum pump to help astronauts clean their teeth without having to swallow the toothpaste? Have any astronauts experienced toothaches during space travel?

Without access to a sink astronauts currently have two choices to discard their toothpaste after brushing their teeth: swallowing their tooth paste or spitting it into a paper towel (or napkin). Up to this point most astronauts have chosen to swallow their tooth paste as it’s easier when compared to a napkin and the difficulty of discarding a wet napkin. Now I’m sure some of the astronauts would love the idea of a dental vacuum, but the size and weight of it at the sizes it is at is launch prohibitive. Now if there was enough desire by astronauts to have one there maybe the possibility of it in the future.
Just like on earth things can happen unexpectedly especially on longer duration flights. On one long-duration flight a cosmonaut (Yuri Romanenko) developed a toothache. It is reported that he endured the pain for two weeks before alerting others to the pain. There was nothing that would help the pain he was in. Doctors on earth suggested he keep his mouth filled with warm water often to reduce the pain. Yuri dealt with the pain another two weeks before he was able to go back to earth. After landing back on earth it was determined that a nerve had been exposed.
References:
https://www.quora.com/International-Space-Station-Is-it-ok-to-swallow-toothpaste-as-Chris-Hadfield-does-in-this-video
https://am.dental-tribune.com/news/interview-extracting-a-tooth-should-be-the-last-resort-in-space/
Drew Price — Robotics Alliance Project – Project Manager

If CPR (Cardiopulmonary Resuscitation) needed to be performed in space, how would that occur?

We have the Crew Medical Restraint System (CMRS). The crew can restrain an unconscious crew member to CMRS. In order to perform CPR, the other crew must push off the “ceiling.” Astronaut Samantha Cristoforetti has an image of the on orbit training. https://twitter.com/astrosamantha/status/482849360871309314
Lyndon Bridgwater — Aerospace Engineer @ JSC

How much do you sweat during the tense times of launching and docking?

> Mark Vande Hei: The suit- we’re connected to a ventilation system- so if we’re sweating a lot, it gets dried up…As long as the suit system is working. There were a lot of simulators where we sweated a ton.
> Joe Acaba: I think it’s a lot easier real-time. You’ve trained for so long, and you’re ready for it. You’re pretty prepared.
Source: https://www.youtube.com/watch?v=K0l1RJ4Aag8
Astronauts Joe Acaba and/or Mark Vande Hei

How long did it take for you to get used to 1G once you returned from space?

It varies for everyone.
> For Joe Acaba, it was his vestibular system (balance). After a day or so, he was better, but needed light assistance walking at first upon landing. It’s like walking around on land after being on a boat for a while, only more intense.
> For Mark Vande Hei, he felt pretty good, but had the feelings of soreness in his small muscles that assist with balance.
Source: https://www.youtube.com/watch?v=K0l1RJ4Aag8

Astronauts Joe Acaba and/or Mark Vande Hei

See video of Mark Kelly doing physical tests, including walking a straight line, after spending 1 year in space: https://www.theverge.com/2017/10/28/16549898/scott-kelly-pbs-exclusive-clip-year-in-space-walking-problems-zero-gravity


Q&A about Social Health of Astronauts during Spaceflight

What is the most challenging social problem an astronaut faces in space? Please describe how this problem is being mitigated.

The most challenging social problem of crew members is generalized as problems resulting from being in isolated, confined, and extreme environments, or ICE. Just like many problems, ICE is addressed in many, many different ways. At NASA, we refer to solutions that counteract the effects of space as “Countermeasures”. Katherine (O’Brien Bachman), Christian Ott, Lauren Leveton, and their colleagues at the NASA Johnson Space Center conducted a study entitled “Countermeasures to Mitigate the Negative Impact of Sensory Deprivation and Social Isolation in Long-Duration Space Flight” at https://ntrs.nasa.gov/archive/nasa/casi.ntrs.nasa.gov/20120002722.pdf. More about the challenges of social isolation faced by astronauts in space comes from a great study conducted with the NASA Johnson Space Center Crew Office that involved astronauts that worked at the International Space Station including surveys and excerpts from the astronauts journals entitled ” Behavioral Issues Associated With Long Duration Space Expeditions: Review and Analysis of Astronaut Journals” at https://ntrs.nasa.gov/archive/nasa/casi.ntrs.nasa.gov/20100026549.pdf . The references in each of these reports have even more terrific volumes of information about your question. THANK YOU for being part of space exploration. -Lucien
Lucien Junkin — NASA Space Exploration Vehicle (SEV) Chief Engineer

How do the astronauts keep mentally well on the International Space Station?

Astronauts train for a long time for their missions so they are able to stay mentally focused on their tasks. On Shuttle flights, the crews experienced long, long days, and hardly no time off. On the ISS, astronauts get more regular schedules, exercise, and have private quarters that is sound & light-proof for 8 hours a day. They can spend that time as they wish- talking with family, checking email, reading a book, sleeping, etc. Having a decent amount of down-time helps astronauts stay mentally-well for extended periods of time. Another morale boost is windows! Windows are a must!
Astronaut Mike Gernhardt — Astronaut; Associate Division Chief of ER (Software, Robotics, & Simulation Division) @ JSC

You find out, it’s what I term ‘What’s important to you, and what’s not important to you’ up here. Honestly, one of our biggest behavioral assets that we have up here is our exercise equipment. When those pieces of equipment go down, we try to fix them as fast as we can, because that’s important to us. We watch the nightly news every evening. Now, we kinda get it a day later than everybody else, but we like to keep in touch with what’s going on down there on Earth. We like to keep in touch with our families, so we have weekly video conferences with our families. We have voice-over internet phones, so I can call pretty much any cell phone on the planet at any time. It’s small things like that, HTV arriving… We know and we’re working hard to open that hatch tonight, because we’ve got goodies waiting in there for us. Foods that we haven’t had maybe in a few months that we’re really looking forward to. So those are just some small things, but what we found is that even when you eliminate even the small things, they make a huge impact psychologically. Source: https://www.youtube.com/watch?v=LVCW_ubmwJ0&index=2&list=PLTUZypZ67cdv9X4DxjUGvN4wfN6EdHTUo
Astronaut Serena M. Auñón-Chancellor (M.D.)

What would a crew do about one of their members being a threat to others (due to stress, adverse situations, etc)? (Especially when on a long-term mission with no option to come back early.)

NASA handles the concerns of astronaut sociability in several ways. The first is selecting the right people. As early as NASA astronaut selection, the psychiatrists and psychologists are evaluating candidates for those qualities that will make them resilient. If a crewmember had an acute episode on an exploration mission, we could use behavioral therapy and counseling, and we could use pharmaceutical measures if needed. But Shackelton discovered in his sea voyages that what helped most was giving the crew “purpose”. Purposeful work, exercise, and a concentration on the task and mission at hand, was the most important treatment and condition for the crews on long exploration missions.
Dr. J.D. Polk — NASA Chief Health and Medical Officer

Is there currently any monitoring of astronaut mood and psychological well-being? If so, how does that work? Would it be helpful for astronauts to wear a wristband that would monitor the astronaut’s mood & well-being, prompting them to change activities, or would perhaps change colors, play music, or give off an aroma?

The astronauts have check-up sessions with the NASA psychiatrist and psychologist on a periodic basis even when in space, via our telecommunications system with Mission Control. They also have methods to monitor their sleep (Actiwatch) and also fill out periodic questionnaires and journal to monitor their moods. But feeling connected to family, friends, and socialization with each other helps mood greatly. Exercise and fruitful work also impacts mood greatly. Even reading a good book and listening to music can help mood. Work, believe it or not, also helps mood sometimes. Think about the movie “The Martian” for a moment. The character in the movie had a distinct mood change after fruitful and purpose work and concentration on the mission and task (when Mark exclaims “I have officially colonized Mars”).
Side Resources:
http://nsbri.org/researches/optical-computer-recognition-of-stress-affect-and-fatigue-in-space-flight/
https://taskbook.nasaprs.com/publication/index.cfm?action=public_query_taskbook_content&TASKID=8388
https://psychology.msu.edu/students/independent-study/team-interaction-dynamics-nasa-research
Dr. J.D. Polk — NASA Chief Health and Medical Officer

Do astronauts suffer from depression, homesickness and isolation when they are in space for extended periods of time? If so, how do they deal with their homesickness?

I have not experienced depression and feelings of isolation, but others certainly have. The middle of the mission is the most difficult because the initial excitement of arriving in space has worn off, but the excitement of returning home to family, friends, and the accustomed life on Earth is still out of sight. Everyone deals with their feelings of homesickness in different ways.
Astronaut Mike Gernhardt — Astronaut; Associate Division Chief of ER (Software, Robotics, & Simulation Division) @ JSC

The isolation is real, I mean you’re separated from your family and friends and kind of control of your life for the 197 days we’ll be up here. That being said, we believe why we’re here is important, and so when you’re doing something you believe in and doing something that’s important, the challenges kinda seem inconsequential, as long as you can reach your goal. I think about pre-service teachers, that’s kinda the message I’d like to share with them. If they believe the work they’re doing is important, there’s gonna be challenges. Keep pluggin ahead until you reach your goal. Confinement… I get asked that question a lot. The Space Station is huge! It’s about the internal volume of a jumbo 747, and we only have 6 people up here, so there’s plenty of room for us to live and work and relax. And having ample windows makes a huge difference. It’s kind of interesting… during my time here, I feel like I’ve traveled to locations on the planet that I will never ever see in real life, flying over French Polynesia, I saw Antarctica, the Antarctican Peninsula, and I’ll probably never get there. But having that opportunity to just gaze at Earth and see new places from a vantage point of 240 miles up, confinement doesn’t really become a big deal. When I’m heading home, I’m coming home in a Soyuz. It’s a really small spacecraft, and I only have to be in there for a few hours, but that is a very confining space, and I’m glad I don’t have to do that for 197 days, but I guess we would figure out a way to make it work.
Source: https://www.youtube.com/watch?v=HPjzXZwlayg&index=2&list=PLTUZypZ67cdv9X4DxjUGvN4wfN6EdHTUo&t=4s
Astronaut Ricky Arnold

Re: Homesickness… Would a 4-D virtual reality experience make an astronaut feel better about being away from home?

A 4-D virtual reality experience would be awesome for the long Mars missions! That would be a powerful tool to help with mental stresses. Whether the virtual reality would simulate you sitting in your dining room with your family, scuba diving on vacation, etc, it would be helpful.
Astronaut Mike Gernhardt — Astronaut; Associate Division Chief of Software, Robotics, & Simulation Division @ JSC

Do astronauts use anything to relax before going to bed?

Personally, in order to help me relax in order to fall asleep, I write in a notebook every night, capturing the essence of what happened that day. This helps me decompress at the end of the day. Plus, I can look back and read it later. It takes me back to that specific experience that I otherwise may have forgotten.
Astronaut Mike Gernhardt — Astronaut; Associate Division Chief of ER (Software, Robotics, & Simulation Division) @ JSC

How do astronauts adjust to fame and media demands?

You’re living an experience that very few people can have. I like sharing that experience with kids. It’s really rewarding. Sometimes I lack time for interviews or to meet with people, but I try to make time for it. Astronauts understand that it’s part of the job. We embrace it.
Astronaut Mike Gernhardt — Astronaut; Associate Division Chief of ER (Software, Robotics, & Simulation Division) @ JSC

How do astronauts deal with mental strain/burn-out/not enough free time?

“Mental strain/burn-out/not enough free time” falls into the category of effects that spaceflight have on astronauts. In general, NASA refers to the problems resulting from being in isolated, confined, and extreme environments, as ICE. Just like many problems, ICE is addressed in many, many different ways. At NASA, we refer to solutions, or “How do astronauts deal with …”, that counteract the effects of space as “Countermeasures”. Katherine (O’Brien Bachman), Christian Ott, Lauren Leveton, and their colleagues at the NASA Johnson Space Center conducted a study entitled “Countermeasures to Mitigate the Negative Impact of Sensory Deprivation and Social Isolation in Long-Duration Space Flight” at https://ntrs.nasa.gov/archive/nasa/casi.ntrs.nasa.gov/20120002722.pdf . Another great study conducted with the NASA Johnson Space Center Crew Office that involved astronauts that worked at the International Space Station including surveys and excerpts from the astronauts journals entitled “Behavioral Issues Associated With Long Duration Space Expeditions: Review and Analysis of Astronaut Journals” at https://ntrs.nasa.gov/archive/nasa/casi.ntrs.nasa.gov/20100026549.pdf . For example, from this report, here are a series of journal entries by different astronauts discussing frustrations:
“Sunday is meant to be a day of rest, but somehow Houston managed to make it feel like Monday. The problem is that I overreached myself a couple of weeks ago, feeling I was really helping out the ground to understand the problem with the ____.”
“The lack of “padding” in the schedule means that there is little time to accomplish small tasks, or to recover from mistakes.”
“Today was a hard day. Small things are getting to me. I am tired. I think that the ground is scheduling less time for tasks than before. So, there is very little, if any fat left in the schedule for me to use to catch up on little things during the day.”
“Only 30 minutes [were scheduled] to execute a 55‐step procedure that required collecting 21 items. It took 3 or 4 hours.”
“Skipped breakfast and finally made up the work and time. Otherwise it would have been a fairly nice pace today.”
“Several of the procedures, as usual, just took much longer than timelined. We have some tasks, as is too often the case, that were written without our input and which we never actually performed, except on paper.”
And for example, from this report, here are a series of journal entries by different astronauts discussing happiness:
“Getting email certainly is appreciated.”
“Loving the phone we have. It makes me feel closer to home.”
“Had a few wonderful phone calls that absolutely made my day. I made a couple of other calls today that were very upbeat and boosted the morale…”
“And the most rewarding tool here—the IP phone! What a treat to talk to family and friends!”
“What a treat to be able to blow them away with a call from space! It brings tears of joy to my eyes every time.”
“This journal would be really different, if we did not have so much ability for communications. We would fall into our routines, oblivious of all the bustle and gossip on Earth, and time would fly all the faster, never distracted to worry about whether someone is going to reply in a day or so! Comm is both a blessing and a curse. Maybe JPL scientists are better off in some ways. They have to treasure the meager flow of bits they receive, like I used to treasure the infrequent scented letters from girls, with long gaps of mystery and anticipation in between. Oh, the good old days when people wrote letters.”
The references in each of these reports have even more terrific volumes of information about your question. https://www.nasa.gov/centers/johnson/astronauts/journals_astronauts.html THANK YOU for being part of space exploration. -Lucien
Lucien Junkin — NASA Space Exploration Vehicle (SEV) Chief Engineer

In interviews, several astronauts on ISS have mentioned they miss being exposed to weather. Does having the ISS set to a constant temperature have any effect on the astronaut’s mood and productivity? Is the temperature or humidity on the ISS or spacecrafts ever varied? Do you think by simulating weather you could improve the astronaut’s productivity or mental health?

Unfortunately, simulating “weather” inside the ISS isn’t possible and it would actually be unsafe. The ISS contains a vast number of mechanical, electrical, and electronic devices. Producing tropical heat, winter cold, or rain would quickly destroy many systems, as well as jeopardize life support systems. As the ISS orbits Earth, the exterior temperatures go from plus 200F to minus 200F, so much of the work performed by the life support systems is to stabilize the environment. The internal conditions in the ISS are maintained at 72F to 78F at 40 to 70 percent humidity. An important component of maintaining a comfortable internal environment is keeping the air moving. The astronauts’ comments usually involve “sun on my face” or “wind blowing through the trees”, which is not possible on the ISS.
References:
1. https://www.nasa.gov/content/cooling-system-keeps-space-station-safe-productive
2. https://www.nasa.gov/sites/default/files/104840main_eclss.pdf
3. https://science.nasa.gov/science-news/science-at-nasa/2001/ast21mar_1/
Robert Kershaw — Electrical Engineer for NASA

What do astronauts do for fun in space? Do you watch TV?

The space station crews can ask mission control to send them shows that they can watch during dinner or off-duty time. They can also watch movies on their laptops. They may bring books, music, and musical instruments with them. Some astronauts enjoy hobbies, such as drawing, photography, and HAM radio. During missions, astronauts are very busy. The few hours of free time may also be spent looking out the window at the beautiful Earth below, listening to music, surfing the web, or corresponding with friends and family back home.
See links: https://www.youtube.com/watch?v=cobJzLQqUmo
We have a few toys up on the ISS. In fact, I think we’re the first crew to ever play a tennis match on the ISS or in space. My buddy, Drew, brought up some miniature tennis rackets. He only had two of them, and we played doubles, so two of us had to use ping pong paddles for tennis. And the fun thing about it was we decided early on that you could hit the ball on either side of the net, since there’s really no up or down up here. So as long as the ball went past the net, you were good to go.
Source: https://www.nasa.gov/centers/johnson/pdf/569954main_astronaut%20_FAQ
https://www.independent.co.uk/life-style/gadgets-and-tech/news/nasa-astronaut-space-tennis-international-station-video-us-open-a8503386.html
various astronauts, including Astronaut Ricky Arnold (talking about the tennis match on the ISS)

Can astronauts in space call their family? Do they have email?

Until a few years ago, we were not able to communicate easily with people on the ground outside of mission control. Now, we can send email to our friends and family directly, and we can make phone calls using a hook-up through our computer. The phone connection is really clear: You’d never know we were calling from space. Many astronauts carry pictures of their family with them on space missions.
Source: https://www.nasa.gov/centers/johnson/pdf/569954main_astronaut%20_FAQ.
various astronauts

Are you scared to fly in space?

A very experienced astronaut who has flown every space vehicle from Gemini to the shuttle once said that if you are not just a little afraid, you don’t know what’s happening. Astronauts are well aware of the risks involved in space travel. We are willing to take these risks because we believe in the mission of space exploration and we think the risk is well worth the benefit. Though these risks can make us nervous at times, when we are flying, we don’t let the nervousness interfere with doing our jobs.
Source: https://www.nasa.gov/centers/johnson/pdf/569954main_astronaut%20_FAQ.
various astronauts

Is Virtual Reality used in space? If so, how, and for what reasons?

The VR Lab team at JSC has flown a modified Oculus Rift to the ISS recently on SPX-14 that when used with the onboard HP Zbook laptop and SAFER (Simplified Aid For EVA Rescue) hand controller, provides an immersive onboard VR trainer for SAFER refresh training for astronauts before each EVA (Extra-Vehicular Activity).
https://www.houstonchronicle.com/techburger/article/NASA-s-virtual-reality-journey-uses-same-12745239.php
“Walk on Mars” VR combined with omnidirectional treadmill on Earth: https://www.extremetech.com/extreme/163189-walk-on-mars-using-the-oculus-rift-and-omni-treadmill
Mr. Eddie J. Paddock — VR Technical Discipline Lead @ JSC

In what ways does having plants on spacecraft help astronauts?

We hope to use plants to help supplement the packed diet to add vitamins and minerals to the diet and to provide variety and new flavors. Plants will also help recycle the atmosphere, generating oxygen and taking up CO2 that humans breath out. Taking care of plants may also be psychologically beneficial for the crew, reminding them of Earth and giving them a relaxing activity.
Gioia Massa — Life sciences project scientist @ NASA’s Kennedy Space Center


Q&A about Human Spaceflight Study, Work, & Training

What advice about school and coursework do you have for students who want to pursue a STEM degree or career? You gotta find what you love. Find that thing that makes you want to get out of bed in the morning and go to work, and work won’t seem like a job. Do not be intimidated by science and math. We know through our studies of the brain and how brain develop occurs that some kids… the skills for mathematics and science… typically don’t develop sometimes until after the kids are in middle school, when they really start to get the ability to understand some of the more abstract concepts. So stick with it. Don’t let people tell you you cannot pursue a degree in the STEM fields, particularly if it’s something you love. Your love of your work will provide enough motivation that you can accomplish anything you put your mind to.
Source: https://www.youtube.com/watch?v=HPjzXZwlayg&index=2&list=PLTUZypZ67cdv9X4DxjUGvN4wfN6EdHTUo&t=4s
Astronaut Ricky Arnold

How do astronauts and their mission controllers train for spaceflight? Flight controllers are selected for a specific flight control position in the Mission Control Center (MCC), such as the computer system or the robotic arm. Depending on the system they are selected for, they can train for several years to be certified to sit in MCC to operate the International Space Station (ISS) and support the astronauts. Their training includes classroom training, much like school, where they learn about the systems on the ISS, studying on their own, and training in simulations (kind of like a serious game) where they practice being a flight controller in a practice Mission Control room. The training simulations ensure that they have learned their system information and also that they can work as an effective team with other flight controllers. Astronauts also have a very long training program. When they are selected, they are considered astronaut candidates, or ASCANs. They receive initial training in their first two years at NASA, and they only become astronauts when they successfully compete their ASCAN training. Once they are assigned to a mission, they receive two more years of training for their specific mission. Much like flight controllers, astronaut training includes classroom training, self-study, and training simulations. Astronauts are trained to work together as an effective team, and they even go camping together to practice their leadership and team skills. What is different from flight controller training is that astronauts are trained on all of the systems on the ISS. They train for space walks in a large swimming pool called the neutral buoyancy lab (NBL), they train how to control ISS systems in a mockup of the Space Station, they train how to control the ISS robotic arm using hand controllers that are a bit like gaming hand controllers, they train how to fly spacecraft by practicing piloting skills in jet aircraft and practicing flying the spacecraft in a mockup, and they train to conduct research experiments in laboratories that have the same equipment they will use in space. Even though it is a lot of training, astronauts say the only thing better than being in space is training to be in space.
Dr. Sherry Thaxton — Human Health & Performance division @ JSC

What does it take to become an astronaut? Approximately 2 years are spent in initial astronaut training, learning the principles of aviation, how aircraft work & how to operate them, how spacecraft systems work & how to operate them, as well as basic science classes. Astronauts are in constant training for space flight. The initial training involves learning about basic space station systems, space walking, and operating the robotic arm. You continue this training while awaiting a mission assignment. Once assigned to a flight, specific training for your mission may take as long as three years. The length of training depends on how complicated the tasks are on your mission. ISS missions require that you speak Russian, which adds time to your training. NASA trains future astronauts in Expeditionary Skills including self-care and team-care, cultural competency, leadership and followership, teamwork, and communication. These skills prepare astronauts to live and work together on long duration spaceflights.
https://www.nasa.gov/audience/foreducators/stem-on-station/expeditionary-skills-for-life.html

What type of training do astronauts go through? Astronauts are in constant training for space flight. The initial training involves learning about basic space station systems, space walking, and operating the robotic arm. You continue this training while awaiting a mission assignment. Once assigned to a flight, specific training for your mission may take as long as three years. The length of training depends on how complicated the tasks are on your mission. ISS missions require that you speak Russian, which adds time to your training.
I went to school. When we were hired as astronauts, we go for about 2 years of kind of initial training, where we learned principles of aviation- how aircraft work & how to operate them, how spacecraft work & how to operate them, a lot of basic science classes, and also Russian language.
NASA trains us in expeditionary skills, including things like self-care/team-care, cultural competency, leadership/followership, teamwork, and communication. These skills prepare us to live and work together on long duration space flights.
Sources: FAQ article– https://www.nasa.gov/centers/johnson/pdf/569954main_astronaut%20_FAQ.pdf
Video interview– https://images.nasa.gov/details-Expedition_56_Education_Interview_with_McAuliffe_Shepard_Discovery_Center_2018_0809_688241.html
Video interview– https://www.nasa.gov/audience/foreducators/stem-on-station/expeditionary-skills-for-life.html
Astronaut Peggy Whitson & Expedition 56 Flight Engineer Ricky Arnold

What do you do on the International Space Station (ISS)?

Astronauts and cosmonauts on the space station stay busy. There’s lots of work to operate the many science experiments on board. The crew also has to make sure that the station is in top shape, so they clean, check equipment, maintain and repair or replace broken equipment. Crew members also must exercise two hours each day to stay fit and keep their bones and muscles strong. Sometimes we need to do a spacewalk to work outside the station in our spacesuit. It’s a tough and dangerous job but the view is terrific. The extra-vehicular activities, or EVAs, help keep the space station running.
Source: https://www.nasa.gov/centers/johnson/pdf/569954main_astronaut%20_FAQ.pdf
See this AWESOME virtual reality tour of the entire ISS, and all sorts of great information about the ISS: https://tommy.jsc.nasa.gov/igoal/ISSExplorer/
various astronauts

What should I study in school if I want to be an astronaut?

Astronauts come from many different backgrounds, all of them scientific or technical. You may be interested in being a jet pilot and a test pilot, which may qualify you to be an astronaut. In general, astronauts are scientists and engineers with a lot of education and broad experience. Find something you like, study hard, and do your best in school. You will have more choices in life if you always do your best.
Source: https://www.nasa.gov/centers/johnson/pdf/569954main_astronaut%20_FAQ.pdf
various astronauts

What do you do when you aren’t on a mission?

When astronauts are not flying on a mission or training for a mission, they support other missions. There are many jobs on the ground required to support the design, preparation, training and flying of a space mission. Astronauts work in mission control (the ‘voice’ that communicates with astronauts in orbit), check out procedures and the checklists the crew in space will use, help verify the space station and vehicle software, develop procedures and tools to be used during spacewalks or robotic operations, help scientists in developing experiments that will be run in space and perform other jobs in support of ISS and vehicle flights.
Source: https://www.nasa.gov/centers/johnson/pdf/569954main_astronaut%20_FAQ.pdf
various astronauts

What opportunities do you see for civilian physicians to go in to space? How would you advise medical students to prepare for a career like this?

This field is burgeoning, especially with commercial space. You’ve seen how NASA has already made it’s commercial crew assignments. We have several vehicles coming down the pike. All those vehicles, all those programs, are going to be utilizing physicians, or aerospace medicine specialists, in a way. So what I tell folks like yourself is to take advantage of clerkships, conferences, any sort of educational meetings you can get your hands on to learn about aerospace medicine. It’s not a widely recognized field. There’s only a few institutions in the U.S. that has this. Certainly the best way to get in to space right now, apply for the astronaut core. I think, honestly, in the next 5-10 years, you’re gonna be seeing people taking rides just sub-orbital flight, so not staying for a long period of time in Low Earth orbit, and I definitely see physicians on those flights coming up. So get yourself involved with those groups of people.
Source: https://www.youtube.com/watch?v=LVCW_ubmwJ0&index=2&list=PLTUZypZ67cdv9X4DxjUGvN4wfN6EdHTUo
Astronaut Serena M. Auñón-Chancellor (M.D.)

Describe the process of becoming an astronaut.

I think some people think you become an astronaut and then you fly just a couple years later. It’s a long process and a lot of my colleagues would agree with me. Once you get chosen as an astronaut, you’re officially known as an astronaut candidate at first, and you go through about a 2-3 year period of initial training. Because we’re in the era of space station, we had training on space station systems, how to operate the robot arm, and again, we used that today to capture the HTV7 vehicle. We learn how to do space walks, we train for space walks in the big Neutral Buoyancy Laboratory, which is our big pool, and we even take Russian language lessons. So a lot of our time consists of that the first couple of years to sort of create that framework and that base for how to operate on station. And then, you wait your turn. You wait your turn for a flight, and one of those days you get called in to the Chief’s office, and they say, ‘We’ve got a flight for you’, and you rejoice, and then it’s another 2 years or so of training to get ready for that mission specifically. And during that time you’re honing your skills, again, robot arm, space walks, ISS systems, but also about some of the specific science that’s gonna be going up during your mission. A lot of times you don’t learn about some of that science until you’re up here. It’s a long process. It’s very much worthwhile. It requires a lot of travel. You are overseas a lot during the training period.
Source: https://www.youtube.com/watch?v=LVCW_ubmwJ0&index=2&list=PLTUZypZ67cdv9X4DxjUGvN4wfN6EdHTUo
Astronaut Serena M. Auñón-Chancellor (M.D.)

How exactly did you decide you wanted to be an astronaut? Where were you in your education or career, and what sparked your interest?

So, I’ve gotta be honest, I’ve wanted to be an astronaut since I was little- probably 8 years old. I remember watching shuttle missions as a child, and my parents, my sisters, everybody had already encouraged me. I wasn’t quite sure what path I would take, and as I entered medical school at McGovern, I began thinking about what residency I wanted to go in to, and what I was gonna do, and I wasn’t sure. But during you’re fourth year you get the chance to do some away electives and some rotations, and I discovered a rotation at Johnson Space Center in Houston, Texas for Aerospace Medicine, and so I was able to take advantage of that. I really wasn’t quite sure what I was walking in to at the time, but that month was absolutely amazing. I learned about the changes that occur in the physiology of the human body, not only in differing environments, like high altitude environment and hyperbaric environments, but also, of course, in space. And that’s what helped lead me to go in to the residency that I chose, which was a combination of internal medicine and aerospace medicine. So I just kind waited for those doors to open, and they continued to open in my career to kinda point me in the way.
Source: https://www.youtube.com/watch?v=LVCW_ubmwJ0&index=2&list=PLTUZypZ67cdv9X4DxjUGvN4wfN6EdHTUo
Astronaut Serena M. Auñón-Chancellor (M.D.)

Can you talk about the importance of medicine in space, and how it relates to medicine on Earth?

Certainly people think about the medical changes that occur to the body when you’re in space, and those are many. There are things that we look at as we push further to The Moon and Mars. For example, your bones and muscles are immediately unloaded. There are changes that occur in your brain and eyes- we start to see some swelling of the optic disc and changes in the retina, even. But more importantly, folks on the ground want to know, ‘Why is the medicine you’re doing in space important for medicine on Earth? Why should I care about that?’ I think people should care because a lot of the experiments we’re doing up here could directly impact them on Earth, and I’ll give you a couple examples. One example I did just about a month ago was research looking at endothelial cells. It turns out endothelial cells up here in orbit really like to grow. So on Earth, they’re not as easy to culture. They don’t last for a very long period of time. But up here, endothelial cells feel like they’re at home. So we actually did some studies where we were testing chemotherapeutic agents on those endothelial cells, with the hopes of perhaps attacking a tumor’s vascular supply one day. Another good example is looking at the protein amyloid. Amyloid, as you know, is a protein that’s been implicated in Alzheimer’s disease and in other chronic disease processes. And again, up here in microgravity, the amyloid protein grows in 3-D, in a sense, and so by growing in that 3-D way, it gives scientists and researchers a better ability to view that protein, look at it’s shape, and perhaps create new targets for novel therapies. That’s just a couple of the examples. I could go on and on about this all day long, but I do want folks to walk away with one message, and that’s… A lot of the science we’re doing up here definitely helps to better the lives of folks down on Earth.
Source: https://www.youtube.com/watch?v=LVCW_ubmwJ0&index=2&list=PLTUZypZ67cdv9X4DxjUGvN4wfN6EdHTUo
Astronaut Serena M. Auñón-Chancellor (M.D.)

Can you share some of your experiences surrounding sanitation, as you were getting ready to fly up to the ISS?

> Joe Acaba: We do go in to quarantine for a couple of weeks before we launch to ensure we don’t take anything up there with us. Once you’re up there [on the ISS], you’re in a pretty good and sterile environment in terms of colds and the flu. It takes a lot of work not only protecting us before we get there, but we spend a lot of time cleaning up the space station, as well. Every Saturday morning- vacuuming and cleaning up, just like at home.
Source: https://www.youtube.com/watch?v=K0l1RJ4Aag8
Astronaut Joe Acaba

What do you do when you return to Earth from a mission in space?

Ricky Arnold w/ Expedition 56 coming to a close: “I have my plans, NASA certainly has plans for me for the next 45 days, and most of that will be involved in rehabilitation. We’re kind of living science experiments ourselves up here. I talked about the work we’re doing to prepare for exploring the solar system, so I’ll have lots of appointments with doctors around the Clear Lake area and at NASA. And then also debriefing, talking about what we learned on this mission, how we can do things better, how I could’ve done things better, … So it’ll be a busy couple of months once I get back. After that, there’s plenty of work at NASA, and I’m really excited to get back and be a part of this amazing organization that makes this facility possible.”
Source: https://www.youtube.com/watch?v=HPjzXZwlayg&index=2&list=PLTUZypZ67cdv9X4DxjUGvN4wfN6EdHTUo&t=4s
Astronaut Ricky Arnold

What are the chances of Astronauts returning home from Mars? Is it most likely going to be a one way trip?

NASA absolutely plans for the Mars-bound astronauts to return to Earth. Manned missions to Mars have not yet been fully planned, so I don’t have any details to share with you on those missions, but NASA should be determining and releasing that information within the next few years. The current plan is for the astronauts to spend approximately 6 months journeying to Mars (when Earth and Mars are aligned), spend a year and a half of so on Mars, and then another six months journeying back. This photo (https://www.space.com/16875-how-far-away-is-mars.html) shows why some times are much better than others to journey between Earth and Mars (for the sake of elapsed time and resources). The elliptical shape and speed of Mars’ orbit determines that timing.

What does NASA do to ensure samples brought to Earth from outer space do not contain organisms that could endanger the human race?

Check out this source on your question about samples brought to Earth from space: https://www.lpi.usra.edu/pss/presentations/200803/04-Atlas-PPSonMSR.pdf

Concerning launch training: How often do evacuation drills occur during training? Regarding an evacuation on the launch pad, what is the most difficult aspect of the protocol to carry out and why? What types of scenarios might call for an evacuation while on the launch pad?

https://www.nasa.gov/missions/shuttle/pad_drill.html
https://www.nasa.gov/mission_pages/shuttle/flyout/launchpadflyout.html
https://en.wikipedia.org/wiki/Advanced_Crew_Escape_Suit
Even though this is not a NASA-affiliated site, the pre-launch evacuation of a space shuttle (obviously the process would be different in Kazakhstan for the currently-used Soyuz), RAP has been told the description found on this page is accurate: http://americandigest.org/mt-archives/american_studies/how_to_get_out.php



Check out NASA Johnson Space Center’s “Houston, We Have a Podcast”!

https://www.nasa.gov/johnson/HWHAP

There are 20+ other NASA podcasts available from that webpage, as well.

Videos of Astronauts Onboard the ISS Answering Live Questions

Expedition 56 – Astronaut Dr. Serena Auñón-Chancellor
Public Affairs Office Educational Event with Poudre High School
September 12, 2018

Questions Covered:

  • What is the training like before you to the Space Station? What is the most difficult part of astronaut training?
  • What is it like to sleep in space? Have you or others experienced sleepwalking in microgravity?
  • What does your daily routine look like?
  • What type of experiments do you do in space? What makes them important?
  • Do you ever bring animals into space? Is it true birds could not survive in a microgravity environment?
  • How do you wash your clothes?
  • What is the most difficult task to do on the Space Station?
  • How do you and your fellow astronauts stay entertained?
  • Has your understanding of the cosmos changed since you’ve been in space.
  • Can you see the effects of climate change from the ISS?
  • I imagine the views you have from the Space Station are incredible. Have you ever been able to witness a meteor shower or Aurora Borealis during your time up there?
  • What is different about actual spacewalks as opposed to what you see in Hollywood?
  • I would love to hear about your experience as a Poudre High School student. How did Poudre prepare you for your career?
  • Does microgravity affect your digestion?
  • Can you put on makeup in space?
  • Can you please explain how similar or different training in the Neutral Buoyancy Lab compares to when you conduct a spacewalk on the ISS?

Expedition 52 – Flight Engineer Jack Fisher
Space Station Crew Member Discusses Life in Space with Georgia Students
June 19, 2017

Questions Covered:

  • Will you go to Mars and how long will you stay in space?
  • What cool things do you see in space that you do not see in Earth?
  • What are some of the most surprising personal adjustments did you or other astronauts face from beginning a stint in the ISS?
  • Would you like to go to the Moon?
  • How long can you breathe in space with your oxygen tank?
  • What do you eat? Is there a certain diet?
  • Have you seen a shooting star? If you have, what does it look like up close?
  • How do you sleep?
  • Because water floats in space, how do you wash up? Do you have a bathtub or a shower?
  • How long did you have to stay in school to become an Astronaut?
  • Do the astronauts ever get sick since they shouldn’t be exposed to germs up there?
  • What do you do when you are not in space?
  • What is one thing you were not prepared for as an astronaut?
  • What are you currently reading?
  • What are your three favorite things to do when you are in space?
  • What people and space vehicles have you flown into outer space?
  • Communication between Earth and the Space Station only has a few seconds of delay. How will communication systems change as you explore deep space?
  • What kind of weather do you see when observing Earth?
  • Is it true that time is slower in space?
  • What method of oxygen creation do you use? How much water is needed? And how much does it yield?

Expedition 56 – Flight Engineer Ricky Arnold
Public Affairs Office Educational Event with Microsoft Young Spark
August 13, 2018

Questions Covered:

  • What are some of the Pro’s & Con’s of being an astronaut?
  • How old were you when you first became interested in STEM? What cause you to have this interest?
  • What difficulties have you encountered when conducting experiments in space, as compared to on Earth?
  • What obstacles have you encountered while being in space?
  • How does gravity affect what you do on the space station, as well as the speed of the space station?
  • What piece of technology do you utilize daily that you could not live in space without?
  • What is it like aboard the Space Station?
  • Are you able to breathe normally, or is there oxygen on the station? If so, where does the oxygen come from?
  • What type of music do you listen to in space?
  • Where do you sleep and how do you sleep on the Space Station?
  • What do you eat when you are in space?
  • Where do you store your food supply?
  • Do you feel heavy while wearing a space suit?
  • What type of mental training do y’all receive before entering in to space?
  • What is the scariest moment you have encountered while being in space?
  • What was the most surprising aspect of executing a spacewalk?
  • How are rocket ships created, and how long does it normally take?
  • When traveling through space, where do the parts of the rocket that fall off go?
  • What do you expect to see changing in spaceflight in the coming years?
  • What types of jobs are available with NASA? How do you prepare for a career at NASA?

Expedition 56 – Flight Engineer Ricky Arnold
Public Affairs Office Education Interview with McAuliffe Shepard Discovery Center
August 9, 2018

Questions Covered:

  • How did you train to become an astronaut?
  • How do you keep from making a mess when you brush your teeth?
  • Next year will be the 50th anniversary of the Lunar Landing. What do you dream of us accomplishing in the next 50 years?
  • What do you do all day?
  • What experiments do you do on the Space Station?
  • Besides the Space Station itself, what is the most interesting piece of technology on board the ISS?
  • Is time different on the Space Station?
  • What is it like being weightless in space and is it hard to move around?
  • What was your inspiration to be an astronaut?
  • What are the three things you miss most from Earth?
  • What about the ISS will you miss the most when you return to Earth?
  • What is your favorite space food?
  • How do you sleep on the Space Station?
  • How do you prepare for a spacewalk, especially a long one?
  • What is the coolest or weirdest thing you have seen while on the Space Station?
  • How was the International Space Station constructed and what was it like installing solar array wings and truss?
  • How long does it take to get used to microgravity?
  • Will people make it to Mars?
  • If you could have one thing on the Space Station that you aren’t allowed to have, what would it be?
  • Since you are in space, can we assume you are a science geek? What else do you “geek out” about?
  • Was it scary traveling into space?

Expedition 56 – Astronaut Serena Auñón-Chancellor
Interview with McGovern Medical School in Houston, TX
September 27, 2018

Questions Covered:

  • Can you talk about the importance of medicine in space, and how it relates to medicine on Earth?
  • How exactly did you decide you wanted to be an astronaut? Where were you in your education or career, and what sparked your interest?
  • Describe the process of becoming an astronaut.
  • What was it like going in to space for the first time?
  • Does seeing Earth from space give you a new appreciation for how special our world is? With that in mind, do you think we will ever find life on another world?
  • Is there any special thing astronauts do to keep their mental well-being in space?
  • What are the major concerns about the health effects of your time in space?
  • How much exercise do you do each day, and how much of each, for example ARED vs. aerobic?
  • Are you specifically using any pharmacological countermeasures?
  • What biomedical research are you participating in?
  • What opportunities do you see for civilian physicians to go in to space? How would you advise medical students to prepare for a career like this?

NASA Administrator, Jim Bridenstine, chats with Astronauts Joe Acaba and Mark Vande Hei
July 25, 2018

Questions Covered:

  • What do astronauts do in the Soyuz capsule while you’re on your way to joining the ISS?
  • How long did it take for you to get used to 1G once you got back from space?
  • What is the least known fact about the ISS?
  • Can you see space debris, and what do you do about it?
  • Can you share some of your experiences surrounding sanitation, as you were getting ready to fly up to the ISS?
  • When you launch on a Soyuz, is it smooth, harsh? Do you remember much of it?
  • How much do you sweat during the tense times of launching and docking?
  • Both of you have been educators in your day. Did you use Algebra up there (in space)?

Expedition 56 – Flight Engineer Ricky Arnold
Year of “Education on Station” Wrap-up Show
October 2, 2018

Questions Covered:

  • What unique perspective do you feel that you brought to the table for your mission as someone with a background in education?
  • As you wrap up the Year of Education on Station, what is the one thing you want students to know about the science being conducted onboard the orbiting laboratory?
  • Has your philosophy on education changed since you’ve been onboard the ISS?
  • When did you first decide you were interested in STEM?
  • What type of technology is allowing us to talk to you right now?
  • What advice would you give to new teachers?
  • What advice about school and coursework do you have for students who want to pursue a STEM degree or career?
  • What are your plans when you get back to Earth?
  • After all your time on station, how intuitive is it for you to move about, find things, and flow from task to task? Is there anything that is still challenging?
  • Do you have any toys on the Space Station?
  • What are some of the challenges during the transition to life on the ISS, especially in terms of confinement and isolation?
  • What is it like to step out of the airlock for a spacewalk?
  • What is it like to ride in a Soyuz?